Download as pdf or txt
Download as pdf or txt
You are on page 1of 66

Sequences, Series,

8 and Probability
320
8.1 Sequences and Series
8.2 Arithmetic Sequences and Partial Sums
8.3 Geometric Sequences and Series
8.4 The Binomial Theorem

0 30
8.5 Counting Principles
200 8.6 Probability
Section 8.1, Example 10
Population of the United States
kaarsten 2010/used under license from Shutterstock.com

569

Copyright 2011 Cengage Learning. All Rights Reserved. May not be copied, scanned, or duplicated, in whole or in part. Due to electronic rights, some third party content may be suppressed from the eBook and/or eChapter(s).
Editorial review has deemed that any suppressed content does not materially affect the overall learning experience. Cengage Learning reserves the right to remove additional content at any time if subsequent rights restrictions require it.
570 Chapter 8 Sequences, Series, and Probability

8.1 Sequences and Series

Sequences What you should learn


● Use sequence notation to write
In mathematics, the word sequence is used in much the same way as in ordinary
the terms of sequences.
English. Saying that a collection is listed in sequence means that it is ordered so that it
● Use factorial notation.
has a first member, a second member, a third member, and so on.
● Use summation notation to
Mathematically, you can think of a sequence as a function whose domain is the set
write sums.
of positive integers. Instead of using function notation, sequences are usually written
● Find sums of infinite series.
using subscript notation, as shown in the following definition.
● Use sequences and series to
model and solve real-life
Definition of Sequence problems.
An infinite sequence is a function whose domain is the set of positive integers. Why you should learn it
The function values Sequences and series are useful in
a1, a2, a3, a4, . . . , an, . . . modeling sets of values in order to
identify patterns. For instance,
are the terms of the sequence. If the domain of a function consists of the first n Exercise 122 on page 579 shows
positive integers only, then the sequence is a finite sequence. how a sequence can be used to
model the population of trout
in a lake.
On occasion, it is convenient to begin subscripting a sequence with 0 instead of 1
so that the terms of the sequence become
a0, a1, a2, a3, . . . .
The domain of the function is the set of nonnegative integers.

Example 1 Writing the Terms of a Sequence


Write the first four terms of each sequence.
a. an ⫽ 3n ⫺ 2 b. an ⫽ 3 ⫹ 共⫺1兲 n

Solution
a. The first four terms of the sequence given by
an ⫽ 3n ⫺ 2
are
a1 ⫽ 3共1兲 ⫺ 2 ⫽ 1 1st term

a2 ⫽ 3共2兲 ⫺ 2 ⫽ 4 2nd term

a3 ⫽ 3共3兲 ⫺ 2 ⫽ 7 3rd term

a4 ⫽ 3共4兲 ⫺ 2 ⫽ 10. 4th term

b. The first four terms of the sequence given by


an ⫽ 3 ⫹ 共⫺1兲n
are
a1 ⫽ 3 ⫹ 共⫺1兲1 ⫽ 3 ⫺ 1 ⫽ 2 1st term

a2 ⫽ 3 ⫹ 共⫺1兲 ⫽ 3 ⫹ 1 ⫽ 4 2
2nd term

a3 ⫽ 3 ⫹ 共⫺1兲3 ⫽ 3 ⫺ 1 ⫽ 2 3rd term

a4 ⫽ 3 ⫹ 共⫺1兲 ⫽ 3 ⫹ 1 ⫽ 4. 4
4th term

Now try Exercise 7.


Ljupco Smokovski 2010/used under license from Shutterstock.com

Copyright 2011 Cengage Learning. All Rights Reserved. May not be copied, scanned, or duplicated, in whole or in part. Due to electronic rights, some third party content may be suppressed from the eBook and/or eChapter(s).
Editorial review has deemed that any suppressed content does not materially affect the overall learning experience. Cengage Learning reserves the right to remove additional content at any time if subsequent rights restrictions require it.
Section 8.1 Sequences and Series 571

Example 2 Writing the Terms of a Sequence

Write the first five terms of the sequence given by


共⫺1兲n
an ⫽ .
2n ⫺ 1

Algebraic Solution Numerical Solution


The first five terms of the sequence are as follows. Set your graphing utility to sequence mode and enter the sequence.
Use the table feature to create a table showing the terms of the
共⫺1兲1 ⫺1
a1 ⫽ ⫽ ⫽ ⫺1 1st term sequence, as shown in Figure 8.1.
2共1兲 ⫺ 1 2 ⫺ 1
共⫺1兲2 1 1
a2 ⫽ ⫽ ⫽ 2nd term
2共2兲 ⫺ 1 4 ⫺ 1 3 Use the table
feature in ask
共⫺1兲3 ⫺1 1 mode.
a3 ⫽ ⫽ ⫽⫺ 3rd term
2共3兲 ⫺ 1 6 ⫺ 1 5
共⫺1兲4 1 1 Figure 8.1
a4 ⫽ ⫽ ⫽ 4th term
2共4兲 ⫺ 1 8 ⫺ 1 7
共⫺1兲5 ⫺1 1 So, you can estimate the first five terms of the sequence as follows.
a5 ⫽ ⫽ ⫽⫺ 5th term
2共5兲 ⫺ 1 10 ⫺ 1 9 u1 ⫽ ⫺1, u2 ⫽ 0.33333 ⬇ 13, u3 ⫽ ⫺0.2 ⫽ ⫺ 15,
Now try Exercise 17. u4 ⫽ 0.14286 ⬇ 17, and u5 ⫽ ⫺0.1111 ⬇ ⫺ 19

Simply listing the first few terms is not sufficient to define a unique sequence—the
nth term must be given. To see this, consider the following sequences, both of which
have the same first three terms.
1 1 1 1 1
, , , , . . . , n, . . .
2 4 8 16 2
1 1 1 1 6
, , , ,. . . , ,. . .
2 4 8 15 共n ⫹ 1兲共n ⫺ n ⫹ 6兲
2

Example 3 Finding the nth Term of a Sequence


Write an expression for the apparent nth term an of each sequence.
a. 1, 3, 5, 7, . . .
b. 2, 5, 10, 17, . . .

Solution
a. n: 1 2 3 4 . . . n
Terms: 1 3 5 7 . . . an
Apparent Pattern: Each term is 1 less than twice n. So, the apparent nth term is
an ⫽ 2n ⫺ 1.
b. n: 1 2 3 4 . . . n
Terms: 2 5 10 17 . . . an
Apparent Pattern: Each term is 1 more than the square of n. So, the apparent nth
term is
an ⫽ n 2 ⫹ 1.
Now try Exercise 39.

Copyright 2011 Cengage Learning. All Rights Reserved. May not be copied, scanned, or duplicated, in whole or in part. Due to electronic rights, some third party content may be suppressed from the eBook and/or eChapter(s).
Editorial review has deemed that any suppressed content does not materially affect the overall learning experience. Cengage Learning reserves the right to remove additional content at any time if subsequent rights restrictions require it.
572 Chapter 8 Sequences, Series, and Probability

Some sequences are defined recursively. To define a sequence recursively, you


need to be given one or more of the first few terms. All other terms of the sequence are
then defined using previous terms. Technology Tip
To graph a sequence
Example 4 A Recursive Sequence using a graphing
utility, set the mode
A sequence is defined recursively as follows. to dot and sequence and enter
a1 ⫽ 3, ak ⫽ 2ak⫺1 ⫹ 1, where k ⱖ 2 the sequence. Try graphing
the sequence in Example 4
Write the first five terms of this sequence. and using the trace feature
Solution to identify the terms. For
instructions on how to use
a1 ⫽ 3 1st term is given. the sequence mode, see
Appendix A; for specific
a2 ⫽ 2a2⫺1 ⫹ 1 ⫽ 2a1 ⫹ 1 ⫽ 2(3) ⫹ 1 ⫽ 7 Use recursion formula.
keystrokes, go to this
a3 ⫽ 2a3⫺1 ⫹ 1 ⫽ 2a2 ⫹ 1 ⫽ 2(7) ⫹ 1 ⫽ 15 Use recursion formula. textbook’s Companion Website.
a4 ⫽ 2a4⫺1 ⫹ 1 ⫽ 2a3 ⫹ 1 ⫽ 2(15) ⫹ 1 ⫽ 31 Use recursion formula.

a5 ⫽ 2a5⫺1 ⫹ 1 ⫽ 2a4 ⫹ 1 ⫽ 2(31) ⫹ 1 ⫽ 63 Use recursion formula.

Now try Exercise 53.

In the next example you will study a well-known recursive sequence,


the Fibonacci sequence.

Example 5 The Fibonacci Sequence: A Recursive Sequence


The Fibonacci sequence is defined recursively as follows.
a0 ⫽ 1, a1 ⫽ 1, ak ⫽ ak⫺2 ⫹ ak⫺1, where k ⱖ 2
Write the first six terms of this sequence.

Solution
a0 ⫽ 1 0th term is given.

a1 ⫽ 1 1st term is given.

a2 ⫽ a2⫺2 ⫹ a2⫺1 ⫽ a0 ⫹ a1 ⫽ 1 ⫹ 1 ⫽ 2 Use recursion formula.

a3 ⫽ a3⫺2 ⫹ a3⫺1 ⫽ a1 ⫹ a2 ⫽ 1 ⫹ 2 ⫽ 3 Use recursion formula.

a4 ⫽ a4⫺2 ⫹ a4⫺1 ⫽ a2 ⫹ a3 ⫽ 2 ⫹ 3 ⫽ 5 Use recursion formula.

a5 ⫽ a5⫺2 ⫹ a5⫺1 ⫽ a3 ⫹ a4 ⫽ 3 ⫹ 5 ⫽ 8 Use recursion formula.

You can check this result using the table feature of a graphing utility, as shown in
Figure 8.2.

Figure 8.2

Now try Exercise 57.


4x6/iStockphoto.com

Copyright 2011 Cengage Learning. All Rights Reserved. May not be copied, scanned, or duplicated, in whole or in part. Due to electronic rights, some third party content may be suppressed from the eBook and/or eChapter(s).
Editorial review has deemed that any suppressed content does not materially affect the overall learning experience. Cengage Learning reserves the right to remove additional content at any time if subsequent rights restrictions require it.
Section 8.1 Sequences and Series 573

Factorial Notation
Some very important sequences in mathematics involve terms that are defined with Explore the Concept
special types of products called factorials.
Most graphing utilities
have the capability to
Definition of Factorial compute n!. Use your
If n is a positive integer, then n factorial is defined as graphing utility to compare
3 ⭈ 5! and 共3 ⭈ 5兲!. How do
n! ⫽ 1 ⭈ 2 ⭈3⭈4 . . . 共n ⫺ 1兲 ⭈ n. they differ? How large a value
As a special case, zero factorial is defined as 0! ⫽ 1. of n! will your graphing utility
allow you to compute?

Notice that 0! ⫽ 1 and 1! ⫽ 1. Here are some other values of n!.


2! ⫽ 1 ⭈ 2 ⫽ 2 3! ⫽ 1 ⭈ 2 ⭈3⫽6 4! ⫽ 1 ⭈ 2 ⭈ 3 ⭈ 4 ⫽ 24
The value of n does not have to be very large before the value of n! becomes huge. For
instance, 10! ⫽ 3,628,800.
Factorials follow the same conventions for order of operations as do exponents. For
instance,
⭈2⭈3⭈4. .
2n! ⫽ 2共n!兲 ⫽ 2共1 . n兲

whereas 共2n兲! ⫽ 1 ⭈ 2 ⭈ 3 ⭈ 4 . . . 2n.

Example 6 Writing the Terms of a Sequence Involving Factorials


2n
Write the first five terms of the sequence given by an ⫽ . Begin with n ⫽ 0.
n!

Algebraic Solution Graphical Solution


20 1 Using a graphing utility set to dot and sequence modes, enter the
a0 ⫽ ⫽ ⫽ 1 0th term
0! 1 sequence. Next, graph the sequence, as shown in Figure 8.3.
21 2
a1 ⫽ ⫽ ⫽2 1st term 4
1! 1
22 4
a2 ⫽ ⫽ ⫽2 2nd term Use the trace
2! 2 feature to
23 8 4 approximate the
a3 ⫽ ⫽ ⫽ 3rd term first five terms. 0 6
3! 6 3 0

24 16 2 Figure 8.3
a4 ⫽ ⫽ ⫽ 4th term
4! 24 3
You can estimate the first five terms of the sequence as follows.
u0 ⫽ 1, u1 ⫽ 2, u2 ⫽ 2,
4
Now try Exercise 63. u3 ⬇ 1.333 ⬇ 3, u4 ⬇ 0.667 ⬇ 23

Example 7 Simplifying Factorial Expressions


Study Tip
1⭈2
8! ⭈ 3 ⭈ 4 ⭈ 5 ⭈ 6 ⭈ 7 ⭈ 8 ⫽ 7 ⭈ 8 ⫽ 28
a. ⫽
2! ⭈ 6! 1 ⭈ 2 ⭈1⭈2⭈3⭈4⭈5⭈6 2 In Example 7(a), you
can also simplify the
n! 1 ⭈ 2 ⭈ 3. . .共n ⫺ 1兲 ⭈ n computation as
b. ⫽ ⫽n
共n ⫺ 1兲! 1 ⭈ 2 ⭈ 3. . .共n ⫺ 1兲
8! 8 ⭈ 7 ⭈ 6!
⫽ ⫽ 28.
Now try Exercise 73. 2! ⭈ 6! 2! ⭈ 6!

Copyright 2011 Cengage Learning. All Rights Reserved. May not be copied, scanned, or duplicated, in whole or in part. Due to electronic rights, some third party content may be suppressed from the eBook and/or eChapter(s).
Editorial review has deemed that any suppressed content does not materially affect the overall learning experience. Cengage Learning reserves the right to remove additional content at any time if subsequent rights restrictions require it.
574 Chapter 8 Sequences, Series, and Probability

Summation Notation
There is a convenient notation for the sum of the terms of a finite sequence. It is called Study Tip
summation notation or sigma notation because it involves the use of the uppercase
Greek letter sigma, written as 兺. Summation notation is
an instruction to add
the terms of a
Definition of Summation Notation sequence. From the definition
The sum of the first n terms of a sequence is represented by at the left, the upper limit of
summation tells you where to
n

兺a ⫽ a ⫹ a2 ⫹ a3 ⫹ a4 ⫹ . . . ⫹ an end the sum. Summation


i 1
i⫽1 notation helps you generate the
appropriate terms of the
where i is called the index of summation, n is the upper limit of summation,
sequence prior to finding the
and 1 is the lower limit of summation.
actual sum, which may be
unclear.

Example 8 Sigma Notation for Sums


5
a. 兺 3i ⫽ 3共1兲 ⫹ 3共2兲 ⫹ 3共3兲 ⫹ 3共4兲 ⫹ 3共5兲
i⫽1

⫽ 3共1 ⫹ 2 ⫹ 3 ⫹ 4 ⫹ 5兲
⫽ 3共15兲
⫽ 45
6
b. 兺 共1 ⫹ k 兲 ⫽ 共1 ⫹ 3 兲 ⫹ 共1 ⫹ 4 兲 ⫹ 共1 ⫹ 5 兲 ⫹ 共1 ⫹ 6 兲
k⫽3
2 2 2 2 2

⫽ 10 ⫹ 17 ⫹ 26 ⫹ 37
⫽ 90
8 1 1 1 1 1 1 1 1 1 1
c. 兺 n! ⫽ 0! ⫹ 1! ⫹ 2! ⫹ 3! ⫹ 4! ⫹ 5! ⫹ 6! ⫹ 7! ⫹ 8!
n⫽0

1 1 1 1 1 1 1
⫽1⫹1⫹ ⫹ ⫹ ⫹ ⫹ ⫹ ⫹
2 6 24 120 720 5040 40,320
⬇ 2.71828
For the summation in part (c), note that the sum is very close to the irrational number
e ⬇ 2.718281828. It can be shown that as more terms of the sequence whose nth term
is 1兾n! are added, the sum becomes closer and closer to e. Study Tip
Now try Exercise 87. Variations in the upper
and lower limits of
In Example 8, note that the lower limit of a summation does not have to be 1. Also summation can
note that the index of summation does not have to be the letter i. For instance, in part produce quite different-looking
(b), the letter k is the index of summation. summation notations for the
same sum. For example, the
following two sums have
Properties of Sums (See the proofs on page 632.) identical terms.
n n n 3
1.
i⫽1
兺 c ⫽ cn, c is a constant. 2. 兺
i⫽1
cai ⫽ c 兺a ,
i⫽1
i c is a constant.
兺 3共2 兲 ⫽ 3共2
i⫽1
i 1 ⫹ 22 ⫹ 23兲
n n n n n n

兺 共a ⫹ b 兲 ⫽ 兺 a ⫹ 兺 b 兺 共a ⫺ b 兲 ⫽ 兺 a ⫺ 兺 b
2
3.
i⫽1
i i
i⫽1
i
i⫽1
i 4.
i⫽1
i i
i⫽1
i
i⫽1
i
兺 3共2
i⫽0
i⫹1 兲 ⫽ 3共21 ⫹ 22 ⫹ 23兲

Copyright 2011 Cengage Learning. All Rights Reserved. May not be copied, scanned, or duplicated, in whole or in part. Due to electronic rights, some third party content may be suppressed from the eBook and/or eChapter(s).
Editorial review has deemed that any suppressed content does not materially affect the overall learning experience. Cengage Learning reserves the right to remove additional content at any time if subsequent rights restrictions require it.
Section 8.1 Sequences and Series 575

Series
Many applications involve the sum of the terms of a finite or an infinite sequence. Such
a sum is called a series.

Definition of a Series
Consider the infinite sequence a1, a2, a3, . . . , ai, . . . .
1. The sum of the first n terms of the sequence is called a finite series or the
partial sum of the sequence and is denoted by
n
a1 ⫹ a2 ⫹ a3 ⫹ . . . ⫹ an ⫽ 兺a .
i⫽1
i

2. The sum of all the terms of the infinite sequence is called an infinite series
and is denoted by

a1 ⫹ a2 ⫹ a3 ⫹ . . . ⫹ ai ⫹ . . . ⫽ 兺a .
i⫽1
i

Example 9 Finding the Sum of a Series


For the series Technology Tip
⬁ 3
兺 10
i⫽1
i
Most graphing utilities
are able to sum the
find (a) the third partial sum and (b) the sum. first n terms of a
sequence. Try using a graphing
Solution utility to confirm the results in
Example 8 and Example 9(a).
a. The third partial sum is
3
3 3 3 3
兺 10
i⫽1
i
⫽ ⫹ ⫹
101 102 103
3 3 3
⫽ ⫹ ⫹
10 100 1000
⫽ 0.3 ⫹ 0.03 ⫹ 0.003
⫽ 0.333.
b. The sum of the series is
⬁ 3 3 3 3 3 3
兺 10
i⫽1
i

10 1
⫹ 2⫹ 3⫹ 4⫹ 5⫹. . .
10 10 10 10
3 3 3 3 3
⫽ ⫹ ⫹ ⫹ ⫹ ⫹. . .
10 100 1000 10,000 100,000
⫽ 0.3 ⫹ 0.03 ⫹ 0.003 ⫹ 0.0003 ⫹ 0.00003 ⫹ . . .
⫽ 0.33333 . . .
1
⫽ .
3
Now try Exercise 117.

Notice in Example 9(b) that the sum of an infinite series can be a finite number.

Copyright 2011 Cengage Learning. All Rights Reserved. May not be copied, scanned, or duplicated, in whole or in part. Due to electronic rights, some third party content may be suppressed from the eBook and/or eChapter(s).
Editorial review has deemed that any suppressed content does not materially affect the overall learning experience. Cengage Learning reserves the right to remove additional content at any time if subsequent rights restrictions require it.
576 Chapter 8 Sequences, Series, and Probability

Application
Sequences have many applications in situations that involve recognizable patterns. One
such model is illustrated in Example 10.

Example 10 Population of the United States


From 1980 through 2008, the resident population of the United States can be
approximated by the model
an ⫽ 226.4 ⫹ 2.41n ⫹ 0.016n2, n ⫽ 0, 1, . . . , 28
where an is the population (in millions) and n represents the year, with n ⫽ 0
corresponding to 1980. Find the last five terms of this finite sequence. (Source: U.S.
Census Bureau)

Algebraic Solution Graphical Solution


The last five terms of this finite sequence are as follows. Using a graphing utility set to dot and sequence
modes, enter the sequence. Next, graph the sequence,
a24 ⫽ 226.4 ⫹ 2.41共24兲 ⫹ 0.016共24兲2 as shown in Figure 8.4.
⬇ 293.5 2004 population
320
a25 ⫽ 226.4 ⫹ 2.41共25兲 ⫹ 0.016共25兲2 Use the trace
feature to
⬇ 296.7 2005 population
approximate the
a26 ⫽ 226.4 ⫹ 2.41共26兲 ⫹ 0.016共26兲2 last five terms.

⬇ 299.9 2006 population 0 30


200
a27 ⫽ 226.4 ⫹ 2.41共27兲 ⫹ 0.016共27兲2 Figure 8.4
⬇ 303.1 2007 population
You can estimate the last five terms of the sequence as
a28 ⫽ 226.4 ⫹ 2.41共28兲 ⫹ 0.016共28兲2 follows.
⬇ 306.4 2008 population a24 ⬇ 293.5, a25 ⬇ 296.7, a26 ⬇ 299.9,
Now try Exercise 121. a27 ⬇ 303.1, a28 ⬇ 306.4

Explore the Concept


A 3 ⫻ 3 ⫻ 3 cube is created using 27 unit cubes (a unit cube has
a length, width, and height of 1 unit), and only the faces of the
cubes that are visible are painted blue (see Figure 8.5). Complete
the table below to determine how many unit cubes of the 3 ⫻ 3 ⫻ 3 cube
have no blue faces, one blue face, two blue faces, and three blue faces.
Do the same for a 4 ⫻ 4 ⫻ 4 cube, a 5 ⫻ 5 ⫻ 5 cube, and a 6 ⫻ 6 ⫻ 6 cube,
and record your results in the table below. What type of pattern do you
observe in the table? Write a formula you could use to determine the
column values for an n ⫻ n ⫻ n cube.
Figure 8.5
Number of blue
Cube faces 0 1 2 3

3 ⫻ 3 ⫻ 3

Copyright 2011 Cengage Learning. All Rights Reserved. May not be copied, scanned, or duplicated, in whole or in part. Due to electronic rights, some third party content may be suppressed from the eBook and/or eChapter(s).
Editorial review has deemed that any suppressed content does not materially affect the overall learning experience. Cengage Learning reserves the right to remove additional content at any time if subsequent rights restrictions require it.
Section 8.1 Sequences and Series 577

See www.CalcChat.com for worked-out solutions to odd-numbered exercises.


8.1 Exercises For instructions on how to use a graphing utility, see Appendix A.

Vocabulary and Concept Check


In Exercises 1–4, fill in the blank(s).
1. The function values a1, a2, a3, a4, . . . , an, . . . are called the _______ of a
sequence.
2. If you are given one or more of the first few terms of a sequence, and all other
terms of the sequence are defined using previous terms, then the sequence is
defined _______ .
n
3. For the sum 兺 a , i is called the _______ of summation, n is the _______ of
i⫽1
i

summation, and 1 is the _______ of summation.


4. The sum of the terms of a finite or an infinite sequence is called a ________ .

5. Which describes an infinite sequence? a finite sequence?


(a) The domain consists of the first n positive integers.
(b) The domain consists of the set of positive integers.
6. Write 1 ⭈ 2 ⭈ 3 ⭈ 4 ⭈ 5 ⭈ 6 in factorial notation.

Procedures and Problem Solving


Writing the Terms of a Sequence In Exercises 7–16, n⫹1 4n2
29. an ⫽ 1 ⫹ 30. an ⫽
write the first five terms of the sequence. (Assume n n n⫹2
begins with 1.)
31. an ⫽ 共⫺1兲n ⫹ 1 32. an ⫽ 共⫺1兲n⫹1 ⫹ 1
7. an ⫽ 2n ⫹ 5 8. an ⫽ 4n ⫺ 7
10. an ⫽ 共2 兲
1 n Writing an Indicated Term In Exercises 33–38, find the
9. an ⫽ 3n
11. an ⫽ 共⫺ 2 兲
1 n indicated term of the sequence.
12. an ⫽ 共⫺2兲n
n⫹1 n n2 n2
13. an ⫽ 14. an ⫽ 33. an ⫽ 34. an ⫽
n n⫹1 n2 ⫹1 2n ⫹ 1
n 2n a10 ⫽ 䊏 a5 ⫽ 䊏
15. an ⫽ 2 16. an ⫽
n ⫹1 n⫹1 35. an ⫽ 共⫺1兲n 共3n ⫺ 2兲 36. an ⫽ 共⫺1兲n⫺1关n共n ⫺ 1兲兴
a25 ⫽ 䊏 a16 ⫽ 䊏
Writing the Terms of a Sequence In Exercises 17–26, 2n 2n⫹1
write the first five terms of the sequence (a) using the 37. an ⫽ n 38. an ⫽ n
2 ⫹1 2 ⫹1
table feature of a graphing utility and (b) algebraically.
(Assume n begins with 1.) a6 ⫽ 䊏 a7 ⫽ 䊏

1 ⫹ 共⫺1兲n 1 ⫹ 共⫺1兲n Finding the nth Term of a Sequence In Exercises 39–52,


17. an ⫽ 18. an ⫽
n 2n write an expression for the apparent nth term of the
n sequence. (Assume n begins with 1.)
1 3
19. an ⫽ 1 ⫺ n 20. an ⫽ n
2 4 39. 1, 4, 7, 10, 13, . . . 40. 3, 7, 11, 15, 19, . . .
1 1 41. 0, 3, 8, 15, 24, . . .
1 1 1 1
42. 1, 4, 9, 16, 25, . . .
21. an ⫽ 22. an ⫽
n3兾2 冪n 2 3 4 5 6 2 3 4 5 6
43. 3, 4, 5, 6, 7, . . . 44. 1, 3, 5, 7, 9, . . .
共⫺1兲n
冢n ⫹ 1冣
n 1 ⫺1 1 ⫺1 1 2 4 8
23. an ⫽ 24. an ⫽ 共⫺1兲n 45. , ,
2 4 8 16 , , . . . 46. 3, ⫺ 9, 27, ⫺ 81, . . .
n2
47. 1 ⫹ 11, 1 ⫹ 12, 1 ⫹ 13, 1 ⫹ 14, 1 ⫹ 15, . . .
25. an ⫽ 共2n ⫺ 1兲共2n ⫹ 1兲 26. an ⫽ n共n ⫺ 1兲共n ⫺ 2兲 1 3 7 15 31
48. 1 ⫹ 2, 1 ⫹ 4, 1 ⫹ 8, 1 ⫹ 16, 1 ⫹ 32, . . .
Using a Graphing Utility In Exercises 27–32, use the 1 1 1 1 22 23 24 25
49. 1, , , , ,. . . 50. 1, 2, , , , ,. . .
table feature of a graphing utility to find the first 10 2 6 24 120 2 6 24 120
terms of the sequence. (Assume n begins with 1.) 51. 1, 3, 1, 3, 1, . . .
27. an ⫽ 2共3n ⫺ 1兲 ⫹ 5 28. an ⫽ 2n共n ⫹ 1兲共n ⫹ 2兲 52. 1, ⫺1, 1, ⫺1, 1, . . .

Copyright 2011 Cengage Learning. All Rights Reserved. May not be copied, scanned, or duplicated, in whole or in part. Due to electronic rights, some third party content may be suppressed from the eBook and/or eChapter(s).
Editorial review has deemed that any suppressed content does not materially affect the overall learning experience. Cengage Learning reserves the right to remove additional content at any time if subsequent rights restrictions require it.
578 Chapter 8 Sequences, Series, and Probability

A Recursive Sequence In Exercises 53–58, write the (c) 5 (d) 5

first five terms of the sequence defined recursively.


53. a1 ⫽ 28, ak⫹1 ⫽ ak ⫺ 4
54. a1 ⫽ 15, ak⫹1 ⫽ ak ⫹ 3 0 11 0 11
55. a1 ⫽ 3, ak⫹1 ⫽ 2共ak ⫺ 1兲 0 0

56. a1 ⫽ 32, ak⫹1 ⫽ 12ak 8 8n


77. an ⫽ 78. an ⫽
57. a0 ⫽ 1, a1 ⫽ 3, ak ⫽ ak⫺2 ⫹ ak⫺1 n⫹1 n⫹1
58. a0 ⫽ ⫺1, a1 ⫽ 1, ak ⫽ ak⫺2 ⫹ ak⫺1 4n
79. an ⫽ 4共0.5兲n⫺1 80. an ⫽
n!
Finding the nth Term of a Recursive Sequence In
Exercises 59–62, write the first five terms of the sequence Graphing the Terms of a Sequence In Exercises 81–86,
defined recursively. Use the pattern to write the nth use a graphing utility to graph the first 10 terms of the
term of the sequence as a function of n. (Assume n begins sequence. (Assume n begins with 1.)
with 1.) 2 4
81. an ⫽ n 82. an ⫽ 2 ⫺
59. a1 ⫽ 6, ak⫹1 ⫽ ak ⫹ 2 3 n
60. a1 ⫽ 25, ak⫹1 ⫽ ak ⫺ 5 83. an ⫽ 16共⫺0.5兲n⫺1 84. an ⫽ 8共0.75兲n⫺1
61. a1 ⫽ 81, ak⫹1 ⫽ 13ak 2n 3n2
85. an ⫽ 86. an ⫽ 2
62. a1 ⫽ 14, ak⫹1 ⫽ ⫺2ak n⫹1 n ⫹1

Writing the Terms of a Sequence Involving Factorials In Sigma Notation for Sums In Exercises 87–98, find the
Exercises 63– 68, write the first five terms of the sum.
sequence (a) using the table feature of a graphing utility 5 6
and (b) algebraically. (Assume n begins with 0.) 87. 兺 共2i ⫹ 1兲
i⫽1
88. 兺 共3i ⫺ 1兲
i⫽1
1 1 4 5
63. an ⫽ 64. an ⫽
n! 共n ⫹ 1兲! 89. 兺 10
k⫽1
90. 兺4
k⫽1
n! n2 4 5
65. an ⫽ 66. an ⫽
2n ⫹ 1 共n ⫹ 1兲! 91. 兺i
i⫽0
2
92. 兺 3i
i⫽0
2

共⫺1兲2n 共⫺1兲2n⫹1 3 1 5 1
67. an ⫽ 68. an ⫽
共2n兲! 共2n ⫹ 1兲! 93. 兺k
k⫽0
2 ⫹1
94. 兺j
j⫽3
4 5
Simplifying Factorial Expressions In Exercises 69–76,
simplify the factorial expression.
95. 兺 关共i ⫺ 1兲
i⫽1
2 ⫹ 共i ⫹ 1兲3兴 96. 兺 共k ⫹ 1兲共k ⫺ 3兲
k⫽2
4 4

69.
2!
70.
5! 97. 兺2
i⫽1
i
98. 兺 共⫺2兲
j⫽0
j

4! 7!
12! 10! ⭈ 3! Using a Graphing Utility In Exercises 99–102, use a
71. 72.
4! ⭈ 8! 4! ⭈ 6! graphing utility to find the sum.
共n ⫹ 1兲! 共n ⫹ 2兲! 6 10 3
73.
n!
74.
n! 99. 兺
j⫽1
共24 ⫺ 3j兲 100. 兺 j⫹1
j⫽1
共2n ⫺ 1兲! 共2n ⫹ 2兲! 共⫺1兲k
4 4 共⫺1兲k
75.
共2n ⫹ 1兲!
76.
共2n兲! 101. 兺
k⫽0 k ⫹ 1
102. 兺
k⫽0 k!

Identifying a Graph In Exercises 77–80, match the Writing a Sum Using Sigma Notation In Exercises
sequence with its graph. [The graphs are labeled (a), (b), 103–112, use sigma notation to write the sum. Then use
(c), and (d).] a graphing utility to find the sum.
(a) 12 (b) 8
1 1 1 1
103. ⫹ ⫹ ⫹. . .⫹
3共1兲 3共2兲 3共3兲 3共9兲
5 5 5 5
104. ⫹ ⫹ ⫹. . .⫹
0 11 0 11
1⫹1 1⫹2 1⫹3 1 ⫹ 15
0 0

Copyright 2011 Cengage Learning. All Rights Reserved. May not be copied, scanned, or duplicated, in whole or in part. Due to electronic rights, some third party content may be suppressed from the eBook and/or eChapter(s).
Editorial review has deemed that any suppressed content does not materially affect the overall learning experience. Cengage Learning reserves the right to remove additional content at any time if subsequent rights restrictions require it.
Section 8.1 Sequences and Series 579

105. 关2共 18 兲 ⫹ 3兴 ⫹ 关2共 28 兲 ⫹ 3兴 ⫹ . . . ⫹ 关2共 88 兲 ⫹ 3兴 (c) Use a graphing utility to find the number of trout
in the lake as time passes infinitely. Explain your result.
106. 关1 ⫺ 共 16 兲2兴 ⫹ 关1 ⫺ 共 26 兲2兴 ⫹ . . . ⫹ 关1 ⫺ 共 66 兲2兴
123. Business The revenues an (in millions of dollars) of
107. 3 ⫺ 9 ⫹ 27 ⫺ 81 ⫹ 243 ⫺ 729
Amazon.com from 2001 through 2008 are shown in
1 ⫺ 12 ⫹ 14 ⫺ 18 ⫹ . . . ⫺ 128
1
108. the bar graph. The revenues can be approximated by
1 1 1 1 1 the model
109. 2 ⫺ 2 ⫹ 2 ⫺ 2 ⫹ . . . ⫺ 2
1 2 3 4 20 an ⫽ 296.477n2 ⫺ 469.11n ⫹ 3606.2
1 1 1 1
110. ⫹ ⫹ ⫹. . .⫹ n ⫽ 1, 2, . . . , 8
1⭈3 2⭈4 3⭈5 10 ⭈ 12
1
⫹ 38 ⫹ 167
⫹ 15 31 where n is the year, with n ⫽ 1 corresponding to 2001.
111. 4 32 ⫹ 64
1 Use this model to approximate the total revenue from
112. 2 ⫹ 24 ⫹ 68 ⫹ 24 120 720
16 ⫹ 32 ⫹ 64 2001 through 2008. Compare this sum with the result
of adding the revenues shown in the bar graph.
Finding a Partial Sum In Exercises 113–116, find the
(Source: Amazon.com)
indicated partial sum of the series.
an
⬁ ⬁
兺 5共 兲 兺 2共 兲
1 i 1 i
113. 2 114. 3 21,000

(in millions of dollars)


i⫽1 i⫽1
18,000
Fourth partial sum Fifth partial sum 15,000

Revenue
⬁ ⬁
兺 4共 兲 兺 8共 兲
n n
12,000
115. ⫺ 12 116. ⫺ 14
n⫽1 n⫽1 9,000
6,000
Third partial sum Fourth partial sum
3,000
n
Finding the Sum of a Series In Exercises 117–120, find 1 2 3 4 5 6 7 8
(a) the fourth partial sum and (b) the sum of the infinite Year (1 ↔ 2001)
series.
⬁ ⬁ 124. MODELING DATA
兺 6共10 兲 兺 4共10 兲
1 i 1 k
117. 118.
i⫽1 k⫽1 The numbers Rn (in thousands) of registrations of
⬁ ⬁ publicly owned buses in the United States from 2003
兺共 兲 兺 2共 兲
1 k 1 i
119. 10 120. 10 through 2008 are shown in the table. (Source: U.S.
k⫽1 i⫽1
Federal Highway Administration)
121. Compound Interest A deposit of $5000 is made
in an account that earns 3% interest compounded Registrations, Rn
Year
quarterly. The balance in the account after n quarters is (in thousands)
given by 2003 452
2004 465
冢 冣
0.03 n
An ⫽ 5000 1 ⫹ , n ⫽ 1, 2, 3, . . . . 2005 476
4
2006 483
(a) Compute the first eight terms of this sequence.
2007 490
(b) Find the balance in this account after 10 years by 2008 493
computing the 40th term of the sequence.
122. (p. 570) A landlocked (a) Use a graphing utility to graph the data. Let n
lake has been selected to be stocked in the represent the year, with n ⫽ 3 corresponding to
year 2012 with 5500 trout, and to be 2003.
restocked each year thereafter with 500 trout. (b) Use the regression feature of the graphing utility to
Each year the fish population declines 25% find a linear sequence and a quadratic sequence
due to harvesting and other natural causes. that model the data. Identify the coefficient of
(a) Write a recursive sequence that gives determination for each model.
the population pn of trout in the lake in (c) Graph each model with the data. Decide which
terms of the year n since stocking began. model is a better fit for the data. Explain.
(b) Use the recursion formula from part (a)
(d) Use the model you chose in part (c) to predict the
to find the numbers of trout in the lake
number of bus registrations in 2015. Does your
in the years 2013, 2014, and 2015.
answer seem reasonable? Explain.
Ljupco Smokovski 2010/used under license from Shutterstock.com
Rob Wilson 2010/used under license from Shutterstock.com

Copyright 2011 Cengage Learning. All Rights Reserved. May not be copied, scanned, or duplicated, in whole or in part. Due to electronic rights, some third party content may be suppressed from the eBook and/or eChapter(s).
Editorial review has deemed that any suppressed content does not materially affect the overall learning experience. Cengage Learning reserves the right to remove additional content at any time if subsequent rights restrictions require it.
580 Chapter 8 Sequences, Series, and Probability

Conclusions 共⫺1兲n⫹1共x ⫹ 1兲n


141. an ⫽
n!
True or False? In Exercises 125 and 126, determine
whether the statement is true or false. Justify your 共⫺1兲 共x ⫺ 1兲n
n
142. an ⫽
answer. 共n ⫹ 1兲!
4 4 4
125. 兺 共i
i⫽1
2 ⫹ 2i兲 ⫽ 兺i
i⫽1
2 ⫹2 兺i
i⫽1
Writing Partial Sums In Exercises 143–146, write the
first five terms of the sequence. Then find an expression
4 6
126. 兺2 ⫽ 兺2
j⫽1
j
j⫽3
j⫺2 for the nth partial sum.
1 1
143. an ⫽ ⫺
2n 2n ⫹ 2
Fibonacci Sequence In Exercises 127 and 128, use the
Fibonacci sequence. (See Example 5.) 1 1
144. an ⫽ ⫺
n n⫹1
127. Write the first 12 terms of the Fibonacci sequence an
1 1
and the first 10 terms of the sequence given by 145. an ⫽ ⫺
n⫹1 n⫹2
an⫹1
bn ⫽ , n > 0. 1
146. an ⫽ ⫺
1
an n n⫹2
128. Using the definition of bn given in Exercise 127, show
147. Think About It Does every finite series whose terms
that bn can be defined recursively by
are integers have a finite sum? Explain.
1
bn ⫽ 1 ⫹ .
bn⫺1 148. C A P S T O N E The graph represents the first six
terms of a sequence.
Exploration In Exercises 129–132, let
an

an ⴝ
共1 ⴙ 冪5 兲n ⴚ 共1 ⴚ 冪5 兲 n 7
2n 冪5 (6, 6)
6
5
(5, 5)
be a sequence with nth term an . 4
(3, 3) (4, 4)
3
129. Use the table feature of a graphing utility to find the 2 (2, 2)
first five terms of the sequence. 1 (1, 1)
n
130. Do you recognize the terms of the sequence in 1 2 3 4 5 6 7
Exercise 129? What sequence is it?
131. Find expressions for an⫹1 and an⫹2 in terms of n. (a) Write the first six terms of the sequence.
132. Use the result from Exercise 131 to show that (b) Write an expression for the apparent nth term
an⫹2 ⫽ an⫹1 ⫹ an. Is this result the same as your an of the sequence.
answer to Exercise 129? Explain. (c) Use sigma notation to represent the partial sum
of the first 50 terms of the sequence.
A Sequence Involving x In Exercises 133–142, write the
first five terms of the sequence.
xn x2 Cumulative Mixed Review
133. an ⫽ 134. an ⫽
n! n2 Operations with Matrices In Exercises 149–152, find, if
共⫺1兲n x2n⫹1 共⫺1兲nxn⫹1 possible, (a) A ⴚ B, (b) 2B ⴚ 3A, (c) AB, and (d) BA.
135. an ⫽ 136. an ⫽
2n ⫹ 1 n⫹1
共⫺1兲nx2n
149. A ⫽ 冤63 54冥, B ⫽ 冤⫺26 ⫺34冥
137. an ⫽
共2n兲! 0 ⫺12
150. A ⫽ 冤 冥, B⫽冤
8 11冥
10 7
共⫺1兲nx2n⫹1 ⫺4 6
138. an ⫽
共2n ⫹ 1兲!

冤 冥 冤 冥
⫺2 ⫺3 6 1 4 2
共⫺1兲nxn 151. A ⫽ 4 5 7 , B⫽ 0 1 6
139. an ⫽
n! 1 7 4 0 3 1
共⫺1兲nxn⫹1

冤 冥 冤 冥
⫺1 4 0 0 4 0
140. an ⫽
共n ⫹ 1兲! 152. A ⫽ 5 1 2 , B⫽ 3 1 ⫺2
0 ⫺1 3 ⫺1 0 2

Copyright 2011 Cengage Learning. All Rights Reserved. May not be copied, scanned, or duplicated, in whole or in part. Due to electronic rights, some third party content may be suppressed from the eBook and/or eChapter(s).
Editorial review has deemed that any suppressed content does not materially affect the overall learning experience. Cengage Learning reserves the right to remove additional content at any time if subsequent rights restrictions require it.
Section 8.2 Arithmetic Sequences and Partial Sums 581

8.2 Arithmetic Sequences and Partial Sums

Arithmetic Sequences What you should learn


● Recognize, write, and find the
A sequence whose consecutive terms have a common difference is called an arithmetic
nth terms of arithmetic
sequence.
sequences.
● Find nth partial sums of
Definition of Arithmetic Sequence arithmetic sequences.
● Use arithmetic sequences to
A sequence is arithmetic when the differences between consecutive terms are
model and solve real-life
the same. So, the sequence
problems.
a1, a2, a3, a4, . . . , an, . . .
Why you should learn it
is arithmetic when there is a number d such that Arithmetic sequences can reduce the
a2 ⫺ a1 ⫽ a3 ⫺ a2 ⫽ a4 ⫺ a 3 ⫽ . . . ⫽ d. amount of time it takes to find the
sum of a sequence of numbers with
The number d is the common difference of the sequence. a common difference. In Exercise 83
on page 587, you will use an arith-
metic sequence to find the number
Example 1 Examples of Arithmetic Sequences of bricks needed to lay a brick patio.

a. The sequence whose nth term is


4n ⫹ 3
is arithmetic. The common difference between consecutive terms is 4.
7, 11, 15, 19, . . . , 4n ⫹ 3, . . . Begin with n ⫽ 1.

11 ⫺ 7 ⫽ 4

b. The sequence whose nth term is


7 ⫺ 5n
is arithmetic. The common difference between consecutive terms is ⫺5.
2, ⫺3, ⫺8, ⫺13, . . . , 7 ⫺ 5n, . . . Begin with n ⫽ 1.

⫺3 ⫺ 2 ⫽ ⫺5

c. The sequence whose nth term is

4 共n ⫹ 3兲
1

1
is arithmetic. The common difference between consecutive terms is 4.
5 3 7 n⫹3
1, , , , . . . , ,. . . Begin with n ⫽ 1.
4 2 4 4
5
4 ⫺ 1 ⫽ 14

Now try Exercise 11.

The sequence 1, 4, 9, 16, . . . , whose nth term is n2, is not arithmetic. The
difference between the first two terms is
a2 ⫺ a1 ⫽ 4 ⫺ 1 ⫽ 3
but the difference between the second and third terms is
a3 ⫺ a2 ⫽ 9 ⫺ 4 ⫽ 5.
Gordon Swanson 2010/used under license from Shutterstock.com

Copyright 2011 Cengage Learning. All Rights Reserved. May not be copied, scanned, or duplicated, in whole or in part. Due to electronic rights, some third party content may be suppressed from the eBook and/or eChapter(s).
Editorial review has deemed that any suppressed content does not materially affect the overall learning experience. Cengage Learning reserves the right to remove additional content at any time if subsequent rights restrictions require it.
582 Chapter 8 Sequences, Series, and Probability

The nth term of an arithmetic sequence can be derived from the pattern below.
a1 ⫽ a1 1st term Explore the Concept
a2 ⫽ a1 ⫹ d 2nd term
Consider the following
a3 ⫽ a1 ⫹ 2d 3rd term sequences.
a4 ⫽ a1 ⫹ 3d 4th term 1, 4, 7, 10, . . . , 3n ⫺ 2, . . .
a5 ⫽ a1 ⫹ 4d 5th term ⫺5, 1, 7, 13, . . . , 6n ⫺ 11, . . .
5 3 1 1 7
2, 2, 2, ⫺ 2, . . . , 2 ⫺ n, . . .
1 less
⯗ What relationship do you
an ⫽ a1 ⫹ 共n ⫺ 1兲 d nth term observe between successive
terms of these sequences?
1 less

The result is summarized in the following definition.

The nth Term of an Arithmetic Sequence


The nth term of an arithmetic sequence has the form
an ⫽ a1 ⫹ 共n ⫺ 1兲 d
where d is the common difference between consecutive terms of the sequence and
a1 is the first term of the sequence.

Example 2 Finding the nth Term of an Arithmetic Sequence


Technology Tip
Find a formula for the n th term of the arithmetic sequence whose common difference
is 3 and whose first term is 2. You can use a graphing
utility to generate the
Solution arithmetic sequence
You know that the formula for the nth term is of the form an ⫽ a1 ⫹ 共n ⫺ 1兲 d. in Example 2 by using the
Moreover, because the common difference is d ⫽ 3 and the first term is a1 ⫽ 2, the following steps.
formula must have the form 2 ENTER
an ⫽ 2 ⫹ 3共n ⫺ 1兲 Substitute 2 for a1 and 3 for d. 3 ⴙ ANS
Now press the enter key
or an ⫽ 3n ⫺ 1. The sequence therefore has the following form.
repeatedly to generate the terms
2, 5, 8, 11, 14, . . . , 3n ⫺ 1, . . . of the sequence.
A graph of the first 15 terms of the sequence is shown in Figure 8.6. Notice that the
points lie on a line. This makes sense because an is a linear function of n. In other
words, the terms “arithmetic” and “linear” are closely connected.

50

an = 3n − 1

0 15
0

Figure 8.6

Now try Exercise 21.

Copyright 2011 Cengage Learning. All Rights Reserved. May not be copied, scanned, or duplicated, in whole or in part. Due to electronic rights, some third party content may be suppressed from the eBook and/or eChapter(s).
Editorial review has deemed that any suppressed content does not materially affect the overall learning experience. Cengage Learning reserves the right to remove additional content at any time if subsequent rights restrictions require it.
Section 8.2 Arithmetic Sequences and Partial Sums 583

Example 3 Writing the Terms of an Arithmetic Sequence


The fourth term of an arithmetic sequence is 20, and the 13th term is 65. Write the first
several terms of this sequence.

Solution
You know that a4 ⫽ 20 and a13 ⫽ 65. So, you must add the common difference d nine
times to the fourth term to obtain the 13th term. Therefore, the fourth and 13th terms of
the sequence are related by
a13 ⫽ a4 ⫹ 9d. a4 and a13 are nine terms apart.

Using a4 ⫽ 20 and a13 ⫽ 65, you have


65 ⫽ 20 ⫹ 9d.
So, you can conclude that d ⫽ 5, which implies that the sequence is as follows.
a1 a2 a3 a4 a5 a6 a7 a8 a9 a10 a11 a12 a13 . . .
5, 10, 15, 20, 25, 30, 35, 40, 45, 50, 55, 60, 65, . . .
Now try Exercise 35.

When you know the nth term of an arithmetic sequence and you know the common
difference of the sequence, you can find the 共n ⫹ 1兲th term by using the recursion
formula
Technology Tip
an⫹1 ⫽ an ⫹ d. Recursion formula
Most graphing utilities
With this formula, you can find any term of an arithmetic sequence, provided that you have a built-in function
know the preceding term. For instance, when you know the first term, you can find the that will display the
second term. Then, knowing the second term, you can find the third term, and so on. terms of an arithmetic
sequence. For instructions
Example 4 Using a Recursion Formula on how to use the sequence
feature, see Appendix A; for
Find the ninth term of the arithmetic sequence whose first two terms are 2 and 9. specific keystrokes, go to this
textbook’s Companion Website.
Solution
You know that the sequence is arithmetic. Also, a1 ⫽ 2 and a2 ⫽ 9.
So, the common difference for this sequence is
d ⫽ 9 ⫺ 2 ⫽ 7.
There are two ways to find the ninth term. One way is simply to
write out the first nine terms (by repeatedly adding 7).
2, 9, 16, 23, 30, 37, 44, 51, 58
Another way to find the ninth term is to first find a formula for the nth term. Because
the common difference is d ⫽ 7 and the first term is a1 ⫽ 2, the formula must have the
form
an ⫽ 2 ⫹ 7共n ⫺ 1兲. Substitute 2 for a1 and 7 for d.

Therefore, a formula for the nth term is


an ⫽ 7n ⫺ 5
which implies that the ninth term is
a9 ⫽ 7共9兲 ⫺ 5 ⫽ 58.
Now try Exercise 43.
Edyta Pawlowska 2010/used under license from Shutterstock.com

Copyright 2011 Cengage Learning. All Rights Reserved. May not be copied, scanned, or duplicated, in whole or in part. Due to electronic rights, some third party content may be suppressed from the eBook and/or eChapter(s).
Editorial review has deemed that any suppressed content does not materially affect the overall learning experience. Cengage Learning reserves the right to remove additional content at any time if subsequent rights restrictions require it.
584 Chapter 8 Sequences, Series, and Probability

The Sum of a Finite Arithmetic Sequence


There is a simple formula for the sum of a finite arithmetic sequence.

The Sum of a Finite Arithmetic Sequence (See the proof on page 633.)
The sum of a finite arithmetic sequence with n terms is given by
n
Sn ⫽ 共a1 ⫹ an 兲.
2

Be sure you see that this formula works only for arithmetic sequences.

Example 5 Finding the Sum of a Finite Arithmetic Sequence


Find the sum: 1 ⫹ 3 ⫹ 5 ⫹ 7 ⫹ 9 ⫹ 11 ⫹ 13 ⫹ 15 ⫹ 17 ⫹ 19.

Solution
To begin, notice that the sequence is arithmetic (with a common difference of 2).
Moreover, the sequence has 10 terms. So, the sum of the sequence is
Sn ⫽ 1 ⫹ 3 ⫹ 5 ⫹ 7 ⫹ 9 ⫹ 11 ⫹ 13 ⫹ 15 ⫹ 17 ⫹ 19
n
⫽ 共a1 ⫹ an 兲 Formula for sum of an arithmetic sequence
2
10
⫽ 共1 ⫹ 19兲 Substitute 10 for n, 1 for a1, and 19 for an.
2
⫽ 5共20兲 ⫽ 100. Simplify.

Now try Exercise 57.

The sum of the first n terms of an infinite sequence is called the nth partial sum.
The nth partial sum of an arithmetic sequence can be found by using the formula for the
sum of a finite arithmetic sequence.

Example 6 Finding a Partial Sum of an Arithmetic Sequence


Find the 150th partial sum of the arithmetic sequence 5, 16, 27, 38, 49, . . . .

Solution
For this arithmetic sequence, you have a1 ⫽ 5 and d ⫽ 16 ⫺ 5 ⫽ 11. So,
an ⫽ 5 ⫹ 11共n ⫺ 1兲
and the nth term is an ⫽ 11n ⫺ 6. Therefore, a150 ⫽ 11共150兲 ⫺ 6 ⫽ 1644, and the
sum of the first 150 terms is
n
Sn ⫽ 共a1 ⫹ an 兲 nth partial sum formula
2
150
⫽ 共5 ⫹ 1644兲 Substitute 150 for n, 5 for a1, and 1644 for an.
2
⫽ 75共1649兲 Simplify.

⫽ 123,675. nth partial sum

Now try Exercise 65.

Copyright 2011 Cengage Learning. All Rights Reserved. May not be copied, scanned, or duplicated, in whole or in part. Due to electronic rights, some third party content may be suppressed from the eBook and/or eChapter(s).
Editorial review has deemed that any suppressed content does not materially affect the overall learning experience. Cengage Learning reserves the right to remove additional content at any time if subsequent rights restrictions require it.
Section 8.2 Arithmetic Sequences and Partial Sums 585

Applications

Example 7 Total Sales


A small business sells $20,000 worth of sports memorabilia during its first year. The
owner of the business has set a goal of increasing annual sales by $15,000 each year for
19 years. Assuming that this goal is met, find the total sales during the first 20 years this
business is in operation.

Algebraic Solution Numerical Solution


The annual sales form an arithmetic sequence in which The annual sales form an arithmetic sequence in which
a1 ⫽ 20,000 and d ⫽ 15,000. So, a1 ⫽ 20,000 and d ⫽ 15,000. So,
an ⫽ 20,000 ⫹ 15,000共n ⫺ 1兲 an ⫽ 20,000 ⫹ 15,000共n ⫺ 1兲
and the nth term of the sequence is and the nth term of the sequence is
an ⫽ 15,000n ⫹ 5000. an ⫽ 15,000n ⫹ 5000.
This implies that the 20th term of the sequence is Use the list editor of a graphing utility to create a table that
shows the sales for each of the first 20 years and the total
a20 ⫽ 15,000共20兲 ⫹ 5000
sales for the first 20 years, as shown in Figure 8.7.
⫽ 300,000 ⫹ 5000
Enter 15000L1 + 5000 for L 2.
⫽ 305,000.
Enter 1 Enter
The sum of the first 20 terms of the sequence is through cumSum(L 2)
20 for L1. for L 3.
n
Sn ⫽ 共a1 ⫹ an兲 nth partial sum formula
2
The total sales
20 Substitute 20 for n, for the first 20
⫽ 共20,000 ⫹ 305,000兲 20,000 for a1, and
2 years.
305,000 for an.
⫽ 10共325,000兲 Simplify. Figure 8.7

⫽ 3,250,000. Simplify. So, the total sales for the first 20 years are $3,250,000.
So, the total sales for the first 20 years are $3,250,000.
Now try Exercise 85.

Figure 8.8 shows the annual sales for the business in Example 7. Notice that the
annual sales for the business follow a linear growth pattern. In other words, saying that
a quantity increases arithmetically is the same as saying that it increases linearly.

y
Sales (in thousands of dollars)

350
300
250
200
150
100
50
x
1 2 3 4 5 6 7 8 9 10 11 12 13 14 15 16 17 18 19 20
Year
Figure 8.8

If you go on to take a course in calculus, you will study sequences and series in detail.
You will learn that sequences and series play a major role in the study of calculus.

Copyright 2011 Cengage Learning. All Rights Reserved. May not be copied, scanned, or duplicated, in whole or in part. Due to electronic rights, some third party content may be suppressed from the eBook and/or eChapter(s).
Editorial review has deemed that any suppressed content does not materially affect the overall learning experience. Cengage Learning reserves the right to remove additional content at any time if subsequent rights restrictions require it.
586 Chapter 8 Sequences, Series, and Probability

See www.CalcChat.com for worked-out solutions to odd-numbered exercises.


8.2 Exercises For instructions on how to use a graphing utility, see Appendix A.

Vocabulary and Concept Check


In Exercises 1 and 2, fill in the blank.
1. The nth term of an arithmetic sequence has the form _______ .
n
2. The formula Sn ⫽ 共a1 ⫹ an 兲 can be used to find the sum of the first n terms of
2
an arithmetic sequence, called the _______ .

3. How do you know when a sequence is arithmetic?


4. Is 4 or 1 the common difference of the arithmetic sequence an ⫽ 4n ⫹ 1?

Procedures and Problem Solving


Determining Whether a Sequence Is Arithmetic In Writing Terms of an Arithmetic Sequence In Exercises
Exercises 5–10, determine whether or not the sequence is 39– 42, write the first five terms of the arithmetic
arithmetic. If it is, find the common difference. sequence. Find the common difference and write the nth
term of the sequence as a function of n.
5. 10, 8, 6, 4, 2, . . . 6. 4, 9, 14, 19, 24, . . .
5 3
7. 3, 2, 2, 2, 1, . . .
1 2 4 8 16
8. 3, 3, 3, 3, 3 , . . . 39. a1 ⫽ 15, ak⫹1 ⫽ ak ⫹ 4
9. 3.7, 4.3, 4.9, 5.5, 6.1, . . . 10. 12, 22, 32, 42, 52, . . . 40. a1 ⫽ 6, ak⫹1 ⫽ ak ⫹ 5
3 1
41. a1 ⫽ 5, ak⫹1 ⫽ ⫺ 10 ⫹ ak
Writing the Terms of a Sequence In Exercises 11–20, 42. a1 ⫽ 1.5, ak⫹1 ⫽ ak ⫺ 2.5
write the first five terms of the sequence. Determine
whether or not the sequence is arithmetic. If it is, find the
Using a Recursion Formula In Exercises 43–46, the first
common difference. (Assume n begins with 1.)
two terms of the arithmetic sequence are given. Find the
11. an ⫽ 8 ⫹ 13n 12. an ⫽ 2n ⫹ n missing term. Use the table feature of a graphing utility
1 to verify your results.
13. an ⫽ 14. an ⫽ 1 ⫹ 共n ⫺ 1兲4
n⫹1 43. a1 ⫽ 5, a2 ⫽ 11, a10 ⫽ 䊏
15. an ⫽ 150 ⫺ 7n 16. an ⫽ 2n⫺1 44. a1 ⫽ 3, a2 ⫽ 13, a9 ⫽ 䊏
17. an ⫽ 3 ⫹ 2共⫺1兲n 18. an ⫽ 3 ⫺ 4共n ⫹ 6兲 45. a1 ⫽ 4.2, a2 ⫽ 6.6, a 7 ⫽ 䊏
19. an ⫽ 共⫺1兲n 20. an ⫽ 共⫺1兲2n⫹1 46. a1 ⫽ ⫺0.7, a2 ⫽ ⫺13.8, a8 ⫽ 䊏

Finding the nth Term of an Arithmetic Sequence In Graphing Terms of a Sequence In Exercises 47–50, use
Exercises 21–30, find a formula for an for the arithmetic a graphing utility to graph the first 10 terms of the
sequence. sequence. (Assume n begins with 1.)
21. a1 ⫽ 1, d ⫽ 3 22. a1 ⫽ 15, d ⫽ 4 3
47. an ⫽ 15 ⫺ 2n 48. an ⫽ ⫺5 ⫹ 2n
23. a1 ⫽ 100, d ⫽ ⫺8 24. a1 ⫽ 0, d ⫽ ⫺ 23 49. an ⫽ 0.4n ⫺ 2 50. an ⫽ ⫺1.3n ⫹ 7
25. 4, 32, ⫺1, ⫺ 27, . . . 26. 10, 5, 0, ⫺5, ⫺10, . . .
27. a1 ⫽ 5, a4 ⫽ 15 28. a1 ⫽ ⫺4, a5 ⫽ 16 Finding Terms of a Sequence In Exercises 51–56, use
the table feature of a graphing utility to find the first 10
29. a3 ⫽ 94, a6 ⫽ 85 30. a5 ⫽ 190, a10 ⫽ 115
terms of the sequence. (Assume n begins with 1.)
Writing the Terms of an Arithmetic Sequence In 51. an ⫽ 4n ⫺ 5 52. an ⫽ 17 ⫹ 3n
3 4
Exercises 31–38, write the first five terms of the 53. an ⫽ 20 ⫺ 4 n 54. an ⫽ 5n ⫹ 12
arithmetic sequence. Use the table feature of a graphing 55. an ⫽ 1.5 ⫹ 0.05n 56. an ⫽ 8 ⫺ 12.5n
utility to verify your results.
31. a1 ⫽ 5, d ⫽ 6 32. a1 ⫽ 5, d ⫽ ⫺ 34 Finding the Sum of a Finite Arithmetic Sequence In
33. a1 ⫽ ⫺10, d ⫽ ⫺12 34. a4 ⫽ 16, a10 ⫽ 46 Exercises 57–64, find the sum of the finite arithmetic
sequence.
35. a8 ⫽ 26, a12 ⫽ 42 36. a6 ⫽ ⫺38, a11 ⫽ ⫺73
37. a3 ⫽ 19, a15 ⫽ ⫺1.7 38. a5 ⫽ 16, a14 ⫽ 38.5 57. 2 ⫹ 4 ⫹ 6 ⫹ 8 ⫹ 10 ⫹ 12 ⫹ 14 ⫹ 16 ⫹ 18 ⫹ 20
58. 1 ⫹ 4 ⫹ 7 ⫹ 10 ⫹ 13 ⫹ 16 ⫹ 19

Copyright 2011 Cengage Learning. All Rights Reserved. May not be copied, scanned, or duplicated, in whole or in part. Due to electronic rights, some third party content may be suppressed from the eBook and/or eChapter(s).
Editorial review has deemed that any suppressed content does not materially affect the overall learning experience. Cengage Learning reserves the right to remove additional content at any time if subsequent rights restrictions require it.
Section 8.2 Arithmetic Sequences and Partial Sums 587

59. ⫺1 ⫹ 共⫺3兲 ⫹ 共⫺5兲 ⫹ 共⫺7兲 ⫹ 共⫺9兲 84. Performing Arts An auditorium has 20 rows of seats.
60. ⫺5 ⫹ 共⫺3兲 ⫹ 共⫺1兲 ⫹ 1 ⫹ 3 ⫹ 5 There are 20 seats in the first row, 21 seats in the second
row, 22 seats in the third row, and so on. How many
61. Sum of the first 100 positive integers
seats are there in all 20 rows?
62. Sum of the first 50 negative integers
63. Sum of the integers from ⫺100 to 30
64. Sum of the integers from ⫺10 to 50

Finding a Partial Sum of an Arithmetic Sequence In


Exercises 65–70, find the indicated nth partial sum of the 20
arithmetic sequence.
85. Sales A hardware store makes a profit of $30,000
65. 8, 20, 32, 44, . . . , n ⫽ 10
during its first year. The store owner sets a goal of
66. 2, 8, 14, 20, . . . , n ⫽ 25 increasing profits by $5000 each year for 4 years.
67. 0.5, 1.3, 2.1, 2.9, . . . , n ⫽ 10 Assuming that this goal is met, find the total profit
68. 4.2, 3.7, 3.2, 2.7, . . . , n ⫽ 12 during the first 5 years of business.
69. a1 ⫽ 100, a25 ⫽ 220, n ⫽ 25 86. Physics An object with negligible air resistance is
dropped from a plane. During the first second of fall,
70. a1 ⫽ 15, a100 ⫽ 307, n ⫽ 100
the object falls 16 feet; during the second second, it falls
48 feet; during the third second, it falls 80 feet; and
Finding a Partial Sum of an Arithmetic Sequence In
during the fourth second, it falls 112 feet. Assume this
Exercises 71–76, find the partial sum without using a
pattern continues. How many feet will the object fall in
graphing utility.
8 seconds?
50 100
71. 兺n
n⫽1
72. 兺 2n
n⫽1
87. MODELING DATA
30 10 100 50 The table shows the sales an (in billions of dollars) for
73. 兺
n⫽11
n⫺ 兺
n⫽1
n 74. 兺
n⫽51
n⫺ 兺
n⫽1
n Coca-Cola Enterprises from 2001 through 2008.
(Source: Coca-Cola Enterprises, Inc.)
500 100 8 ⫺ 3n
75. 兺 共n ⫹ 8兲
n⫽1
76. 兺
n⫽1 16
Sales, an
Year
Finding a Partial Sum Using a Graphing Utility In (in billions of dollars)
Exercises 77–82, use a graphing utility to find the partial 2001 15.7
sum. 2002 16.9
20 50 2003 17.3
77. 兺
n⫽1
共2n ⫹ 1兲 78. 兺 共50 ⫺ 2n兲
n⫽0
2004 18.2
100 n⫹1 100 4⫺n 2005 18.7
79. 兺
n⫽1 2
80. 兺
n⫽0 4 2006 19.8
60 200 2007 20.9
81. 兺 共250 ⫺ 5i兲
i⫽1
2
82. 兺 共10.5 ⫹ 0.025j兲
j⫽1
2008 21.8

83. (p. 581) A brick patio has (a) Use the regression feature of a graphing utility to
the approximate shape of a trapezoid, as find an arithmetic sequence for the data. Let n
shown in the figure. The patio has 18 rows represent the year, with n ⫽ 1 corresponding to 2001.
of bricks. The first row has 14 bricks and (b) Use the sequence from part (a) to approximate the
the 18th row has 31 bricks. How many annual sales for Coca-Cola Enterprises for the
bricks are in the patio? years 2001 through 2008. How well does the
model fit the data?
31 (c) Use the sequence to find the total sales for
Coca-Cola Enterprises over the period from 2001
through 2008.
(d) Use the sequence to predict the total sales over
the period from 2009 through 2016. Is your total
reasonable? Explain.
14
Gordon Swanson 2010/used under license from Shutterstock.com

Copyright 2011 Cengage Learning. All Rights Reserved. May not be copied, scanned, or duplicated, in whole or in part. Due to electronic rights, some third party content may be suppressed from the eBook and/or eChapter(s).
Editorial review has deemed that any suppressed content does not materially affect the overall learning experience. Cengage Learning reserves the right to remove additional content at any time if subsequent rights restrictions require it.
588 Chapter 8 Sequences, Series, and Probability

88. MODELING DATA 95. Think About It The sum of the first n terms of an
arithmetic sequence with first term a1 and common
The table shows the numbers an (in thousands) of
difference d is Sn. Determine the sum when each term
master’s degrees conferred in the United States from
is increased by 5. Explain.
2000 through 2007. (Source: U.S. National Center
for Education Statistics) 96. Think About It In each sequence, decide whether it
is possible to fill in the blanks to form an arithmetic
sequence. If so, find a recursion formula for the
Master’s degrees conferred, sequence. Explain how you found your answers.
Year
an (in thousands)
(a) ⫺7, 䊏, 䊏, 䊏, 䊏, 䊏, 11
(b) 17, 䊏, 䊏, 䊏, 䊏, 䊏, 䊏, 59
2000 457
2001 468
(c) 2, 6, 䊏, 䊏, 162
2002 482
2003 513 (d) 4, 7.5, 䊏, 䊏, 䊏, 䊏, 䊏, 28.5
2004 559 (e) 8, 12, 䊏, 䊏, 䊏, 60.75
2005 575 97. Writing Carl Friedrich Gauss, a famous nineteenth
2006 594 century mathematician, was a child prodigy. It was
2007 605 said that when Gauss was 10 he was asked by his
teacher to add the numbers from 1 to 100. Almost
(a) Use the regression feature of a graphing utility to immediately, Gauss found the answer by mentally
find an arithmetic sequence for the data. Let n finding the summation. Write an explanation of how
represent the year, with n ⫽ 0 corresponding to 2000. he arrived at his conclusion, and then find the formula
for the sum of the first n natural numbers.
(b) Use the sequence from part (a) to approximate the
numbers of master’s degrees conferred for the
98. C A P S T O N E Describe the characteristics of an
years 2000 through 2007. How well does the
arithmetic sequence. Give an example of a sequence
model fit the data?
that is arithmetic and one that is not arithmetic.
(c) Use the sequence to find the total number of
master’s degrees conferred over the period from
2000 through 2007. Finding the Sum of a Finite Arithmetic Sequence In
(d) Use the sequence to predict the total number of Exercises 99–102, find the sum using the method from
master’s degrees conferred over the period from Exercise 97.
2008 through 2018. Is your total reasonable? 99. The first 200 natural numbers
Explain. 100. The first 100 even natural numbers from 2 to 200,
inclusive
Conclusions 101. The first 51 odd natural numbers from 1 to 101, inclusive
102. The first 100 multiples of 4 from 4 to 400, inclusive
True or False? In Exercises 89 and 90, determine
whether the statement is true or false. Justify your answer.
Cumulative Mixed Review
89. Given the nth term and the common difference of an
Gauss-Jordan Elimination In Exercises 103 and 104, use
arithmetic sequence, it is possible to find the 共n ⫹ 1兲th
Gauss-Jordan elimination to solve the system of equations.
term.
2x ⫺ y ⫹ 7z ⫽ ⫺10


103.
90. If the only known information about a finite arithmetic
sequence is its first term and its last term, then it is 3x ⫹ 2y ⫺ 4z ⫽ 17
possible to find the sum of the sequence. 6x ⫺ 5y ⫹ z ⫽ ⫺20
⫺x ⫹ 4y ⫹ 10z ⫽ 4


104.
Finding Terms of a Sequence In Exercises 91 and 92,
5x ⫺ 3y ⫹ z ⫽ 31
find the first 10 terms of the sequence.
8x ⫹ 2y ⫺ 3z ⫽ ⫺5
91. a1 ⫽ x, d ⫽ 2x 92. a1 ⫽ ⫺y, d ⫽ 5y

93. Think About It The sum of the first 20 terms of an 105. Make a Decision To work an extended application
arithmetic sequence with a common difference of 3 is analyzing the amount of municipal waste recovered in
650. Find the first term. the United States from 1983 through 2008, visit this
textbook’s Companion Website. (Data Source: U.S.
94. Writing Explain how to use the first two terms of an Environmental Protection Agency)
arithmetic sequence to find the nth term.

Copyright 2011 Cengage Learning. All Rights Reserved. May not be copied, scanned, or duplicated, in whole or in part. Due to electronic rights, some third party content may be suppressed from the eBook and/or eChapter(s).
Editorial review has deemed that any suppressed content does not materially affect the overall learning experience. Cengage Learning reserves the right to remove additional content at any time if subsequent rights restrictions require it.
Section 8.3 Geometric Sequences and Series 589

8.3 Geometric Sequences and Series

Geometric Sequences What you should learn


● Recognize, write, and find
In Section 8.2, you learned that a sequence whose consecutive terms have a common
the nth terms of geometric
difference is an arithmetic sequence. In this section, you will study another important
sequences.
type of sequence called a geometric sequence. Consecutive terms of a geometric
● Find nth partial sums of
sequence have a common ratio.
geometric sequences.
● Find sums of infinite geometric
Definition of Geometric Sequence series.
● Use geometric sequences to
A sequence is geometric when the ratios of consecutive terms are the same. So,
model and solve real-life
the sequence a1, a2, a3, a4, . . . , an, . . . is geometric when there is a number r
problems.
such that
a 2 a 3 a4 . . .
Why you should learn it
a1 ⫽ a 2 ⫽ a3 ⫽ ⫽ r, r ⫽ 0. Geometric sequences can reduce the
amount of time it takes to find the
The number r is the common ratio of the sequence. sum of a sequence of numbers with a
common ratio. For instance, Exercise
109 on page 598 shows how to use
Example 1 Examples of Geometric Sequences a geometric sequence to find the
total vertical distance traveled by
a. The sequence whose nth term is 2n is geometric. For this sequence, the a bouncing ball.
common ratio between consecutive terms is 2.
2, 4, 8, 16, . . . , 2n, . . . Begin with n ⫽ 1.

4
2 ⫽2

b. The sequence whose nth term is 4共3n 兲 is geometric. For this sequence, the
common ratio between consecutive terms is 3.
12, 36, 108, 324, . . . , 4共3n 兲, . . . Begin with n ⫽ 1.

36
12 ⫽3

c. The sequence whose nth term is 共⫺ 3 兲 is geometric. For this sequence, the
1 n

1
common ratio between consecutive terms is ⫺ 3.

冢 冣
n
1 1 1 1 1
⫺ , ,⫺ , ,. . ., ⫺ ,. . . Begin with n ⫽ 1.
3 9 27 81 3
1兾9
⫺1兾3 ⫽ ⫺ 13

Now try Exercise 7.

The sequence
1, 4, 9, 16, . . .
whose nth term is n2, is not geometric. The ratio of the second term to the first term is
Study Tip
a2 4 In Example 1, notice
⫽ ⫽4 that each of the
a1 1
geometric sequences
but the ratio of the third term to the second term is has an nth term of the form ar n,
a3 9 where r is the common ratio of
⫽ . the sequence.
a2 4
Feng Yu 2010/used under license from Shutterstock.com

Copyright 2011 Cengage Learning. All Rights Reserved. May not be copied, scanned, or duplicated, in whole or in part. Due to electronic rights, some third party content may be suppressed from the eBook and/or eChapter(s).
Editorial review has deemed that any suppressed content does not materially affect the overall learning experience. Cengage Learning reserves the right to remove additional content at any time if subsequent rights restrictions require it.
590 Chapter 8 Sequences, Series, and Probability

The nth Term of a Geometric Sequence


The nth term of a geometric sequence has the form
an ⫽ a1r n⫺1
where r is the common ratio of consecutive terms of the sequence. So, every
geometric sequence can be written in the following form.
a1, a2, a3, a 4, a5, . . ., an, . . .

a1, a1r, a1r 2, a1r 3, a1r 4, . . . , a1r n⫺1, . . .

When you know the nth term of a geometric sequence, you can find the 共n ⫹ 1兲th
term by multiplying by r. That is,
an⫹1 ⫽ anr.

Example 2 Finding the Terms of a Geometric Sequence


Write the first five terms of the geometric sequence whose first term is
a1 ⫽ 3
Technology Tip
and whose common ratio is
You can use a graphing
r ⫽ 2.
utility to generate the
Solution geometric sequence
in Example 2 by using the
Starting with 3, repeatedly multiply by 2 to obtain the following.
following steps.
a1 ⫽ 3 1st term
3 ENTER
a2 ⫽ 3共21兲 ⫽ 6 2nd term 2 ⫻ ANS
a3 ⫽ 3共22兲 ⫽ 12 3rd term Now press the enter key
repeatedly to generate the terms
a4 ⫽ 3共23兲 ⫽ 24 4th term of the sequence.
a5 ⫽ 3共24兲 ⫽ 48 5th term

Now try Exercise 17.

Example 3 Finding a Term of a Geometric Sequence


Find the 15th term of the geometric sequence whose first term is 20 and whose common
ratio is 1.05.

Algebraic Solution Numerical Solution


an ⫽ a1 r n⫺1 Formula for a geometric sequence For this sequence, r ⫽ 1.05 and a1 ⫽ 20. So, an ⫽ 20共1.05兲n⫺1.
Substitute 20 for a1, Use the table feature of a graphing utility to create a table that
a15 ⫽ 20共1.05兲15⫺1 1.05 for r, and 15 for n. shows the terms of the sequence, as shown in Figure 8.9.
⬇ 39.60 Use a calculator.
The number in
the 15th row is
the 15th term of
the sequence.

So, a15 ≈ 39.60.


Now try Exercise 31. Figure 8.9

Copyright 2011 Cengage Learning. All Rights Reserved. May not be copied, scanned, or duplicated, in whole or in part. Due to electronic rights, some third party content may be suppressed from the eBook and/or eChapter(s).
Editorial review has deemed that any suppressed content does not materially affect the overall learning experience. Cengage Learning reserves the right to remove additional content at any time if subsequent rights restrictions require it.
Section 8.3 Geometric Sequences and Series 591

Example 4 Finding a Term of a Geometric Sequence


Find a formula for the nth term of the following geometric sequence. What is the ninth Technology Tip
term of the sequence?
Most graphing utilities
5, 15, 45, . . . have a built-in function
that will display the
Solution
terms of a geometric sequence.
The common ratio of this sequence is For instructions on how to
15 use the sequence feature,
r⫽ ⫽ 3. see Appendix A; for specific
5
keystrokes, go to this text-
Because the first term is a1 ⫽ 5, the formula must have the form book’s Companion Website.
an ⫽ a1r n⫺1 ⫽ 5共3兲n⫺1.
You can determine the ninth term 共n ⫽ 9兲 to be
a9 ⫽ 5共3兲9⫺1 Substitute 9 for n.
40,000
⫽ 5共6561兲 Use a calculator.

⫽ 32,805. Simplify.

A graph of the first nine terms of the sequence is shown in


Figure 8.10. Notice that the points lie on an exponential 0 11
curve. This makes sense because an is an exponential 0
function of n. Figure 8.10
Now try Exercise 39.

When you know any two terms of a geometric sequence, you can use that
information to find a formula for the nth term of the sequence.

Example 5 Finding a Term of a Geometric Sequence


The fourth term of a geometric sequence is 125, and the 10th term is
125
.
64
Find the 14th term. (Assume that the terms of the sequence are positive.)

Solution
The 10th term is related to the fourth term by the equation
a10 ⫽ a4r 6. Multiply 4th term by r 10⫺4.
Study Tip
Because a10 ⫽ 125兾64 and a4 ⫽ 125, you can solve for r as follows.
Remember that r is
125
⫽ 125r 6 the common ratio of
64 consecutive terms of a
1 1 geometric sequence. So, in
⫽ r6 ⫽r Example 5,
64 2
You can obtain the 14th term by multiplying the 10th term by r 4. a10 ⫽ a1r 9
⫽ a1 ⭈ r ⭈ r ⭈ r ⭈ r6
冢冣
4
125 1 125
a14 ⫽ a10 r 4 ⫽ ⫽
64 2 1024 a a a
⫽ a1 ⭈ a12 ⭈ a32 ⭈ a43 ⭈ r 6
Now try Exercise 45.

Copyright 2011 Cengage Learning. All Rights Reserved. May not be copied, scanned, or duplicated, in whole or in part. Due to electronic rights, some third party content may be suppressed from the eBook and/or eChapter(s).
Editorial review has deemed that any suppressed content does not materially affect the overall learning experience. Cengage Learning reserves the right to remove additional content at any time if subsequent rights restrictions require it.
592 Chapter 8 Sequences, Series, and Probability

The Sum of a Finite Geometric Sequence


The formula for the sum of a finite geometric sequence is as follows.

The Sum of a Finite Geometric Sequence (See the proof on page 633.)
The sum of the finite geometric sequence
a1, a1r, a1r 2, a1r 3, a1r 4, . . . , a1r n⫺1
with common ratio r ⫽ 1 is given by
1 ⫺ rn
冢 1 ⫺ r 冣.
n
Sn ⫽ 兺 a1r i⫺1 ⫽ a1
i⫽1
Technology Tip
Using the sum
Example 6 Finding the Sum of a Finite Geometric Sequence sequence feature of a
graphing utility, you
can calculate the sum of the
Find the following sum. sequence in Example 6 to be
12 about 1.7142848, as shown
兺 4共0.3兲
n⫽1
n
below.

Solution
By writing out a few terms, you have
12

兺 4共0.3兲
n⫽1
n ⫽ 4共0.3兲1 ⫹ 4共0.3兲2 ⫹ 4共0.3兲3 ⫹ . . . ⫹ 4共0.3兲12.

Now, because Calculate the sum beginning at


n ⫽ 0. You should obtain a sum
a1 ⫽ 4共0.3兲, r ⫽ 0.3, and n ⫽ 12
of about 5.7142848.
you can apply the formula for the sum of a finite geometric sequence to obtain
1 ⫺ rn
冢1⫺r冣
12


Formula for sum of a finite
4共0.3兲n ⫽ a1 geometric sequence
n⫽1

1 ⫺ 共0.3兲12
⫽ 4共0.3兲 冤 1 ⫺ 0.3 冥 Substitute 4共0.3兲 for a1, 0.3 for r,
and 12 for n.

⬇ 1.71. Use a calculator.


What’s Wrong?
You use a graphing utility to
Now try Exercise 55.
find the sum
6
When using the formula for the sum of a geometric sequence, be careful to check
that the index begins at i ⫽ 1. For an index that begins at i ⫽ 0, you must adjust the
兺 7共0.5兲
n⫽1
n

formula for the nth partial sum. For instance, if the index in Example 6 had begun with as shown in the figure. What’s
n ⫽ 0, then the sum would have been wrong?
12 12

兺 4共0.3兲
n⫽0
n ⫽ 4共0.3兲0 ⫹ 兺 4共0.3兲
n⫽1
n

12
⫽4⫹ 兺 4共0.3兲
n⫽1
n

⬇ 4 ⫹ 1.71
⫽ 5.71.

Copyright 2011 Cengage Learning. All Rights Reserved. May not be copied, scanned, or duplicated, in whole or in part. Due to electronic rights, some third party content may be suppressed from the eBook and/or eChapter(s).
Editorial review has deemed that any suppressed content does not materially affect the overall learning experience. Cengage Learning reserves the right to remove additional content at any time if subsequent rights restrictions require it.
Section 8.3 Geometric Sequences and Series 593

Geometric Series
The sum of the terms of an infinite geometric sequence is called an infinite geometric Explore the Concept
series or simply a geometric series.
The formula for the sum of a finite geometric sequence can, depending on the value Notice that the formula
of r, be extended to produce a formula for the sum of an infinite geometric series. for the sum of an
Specifically, if the common ratio r has the property that infinite geometric
series requires that r < 1. ⱍⱍ
ⱍrⱍ < 1 What happens when r ⫽ 1 or
then it can be shown that r n becomes arbitrarily close to zero as n increases r ⫽ ⫺1? Give examples of
without bound. Consequently, infinite geometric series for

1 ⫺ rn 1⫺0
ⱍⱍ
which r > 1 and convince
a1 冢
1⫺r 冣 a1 冢
1⫺r 冣 as n ⬁.
yourself that they do not have
finite sums.
This result is summarized as follows.

The Sum of an Infinite Geometric Series

ⱍⱍ
If r < 1, then the infinite geometric series
a1 ⫹ a1r ⫹ a1r 2 ⫹ a1r 3 ⫹ . . . ⫹ a1r n⫺1 ⫹ . . .
has the sum
⬁ a1
S⫽ 兺a r
i⫽0
1
i ⫽
1⫺r
.
Explore the Concept
On most graphing
utilities, it is not
Note that when
possible to find the
ⱍrⱍ ⱖ 1 sum of the series in Example
7(a) because you cannot enter
the series does not have a sum.
⬁ as the upper limit of
summation. Can you still find
Example 7 Finding the Sum of an Infinite Geometric Series the sum using a graphing utility
without entering ⬁? If so,
Find each sum.
which partial sum
⬁ will result in 10,
a. 兺 4共0.6兲
n⫽0
n
the exact sum
of the series?
b. 3 ⫹ 0.3 ⫹ 0.03 ⫹ 0.003 ⫹ . . .

Solution

a. 兺 4共0.6兲
n⫽0
n ⫽ 4 ⫹ 4共0.6兲 ⫹ 4共0.6兲2 ⫹ 4共0.6兲3 ⫹ . . . ⫹ 4共0.6兲n ⫹ . . .
4 a1

1 ⫺ 0.6 1⫺r

⫽ 10
b. 3 ⫹ 0.3 ⫹ 0.03 ⫹ 0.003 ⫹ . . . ⫽ 3 ⫹ 3共0.1兲 ⫹ 3共0.1兲2 ⫹ 3共0.2兲3 ⫹ . . .
3 a1

1 ⫺ 0.1 1⫺r

10

3
⬇ 3.33
Now try Exercise 75.
Yuri Arcurs 2010/used under license from Shutterstock.com
KennethMan 2010/used under license from Shutterstock.com

Copyright 2011 Cengage Learning. All Rights Reserved. May not be copied, scanned, or duplicated, in whole or in part. Due to electronic rights, some third party content may be suppressed from the eBook and/or eChapter(s).
Editorial review has deemed that any suppressed content does not materially affect the overall learning experience. Cengage Learning reserves the right to remove additional content at any time if subsequent rights restrictions require it.
594 Chapter 8 Sequences, Series, and Probability

Application

Example 8 Increasing Annuity


A deposit of $50 is made on the first day of each month in an account that pays 6%
compounded monthly. What is the balance at the end of 2 years? (This type of savings
plan is called an increasing annuity.)

Solution
Recall from Section 3.1 that the compound interest formula is

冢 冣.
r nt
A⫽P 1⫹ Formula for compound interest
n
To find the balance in the account after 24 months, consider each of the 24 deposits
separately. The first deposit will gain interest for 24 months, and its balance will be
24

冢 冣
0.06
A 24 ⫽ 50 1 ⫹
12

⫽ 50共1.005兲24.

The second deposit will gain interest for 23 months, and its balance will be
23

冢 冣
0.06
A 23 ⫽ 50 1 ⫹
12

⫽ 50共1.005兲23.

The last deposit will gain interest for only 1 month, and its balance will be Personal Financial Advisor

冢 冣
0.06 1
A1 ⫽ 50 1 ⫹
12

⫽ 50共1.005兲.

The total balance in the annuity will be the sum of the balances of the 24 deposits.
Using the formula for the sum of a finite geometric sequence, with a1 ⫽ 50共1.005兲,
r ⫽ 1.005, and n ⫽ 24, you have

冢11 ⫺⫺ rr 冣
n
Formula for sum of a finite
Sn ⫽ a1 geometric sequence

1 ⫺ 共1.005兲24
S24 ⫽ 50共1.005兲 冤 1 ⫺ 1.005 冥 Substitute 50共1.005兲 for a1,
1.005 for r, and 24 for n.

⬇ $1277.96. Simplify.

You can use the sum sequence feature of a graphing utility to verify this result, as shown
in Figure 8.11.

Figure 8.11

Now try Exercise 101.


Edyta Pawlowska 2010/used under license from Shutterstock.com

Copyright 2011 Cengage Learning. All Rights Reserved. May not be copied, scanned, or duplicated, in whole or in part. Due to electronic rights, some third party content may be suppressed from the eBook and/or eChapter(s).
Editorial review has deemed that any suppressed content does not materially affect the overall learning experience. Cengage Learning reserves the right to remove additional content at any time if subsequent rights restrictions require it.
Section 8.3 Geometric Sequences and Series 595

See www.CalcChat.com for worked-out solutions to odd-numbered exercises.


8.3 Exercises For instructions on how to use a graphing utility, see Appendix A.

Vocabulary and Concept Check


In Exercises 1–3, fill in the blank(s).
1. A sequence is called a _______ sequence if the ratios of consecutive terms are
the same. This ratio is called the _______ ratio.
2. The nth term of a geometric sequence has the form an ⫽ _______ .
3. The sum of the terms of an infinite geometric sequence is called a _______ .

4. Can a geometric sequence have a common ratio of 0?


5. For what values of the common ratio r is it possible to find the sum of an infinite
geometric series?
6. Which formula represents the sum of a finite geometric sequence? an infinite
geometric series?

冢11 ⫺⫺ rr 冣
n
a1
(a) S ⫽
1⫺r
, r < 1 ⱍⱍ (b) Sn ⫽ a1

Procedures and Problem Solving


Identifying a Geometric Sequence In Exercises 7–16, 4
33. a1 ⫽ 8, r ⫽ ⫺ 3, 7th term
determine whether or not the sequence is geometric. If it
34. a1 ⫽ 8, r ⫽ ⫺ 34, 9th term
is, find the common ratio.
35. a1 ⫽ ⫺1, r ⫽ 3, 6th term
7. 5, 15, 45, 135, . . . 8. 3, 12, 48, 192, . . . 1
36. a1 ⫽ ⫺ 128 , r ⫽ 2, 12th term
9. 6, 18, 30, 42, . . . 10. 1, ⫺2, 4, ⫺8, . . .
37. a1 ⫽ 500, r ⫽ ⫺1.02, 14th term
11. 1, ⫺ 12, 14, ⫺ 18, . . . 12. 5, 1, 0.2, 0.04, . . .
1 1 1
38. a1 ⫽ 1000, r ⫽ 1.005, 11th term
13. 8 , 4 , 2 , 1, . . . 14. 9, ⫺6, 4, ⫺ 83, . . .
1, 12, 13, 14, . . . 1 2 3 4
15. 16. 5 , 7 , 9 , 11 , . . . Finding a Term of a Geometric Sequence In Exercises
39–42, find a formula for the nth term of the geometric
Finding the Terms of a Geometric Sequence In Exercises sequence. Then find the indicated term of the geometric
17–24, write the first five terms of the geometric sequence. sequence.
17. a1 ⫽ 6, r ⫽ 3 18. a1 ⫽ 4, r⫽2 39. 9th term: 7, 21, 63, . . .
19. a1 ⫽ 1, r ⫽ 12 20. a1 ⫽ 2, r ⫽ 13 40. 7th term: 3, 36, 432, . . .
1 1
21. a1 ⫽ 5, r ⫽ ⫺ 10 22. a1 ⫽ 6, r ⫽ ⫺4 41. 10th term: 5, 30, 180, . . .
23. ai ⫽ 1, r ⫽ e 24. a1 ⫽ 4, r ⫽ 冪3 42. 22nd term: 4, 8, 16, . . .

Finding the nth Term of a Geometric Sequence In Finding a Term of a Geometric Sequence In Exercises
Exercises 25–30, write the first five terms of the 43–46, find the indicated term of the geometric sequence.
geometric sequence. Find the common ratio and write
43. a1 ⫽ 4, a4 ⫽ 12, 10th term
the nth term of the sequence as a function of n.
44. a1 ⫽ 5, a3 ⫽ 454 , 8th term
25. a1 ⫽ 64, ak⫹1 ⫽ 1
2 ak 26. a1 ⫽ 81, ak⫹1 ⫽ 1
3 ak 45. a2 ⫽ ⫺18, a5 ⫽ 23, 6th term
27. a1 ⫽ 9, ak⫹1 ⫽ 2ak 28. a1 ⫽ 5, ak⫹1 ⫽ ⫺3ak
a3 ⫽ 16 64
46. 3 , a5 ⫽ 27 , 7th term
29. a1 ⫽ 6, ak⫹1 ⫽ ⫺ 32ak 30. a1 ⫽ 30, ak⫹1 ⫽ ⫺ 23ak
Using a Graphing Utility In Exercises 47–50, use a
Finding a Term of a Geometric Sequence In Exercises graphing utility to graph the first 10 terms of the sequence.
31–38, find the indicated term of the geometric sequence
(a) using the table feature of a graphing utility and (b) 47. an ⫽ 12共⫺0.75兲n⫺1
algebraically. 48. an ⫽ 20共1.25兲n⫺1
1
31. a1 ⫽ 6, r ⫽ 3, 12th term 49. an ⫽ 2共1.3兲n⫺1
3
32. a1 ⫽ 24, r ⫽ 2, 8th term 50. an ⫽ 10共⫺1.2兲n⫺1

Copyright 2011 Cengage Learning. All Rights Reserved. May not be copied, scanned, or duplicated, in whole or in part. Due to electronic rights, some third party content may be suppressed from the eBook and/or eChapter(s).
Editorial review has deemed that any suppressed content does not materially affect the overall learning experience. Cengage Learning reserves the right to remove additional content at any time if subsequent rights restrictions require it.
596 Chapter 8 Sequences, Series, and Probability

Finding a Sequence of Partial Sums In Exercises 51 and Writing a Repeating Decimal as a Rational Number In
52, find the sequence of the first five partial sums S1, S2, Exercises 83–86, find the rational number representation
S3, S4, and S5 of the geometric sequence by adding terms. of the repeating decimal.
1 81
51. 8, ⫺4, 2, ⫺1, 2, . . . 52. 8, 12, 18, 27, 2 , . . . 83. 0.36 84. 0.297
85. 1.25 86. 1.38
Finding a Sequence of Partial Sums In Exercises 53 and
54, use a graphing utility to create a table showing the Identifying a Sequence In Exercises 87–94, determine
sequence of the first 10 partial sums S1, S2, S3, . . . , and whether the sequence associated with the series is
S10 for the series. arithmetic or geometric. Find the common difference or
⬁ ⬁ ratio and find the sum of the first 15 terms.
兺 16共 兲 兺 4共0.2兲
1 n⫺1 n⫺1
53. 2 54.
n⫽1 n⫽1 87. 8 ⫹ 16 ⫹ 32 ⫹ 64 ⫹ . . .
88. 17 ⫹ 14 ⫹ 11 ⫹ 8 ⫹ . . .
Finding the Sum of a Finite Geometric Sequence In 10
Exercises 55–64, find the sum. Use a graphing utility to 89. 90 ⫹ 30 ⫹ 10 ⫹ 3 ⫹ . . .
5 7 9 11
verify your result. 90. 4 ⫹ 4 ⫹ 4 ⫹ 4 ⫹ . . .
⬁ ⬁
兺 兺2
9 9

兺2 兺 共⫺2兲
n⫺1
n⫺1 n⫺1 91. 6n 92.
55. 56. n⫽1 n⫽1
n⫽1 n⫽1
⬁ ⬁n⫹4
兺 兺
7 6
64共 兲 32共 兲 6共0.8兲n
兺 兺
1 1 i⫺1 93. 94.
57. ⫺ 2 i⫺1 58. 4 4
n⫽0 n⫽0
i⫽1 i⫽1
20 15

兺 3共 兲 兺 2共 兲 95. Compound Interest A principal of $1000 is invested


3 n 4 n
59. 2 60. 3
n⫽0 n⫽0 at 3% interest. Find the amount after
10 10 10 years if the interest is compounded (a) annually,
兺 8共⫺ 兲 兺 5共⫺ 兲
1 i⫺1 1 i⫺1
61. 4 62. 3 (b) semiannually, (c) quarterly, (d) monthly, and (e) daily.
i⫽1 i⫽1
5 6 96. Annuity A deposit of $100 is made at the beginning
63. 兺 300共1.06兲
n⫽0
n
64. 兺 500共1.04兲
n⫽0
n
of each month in an account that pays 3% interest,
compounded monthly. The balance A in the account at
the end of 5 years is given by
Using Summation Notation In Exercises 65–68, use

冢 冣 冢 冣
1 60
summation notation to write the sum. 0.03 0.03
A ⫽ 100 1 ⫹ ⫹ . . . ⫹ 100 1 ⫹ .
65. 5 ⫹ 15 ⫹ 45 ⫹ . . . ⫹ 3645 12 12
66. 7 ⫹ 14 ⫹ 28 ⫹ . . . ⫹ 896 Find A.
2 ⫺ 12 ⫹ 18 ⫺ . . . ⫹ 2048
1
67. 97. Annuity A deposit of P dollars is made at the
68. 15 ⫺ 3 ⫹ 5 ⫺ . . . ⫺ 625
3 3 beginning of each month in an account earning an annu-
al interest rate r, compounded monthly. The balance A
Finding the Sum of an Infinite Geometric Series In after t years is given by

冢 冣 冢 冣
Exercises 69–82, find the sum of the infinite geometric r r 2

series, if possible. If not possible, explain why. A⫽P 1⫹ ⫹P 1⫹ ⫹. . .


12 12
⬁ ⬁
兺 10共5 兲 兺 6共 兲 冢 冣
4 n 2 n r 12t
69. 70. 3 ⫹P 1⫹ .
n⫽0 n⫽0 12
⬁ ⬁
兺 5共⫺ 兲 兺 9共⫺ 兲
1 n 2 n
71. 2 72. 3 Show that the balance is given by
n⫽0 n⫽0

冤冢 冣 冥冢 冣
r 12t 12
⬁ ⬁
2共 兲 8共 兲 A⫽P 1⫹ ⫺1 1⫹
兺 兺
7 n⫺1 5 n⫺1 .
73. 3 74. 3 12 r
n⫽1 n⫽1
⬁ ⬁ 98. Annuity A deposit of P dollars is made at the
75. 兺 10共0.11兲
n⫽0
n
76. 兺 5共0.45兲
n⫽0
n
beginning of each month in an account earning an
⬁ ⬁ annual interest rate r, compounded continuously.
77. 兺 ⫺3共⫺0.9兲 n
78. 兺 ⫺10共⫺0.2兲 n
The balance A after t years is given by
n⫽0 n⫽0 A ⫽ Per兾12 ⫹ Pe 2r兾12 ⫹ . . . ⫹ Pe12tr 兾12. Show that
79. 9 ⫹ 6 ⫹ 4 ⫹ 3 ⫹ . . . 80. 8 ⫹ 6 ⫹ 2 ⫹ 8 ⫹ . . .
8 9 27
the balance is given by
81. 3 ⫺ 1 ⫹ 3 ⫺ 9 ⫹ . . .
1 1
Per兾12共e r t ⫺ 1兲
25 125
82. ⫺6 ⫹ 5 ⫺ 6 ⫹ 36 ⫺ . . . A⫽ .
er兾12 ⫺ 1

Copyright 2011 Cengage Learning. All Rights Reserved. May not be copied, scanned, or duplicated, in whole or in part. Due to electronic rights, some third party content may be suppressed from the eBook and/or eChapter(s).
Editorial review has deemed that any suppressed content does not materially affect the overall learning experience. Cengage Learning reserves the right to remove additional content at any time if subsequent rights restrictions require it.
Section 8.3 Geometric Sequences and Series 597

Annuities In Exercises 99–102, consider making


monthly deposits of P dollars in a savings account
earning an annual interest rate r. Use the results
of Exercises 97 and 98 to find the balances A after
t years if the interest is compounded (a) monthly and
(b) continuously.
99. P ⫽ $50, r ⫽ 7%, t ⫽ 20 years
Image not available due to copyright restrictions
100. P ⫽ $75, r ⫽ 4%, t ⫽ 25 years
101. P ⫽ $100, r ⫽ 5%, t ⫽ 40 years
102. P ⫽ $20, r ⫽ 6%, t ⫽ 50 years

103. Geometry The sides of a square are 16 inches in


length. A new square is formed by connecting the
midpoints of the sides of the original square, and two
of the resulting triangles are shaded. After this process (a) Write a formula in series notation that gives the
is repeated five more times, determine the total area of surface area of the sphereflake.
the shaded region. (b) Write a formula in series notation that gives the
volume of the sphereflake.
(c) Is the surface area of the sphereflake finite or
infinite? Is the volume finite or infinite? If either is
finite, find the value.

104. Geometry The sides of a square are 27 inches in 107. MODELING DATA
length. New squares are formed by dividing the original The table shows the mid-year populations an of China
square into nine squares. The center square is then (in millions) from 2002 through 2008. (Source: U.S.
shaded (see figure). After this process is repeated three Census Bureau)
more times, determine the total area of the shaded
region. Year Population, an
2002 1284.3
2003 1291.5
2004 1298.8
2005 1306.3
2006 1314.0
105. Physics The temperature of water in an ice cube tray
2007 1321.9
is 70⬚F when it is placed in a freezer. Its temperature n
hours after being placed in the freezer is 20% less than 2008 1330.0
1 hour earlier.
(a) Use the exponential regression feature of a
(a) Find a formula for the nth term of the geometric graphing utility to find a geometric sequence that
sequence that gives the temperature of the water n models the data. Let n represent the year, with
hours after it is placed in the freezer. n ⫽ 2 corresponding to 2002.
(b) Find the temperatures of the water 6 hours and (b) Use the sequence from part (a) to describe the rate
12 hours after it is placed in the freezer. at which the population of China is growing.
(c) Use a graphing utility to graph the sequence to (c) Use the sequence from part (a) to predict the
approximate the time required for the water to population of China in 2015. The U.S. Census
freeze. Bureau predicts the population of China will be
106. Fractals In a fractal, a geometric figure is repeated at 1393.4 million in 2015. How does this value
smaller and smaller scales. The sphereflake shown is a compare with your prediction?
computer-generated fractal that was created by Eric (d) Use the sequence from part (a) to determine when
Haines. The radius of the large sphere is 1. Attached to
1 the population of China will reach 1.35 billion.
the large sphere are nine spheres of radius 3. Attached
to each of the smaller spheres are nine spheres of
1
radius 9. This process is continued infinitely.
Eric Haines

Copyright 2011 Cengage Learning. All Rights Reserved. May not be copied, scanned, or duplicated, in whole or in part. Due to electronic rights, some third party content may be suppressed from the eBook and/or eChapter(s).
Editorial review has deemed that any suppressed content does not materially affect the overall learning experience. Cengage Learning reserves the right to remove additional content at any time if subsequent rights restrictions require it.
598 Chapter 8 Sequences, Series, and Probability

108. Manufacturing The manufacturer of a new food Finding a Term of a Geometric Sequence In Exercises
processor plans to produce and sell 8000 units per 115 and 116, find the indicated term of the geometric
year. Each year, 10% of all units sold become sequence.
inoperative. So, 8000 units will be in use after 1 year,
115. a1 ⫽ 100, r ⫽ e x, 9th term
关8000 ⫹ 0.9共8000兲兴 units will be in use after 2 years,
and so on. 4x
116. a1 ⫽ 4, r ⫽ , 6th term
3
(a) Write a formula in series notation for the number
of units that will be operative after n years. 117. Exploration Use a graphing utility to graph each
(b) Find the numbers of units that will be operative function. Identify the horizontal asymptote of the
after 10 years, 15 years, and 20 years. graph and determine its relationship to the sum.
1 ⫺ 共0.5兲x ⬁
冤 冥 兺冢冣
(c) If this trend continues indefinitely, will the number 1 n
(a) f 共x兲 ⫽ 6 , 6
of units that will be operative be finite? If so, how 1 ⫺ 共0.5兲 n⫽0 2
many? If not, explain your reasoning.
1 ⫺ 共0.8兲x ⬁
冤 1 ⫺ 共0.8兲 冥, 兺 2冢5冣
n
4
109. (p. 589) A ball is dropped (b) f 共x兲 ⫽ 2
from a height of 6 feet and begins bouncing, n⫽0

as shown in the figure. The height of each 118. Writing Write a brief paragraph explaining why the
bounce is three-fourths the height of the terms of a geometric sequence decrease in magnitude
previous bounce. Find the total vertical when ⫺1 < r < 1.
distance the ball travels before coming to rest. 119. Writing Write a brief paragraph explaining how to
use the first two terms of a geometric sequence to find
7 the nth term.
6
5 120. C A P S T O N E The terms of a geometric sequence
4 can be written as
3 a1, a2 ⫽ a1r, a3 ⫽ a2r, a4 ⫽ a3r, . . . .
2
Write each term of the sequence in terms of a1 and r.
1
Then, based on the pattern, write the nth term of the
1 2 3 4 5 6 7
geometric sequence. What do you notice?

Conclusions
Cumulative Mixed Review
True or False? In Exercises 110–112, determine whether
the statement is true or false. Justify your answer. 121. Average Speed A truck traveled at an average speed
of 50 miles per hour on a 200-mile trip. On the return
110. For a geometric sequence in which the quotient of trip, the average speed was 42 miles per hour. Find the
term a7 and term a5 is 16, the common ratio is 8. average speed for the round trip.
111. The first n terms of a geometric sequence with a 122. Work Rate Your friend can mow a lawn in 4 hours
common ratio of 1 are the same as the first n terms of and you can mow it in 6 hours. How long will it take
an arithmetic sequence with a common difference of 0 both of you to mow the lawn working together?
if both sequences have the same first term.
112. You can find the nth term of a geometric sequence by Finding a Determinant In Exercises 123 and 124, find
multiplying its common ratio by the first term of the the determinant of the matrix.
sequence raised to the 共n ⫺ 1兲th power.

冤 冥 冤 冥
⫺1 3 4 ⫺1 0 4
123. ⫺2 8 0 124. ⫺4 3 5
Finding the Terms of a Geometric Sequence In
2 5 ⫺1 0 2 ⫺3
Exercises 113 and 114, write the first five terms of the
geometric sequence.
x 125. Make a Decision To work an extended application
113. a1 ⫽ 3, r ⫽ analyzing the monthly profits of a clothing manufacturer
2
over a period of 36 months, visit this textbook’s
1
114. a1 ⫽ , r ⫽ 7x Companion Website.
2
Feng Yu 2010/used under license from Shutterstock.com

Copyright 2011 Cengage Learning. All Rights Reserved. May not be copied, scanned, or duplicated, in whole or in part. Due to electronic rights, some third party content may be suppressed from the eBook and/or eChapter(s).
Editorial review has deemed that any suppressed content does not materially affect the overall learning experience. Cengage Learning reserves the right to remove additional content at any time if subsequent rights restrictions require it.
Section 8.4 The Binomial Theorem 599

8.4 The Binomial Theorem

Binomial Coefficients What you should learn


● Use the Binomial Theorem to
Recall that a binomial is a polynomial that has two terms. In this section, you will study
calculate binomial coefficients.
a formula that provides a quick method of raising a binomial to a power. To begin, look
● Use binomial coefficients to
at the expansion of
write binomial expansions.
共x ⫹ y兲n ● Use Pascal’s Triangle to calculate
binomial coefficients.
for several values of n.
共x ⫹ y兲0 ⫽ 1
Why you should learn it
You can use binomial coefficients to
共x ⫹ y兲1 ⫽ x ⫹ y model and solve real-life problems.
共x ⫹ y兲2 ⫽ x 2 ⫹ 2xy ⫹ y 2 For instance, in Exercise 118 on
page 606, you will use binomial
共x ⫹ y兲3 ⫽ x 3 ⫹ 3x 2 y ⫹ 3xy 2 ⫹ y 3 coefficients to write the expansion
共x ⫹ y兲4 ⫽ x 4 ⫹ 4x 3y ⫹ 6x 2 y 2 ⫹ 4xy 3 ⫹ y 4 of a model that represents the
amount of child support collected in
共x ⫹ y兲5 ⫽ x 5 ⫹ 5x 4y ⫹ 10x 3y 2 ⫹ 10x 2y 3 ⫹ 5xy 4 ⫹ y 5 the United States.
共x ⫹ y兲6 ⫽ x 6 ⫹ 6x5y ⫹ 15x4y 2 ⫹ 20x3y 3 ⫹ 15x2y 4 ⫹ 6xy 5 ⫹ y 6
There are several observations you can make about these expansions.
1. In each expansion, there are n ⫹ 1 terms.
2. In each expansion, x and y have symmetric roles. The powers of x
decrease by 1 in successive terms, whereas the powers of y increase by 1.
3. The sum of the powers of each term is n. For instance, in the expansion of
共x ⫹ y兲5
the sum of the powers of each term is 5.
4⫹1⫽5 3⫹2⫽5

共x ⫹ y兲5 ⫽ x 5 ⫹ 5x 4y1 ⫹ 10x 3y 2 ⫹ 10x 2 y 3 ⫹ 5x1y 4 ⫹ y 5


4. The coefficients increase and then decrease in a symmetric pattern.
The coefficients of a binomial expansion are called binomial coefficients.
To find them, you can use the Binomial Theorem.

The Binomial Theorem (See the proof on page 634.)


In the expansion of 共x ⫹ y兲n
共x ⫹ y兲n ⫽ xn ⫹ nx n ⫺1y ⫹ . . . ⫹n Cr x n ⫺r y r ⫹ . . . ⫹ nxy n ⫺1 ⫹ y n
the coefficient of x n ⫺r y r is
n!
n Cr ⫽ .
共n ⫺ r兲!r!
The symbol

冢r冣
n

is often used in place of n Cr to denote binomial coefficients.

Vasiliy Koval 2010/used under license from Shutterstock.com

Copyright 2011 Cengage Learning. All Rights Reserved. May not be copied, scanned, or duplicated, in whole or in part. Due to electronic rights, some third party content may be suppressed from the eBook and/or eChapter(s).
Editorial review has deemed that any suppressed content does not materially affect the overall learning experience. Cengage Learning reserves the right to remove additional content at any time if subsequent rights restrictions require it.
600 Chapter 8 Sequences, Series, and Probability

Example 1 Finding Binomial Coefficients


Find each binomial coefficient. Technology Tip
a. 8C2 b. 冢103冣 Most graphing utilities
are programmed to
evaluate nC r . For
c. 7C0 d. 冢88冣 instructions on how to use the
nC r feature, see Appendix A;
Solution for specific keystrokes, go to
this textbook’s Companion
8! 共8 ⭈ 7兲 ⭈ 6! 8 ⭈ 7
a. 8C2 ⫽ ⫽ ⫽ ⫽ 28 Website.
6! ⭈ 2! 6! ⭈ 2! 2⭈1
共10 ⭈ 9 ⭈ 8兲 ⭈ 7! 10 ⭈ 9 ⭈ 8
冢 3 冣 ⫽ 7! ⭈ 3! ⫽
10 10!
b. ⫽ ⫽ 120
7! ⭈ 3! 3⭈2⭈1
7!
c. 7C0 ⫽ ⫽1
7! ⭈ 0!

冢8冣 ⫽ 0! ⭈ 8! ⫽ 1
8 8!
d.

Now try Exercise 5.

When r ⫽ 0 and r ⫽ n, as in parts (a) and (b) of Example 1, there is a simple


pattern for evaluating binomial coefficients that works because there will always be
factorial terms that divide out from the expression.
2 factors 3 factors

8⭈7
冢 3 冣 ⫽ 3 ⭈⭈2 ⭈⭈1
10 10 9 8
8C2 ⫽ and
2⭈1

2 factorial 3 factorial

Example 2 Finding Binomial Coefficients


Find each binomial coefficient using the pattern shown above. Explore the Concept
a. 7C3 b. 7C4 c. 12C1 d. 12C11 Find each pair of
binomial coefficients.
Solution
7⭈6
⭈ 5 ⫽ 35 a. 7C0, 7C7 d. 7C1, 7C6
a. 7C3 ⫽
⭈1 3⭈2 b. 8C0, 8C8 e. 8C1, 8C7
7⭈6⭈5⭈4 c.
b. 7C4 ⫽ ⫽ 35 10C0, 10C10 f. 10C1, 10C9
4⭈3⭈2⭈1
What do you observe about
12 the pairs in (a), (b), and (c)?
c. 12C1 ⫽ ⫽ 12
1 What do you observe about
12! 共12兲 ⭈ 11! 12 the pairs in (d), (e), and (f)?
d. 12C11 ⫽ ⫽ ⫽ ⫽ 12 Write two conjectures from
1! ⭈ 11! 1! ⭈ 11! 1
your observations. Develop a
Now try Exercise 7. convincing argument for your
two conjectures.
It is not a coincidence that the results in parts (a) and (b) of Example 2 are the same
and that the results in parts (c) and (d) are the same. In general, it is true that

n Cr ⫽ n Cn ⫺r .
Yuri Arcurs 2010/used under license from Shutterstock.com

Copyright 2011 Cengage Learning. All Rights Reserved. May not be copied, scanned, or duplicated, in whole or in part. Due to electronic rights, some third party content may be suppressed from the eBook and/or eChapter(s).
Editorial review has deemed that any suppressed content does not materially affect the overall learning experience. Cengage Learning reserves the right to remove additional content at any time if subsequent rights restrictions require it.
Section 8.4 The Binomial Theorem 601

Binomial Expansions
As mentioned at the beginning of this section, when you write out the coefficients for
a binomial that is raised to a power, you are expanding a binomial. The formulas for
binomial coefficients give you an easy way to expand binomials, as demonstrated in the
next four examples.

Example 3 Expanding a Binomial


Write the expansion of the expression 共x ⫹ 1兲3.

Solution
The binomial coefficients are

3C0 ⫽ 1, 3C1 ⫽ 3, 3C2 ⫽ 3, and 3C3 ⫽ 1.


Therefore, the expansion is as follows.
共x ⫹ 1兲3 ⫽ 共1兲x 3 ⫹ 共3兲x 2共1兲 ⫹ 共3兲 x共12兲 ⫹ 共1兲共13兲
⫽ x 3 ⫹ 3x 2 ⫹ 3x ⫹ 1
Now try Exercise 23.

To expand binomials representing differences, rather than sums, you alternate


signs, as shown in the next example.

Example 4 Expanding a Binomial


Write the expansion of the expression 共x ⫺ 1兲3.

Solution
Expand using the binomial coefficient from Example 3.
共x ⫺ 1兲3 ⫽ 关x ⫹ 共⫺1兲兴3
⫽ 共1兲x3 ⫹ 共3兲x2共⫺1兲 ⫹ 共3兲x共⫺1兲2 ⫹ 共1兲共⫺1兲3
⫽ x3 ⫺ 3x2 ⫹ 3x ⫺ 1
Now try Exercise 27.

Example 5 Expanding Binomial Expressions Technology Tip


Write the expansion of each expression. You can use a graphing
a. 共2x ⫺ 3兲 4 b. 共x ⫺ 2y兲 4 utility to check the
expansion in Example
Solution 5(a) by graphing the original
The binomial coefficients are binomial expression and the
expansion in the same viewing
4C0 ⫽ 1, 4C1 ⫽ 4, 4C2 ⫽ 6, 4C3 ⫽ 4, and 4C4 ⫽ 1. window. The graphs should
coincide, as shown below.
Therefore, the expansions are as follows.
3
a. 共2x ⫺ 3兲4 ⫽ 共1兲共2x兲4 ⫺ 共4兲共2x兲3共3兲 ⫹ 共6兲共2x兲2共32兲 ⫺ 共4兲共2x兲共33兲 ⫹ 共1兲共34兲
⫽ 16x4 ⫺ 96x3 ⫹ 216x2 ⫺ 216x ⫹ 81
b. 共x ⫺ 2y兲4 ⫽ 共1兲x 4 ⫺ 共4兲x3共2y兲 ⫹ 共6兲x 2共2y兲2 ⫺ 共4兲x共2y兲3 ⫹ 共1兲共2y兲4 −1 5

⫽ x 4 ⫺ 8x 3y ⫹ 24x 2y 2 ⫺ 32xy 3 ⫹ 16y 4


−1
Now try Exercise 35.

Copyright 2011 Cengage Learning. All Rights Reserved. May not be copied, scanned, or duplicated, in whole or in part. Due to electronic rights, some third party content may be suppressed from the eBook and/or eChapter(s).
Editorial review has deemed that any suppressed content does not materially affect the overall learning experience. Cengage Learning reserves the right to remove additional content at any time if subsequent rights restrictions require it.
602 Chapter 8 Sequences, Series, and Probability

Example 6 Expanding a Binomial


Write the expansion of the expression
共x2 ⫹ 4兲3.

Solution
Expand using the binomial coefficients from Example 3.
共x2 ⫹ 4兲3 ⫽ 共1兲共x2兲3 ⫹ 共3兲共x2兲2共4兲 ⫹ 共3兲 x2共42兲 ⫹ 共1兲共43兲
⫽ x 6 ⫹ 12x 4 ⫹ 48x2 ⫹ 64
Now try Exercise 37.

Sometimes you will need to find a specific term in a binomial expansion. Instead
of writing out the entire expansion, you can use the fact that, from the Binomial
Theorem, the 共r ⫹ 1兲th term is

n Cr x n⫺r y r.
For instance, to find the third term of the expression in Example 6, you could use the
formula above with n ⫽ 3 and r ⫽ 2 to obtain

3C2 共x2兲3⫺2 ⭈ 42 ⫽ 3共x2兲 ⭈ 16


⫽ 48x2.

Example 7 Finding a Term or Coefficient in a Binomial Expansion

a. Find the sixth term of 共a ⫹ 2b兲8.


b. Find the coefficient of the term a6b 5 in the expansion of 共2a ⫺ 5b兲11.

Solution
a. Because the formula is for the 共r ⫹ 1兲th term, r is one less than the number of the
term you need. So, to find the sixth term in this binomial expansion, use r ⫽ 5,
n ⫽ 8, x ⫽ a and y ⫽ 2b.

nCr x
n⫺r y r ⫽ 8C5a8⫺5共2b兲5
⫽ 56 ⭈ a3 ⭈ 共2b兲5
⫽ 56共25兲a 3b5
⫽ 1792a 3b5
b. Note that
共2a ⫺ 5b兲11 ⫽ 关2a ⫹ 共⫺5b兲兴11.
So,
n ⫽ 11, r ⫽ 5, x ⫽ 2a, and y ⫽ ⫺5b.
Substitute these values to obtain

nCr x n⫺r y r ⫽ 11C5共2a兲6共⫺5b兲5


⫽ 共462兲共64a6兲共⫺3125b 5兲
⫽ ⫺92,400,000a6b5.
So, the coefficient is ⫺92,400,000.
Now try Exercises 55 and 67.

Copyright 2011 Cengage Learning. All Rights Reserved. May not be copied, scanned, or duplicated, in whole or in part. Due to electronic rights, some third party content may be suppressed from the eBook and/or eChapter(s).
Editorial review has deemed that any suppressed content does not materially affect the overall learning experience. Cengage Learning reserves the right to remove additional content at any time if subsequent rights restrictions require it.
Section 8.4 The Binomial Theorem 603

Pascal’s Triangle
There is a convenient way to remember the pattern for binomial coefficients. By Explore the Concept
arranging the coefficients in a triangular pattern, you obtain the following array, which is
called Pascal’s Triangle. This triangle is named after the famous French mathematician Complete the table and
Blaise Pascal (1623–1662). describe the result.

1
n r nCr nCn⫺r
1 1
1 2 1
9 5 䊏 䊏
1 3 3 1 7 1 䊏 䊏
1 4 6 4 1 12 4 䊏 䊏
4 ⫹ 6 ⫽ 10
1 5 10 10 5 1 6 0 䊏 䊏
1 6 15 20 15 6 1
1 7 21 35 35 21 7 1 15 ⫹ 6 ⫽ 21
10 7 䊏 䊏
The first and last number in each row of Pascal’s Triangle is 1. Every other number in What characteristics of Pascal’s
each row is formed by adding the two numbers immediately above the number. Triangle are illustrated by this
Pascal noticed that the numbers in this triangle are precisely the same numbers as the table?
coefficients of binomial expansions, as follows.
共x ⫹ y兲0 ⫽ 1 0th row

共x ⫹ y兲1 ⫽ 1x ⫹ 1y 1st row

共x ⫹ y兲2 ⫽ 1x 2 ⫹ 2xy ⫹ 1y 2 2nd row

共x ⫹ y兲3 ⫽ 1x 3 ⫹ 3x 2 y ⫹ 3xy 2 ⫹ 1y 3 3rd row


..
共x ⫹ y兲4 ⫽ 1x4 ⫹ 4x 3 y ⫹ 6x 2y 2 ⫹ 4xy 3 ⫹ 1y4 .
共x ⫹ y兲5 ⫽ 1x5 ⫹ 5x4y ⫹ 10x 3y 2 ⫹ 10x 2 y 3 ⫹ 5xy4 ⫹ 1y 5
共x ⫹ y兲6 ⫽ 1x 6 ⫹ 6x5y ⫹ 15x4y 2 ⫹ 20x3y 3 ⫹ 15x 2 y4 ⫹ 6xy5 ⫹ 1y 6
共x ⫹ y兲7 ⫽ 1x7 ⫹ 7x 6y ⫹ 21x 5y 2 ⫹ 35x4y 3 ⫹ 35x3y4 ⫹ 21x 2 y 5 ⫹ 7xy 6 ⫹ 1y7
The top row of Pascal’s Triangle is called the zeroth row because it corresponds to
the binomial expansion
共x ⫹ y兲0 ⫽ 1.
Similarly, the next row is called the first row because it corresponds to the binomial
expansion
共x ⫹ y兲1 ⫽ 1共x兲 ⫹ 1共y兲.
In general, the nth row of Pascal’s Triangle gives the coefficients of 共x ⫹ y兲n.

Example 8 Using Pascal’s Triangle


Use the seventh row of Pascal’s Triangle to find the binomial coefficients.

8C0 8C1 8C2 8C3 8C4 8C5 8C6 8C7 8C8

Solution
1 7 21 35 35 21 7 1 Seventh row
1 8 28 56 70 56 28 8 1

8C0 8C1 8C2 8 C3 8C4 8C5 8C6 8C7 8C8

Now try Exercise 71.

Copyright 2011 Cengage Learning. All Rights Reserved. May not be copied, scanned, or duplicated, in whole or in part. Due to electronic rights, some third party content may be suppressed from the eBook and/or eChapter(s).
Editorial review has deemed that any suppressed content does not materially affect the overall learning experience. Cengage Learning reserves the right to remove additional content at any time if subsequent rights restrictions require it.
604 Chapter 8 Sequences, Series, and Probability

See www.CalcChat.com for worked-out solutions to odd-numbered exercises.


8.4 Exercises For instructions on how to use a graphing utility, see Appendix A.

Vocabulary and Concept Check


In Exercises 1 and 2, fill in the blanks.
1. The notation used to denote a binomial coefficient is _______ or _______ .
2. When you write out the coefficients for a binomial that is raised to a power, you
are _______ a _______ .

3. List two ways to find binomial coefficients.


4. In the expression of 共x ⫹ y兲3, what is the sum of the powers of the third term?

Procedures and Problem Solving


Finding Binomial Coefficients In Exercises 5–16, find 49. 共4x ⫺ 1兲3 ⫺ 2共4x ⫺ 1兲4
the binomial coefficient. 50. 共x ⫹ 3兲5 ⫺ 4共x ⫹ 3兲4
5. 7C5 6. 8C6 51. 2共x ⫺ 3兲4 ⫹ 5共x ⫺ 3兲 2
7. 20C15 8. 12C3 52. 3共x ⫹ 1兲5 ⫹ 4共x ⫹ 1兲3
9. 14C1 10. 18C17 53. ⫺3共x ⫺ 2兲3 ⫺ 4共x ⫹ 1兲6

11. 冢120冣 12. 冢 冣


20
20
54. 5共x ⫹ 2兲5 ⫺ 2共x ⫺ 1兲2

Finding a Term in a Binomial Expansion In Exercises


13. 冢 冣 14. 冢 冣
10 10
4 6 55–62, find the specified nth term in the expansion of the
binomial.
15. 冢
98 冣
16. 冢
2 冣
100 100
55. 共x ⫹ 8兲10, n ⫽ 4 56. 共x ⫺ 5兲6, n ⫽ 7
57. 共x ⫺ 6y兲5, n ⫽ 3
Finding Binomial Coefficients In Exercises 17–22, use a 58. 共x ⫺ 10z兲7, n ⫽ 4
graphing utility to find nCr .
59. 共4x ⫹ 3y兲9, n ⫽ 8
17. 41C36 18. 34C4 60. 共5a ⫹ 6b兲5, n ⫽ 5
19. 50C48 20. 500C498 61. 共10x ⫺ 3y兲12, n ⫽ 10
21. 250C2 22. 1000C2 62. 共7x ⫹ 2y兲15, n ⫽ 7

Expanding a Binomial In Exercises 23–54, use the Finding a Coefficient in a Binomial Expansion In
Binomial Theorem to expand and simplify the expression. Exercises 63–70, find the coefficient a of the given term
23. 共x ⫹ 2兲4 24. 共x ⫹ 1兲6 in the expansion of the binomial.
25. 共a ⫹ 3兲3 26. 共a ⫹ 2兲4 Binomial Term
27. 共 y ⫺ 4兲3 28. 共 y ⫺ 2兲5 63. 共x ⫹ 3兲 12 ax 5
29. 共x ⫹ y兲5 30. 共x ⫹ y兲6 64. 共x ⫹ 4兲12 ax4
31. 共r ⫹ 3s兲6 32. 共4x ⫹ 3y兲4 65. 共x ⫺ 2y兲10 ax 8y 2
33. 共x ⫺ y兲5 34. 共2x ⫺ y兲5 66. 共4x ⫺ y兲10 ax 2y 8
35. 共1 ⫺ 4x兲3 36. 共5 ⫺ 2y兲3 67. 共3x ⫺ 2y兲9 ax6y3
37. 共x2 ⫹ 2兲4 38. 共3 ⫺ y2兲3 68. 共2x ⫺ 3y兲8 ax 4y 4
39. 共x2 ⫺ 5兲5 40. 共 y2 ⫹ 1兲6 69. 共x 2 ⫹ y兲10 ax 8y 6
41. 共x 2 ⫹ y2兲4 42. 共x 2 ⫹ y 2兲6 70. 共z 2 ⫺ 1兲12 az 6
43. 共x3 ⫺ y兲6 44. 共2x3 ⫺ y兲5
Using Pascal’s Triangle In Exercises 71–74, use Pascal’s
冢 冣 冢 冣
5 6
1 1
45. ⫹y 46. ⫹ 2y Triangle to find the binomial coefficient.
x x
71. 7C4 72. 6C3
冢 冣 冢 冣
4 5
2 2
47. ⫺y 48. ⫺ 3y 73. 6C5 74. 5C2
x x

Copyright 2011 Cengage Learning. All Rights Reserved. May not be copied, scanned, or duplicated, in whole or in part. Due to electronic rights, some third party content may be suppressed from the eBook and/or eChapter(s).
Editorial review has deemed that any suppressed content does not materially affect the overall learning experience. Cengage Learning reserves the right to remove additional content at any time if subsequent rights restrictions require it.
Section 8.4 The Binomial Theorem 605

Using Pascal’s Triangle In Exercises 75–80, expand the 109. f 共x兲 ⫽ x 3 ⫺ 4x, g共x兲 ⫽ f 共x ⫹ 4兲
binomial by using Pascal’s Triangle to determine the 110. f 共x兲 ⫽ ⫺x4 ⫹ 4x 2 ⫺ 1, g共x兲 ⫽ f 共x ⫺ 3兲
coefficients.
75. 共3t ⫺ 2v兲4 76. 共5v ⫺ 2z兲4 Using a Graphing Utility In Exercises 111 and 112, use
a graphing utility to graph the functions in the given
77. 共2x ⫺ 3y兲5 78. 共5y ⫹ 2兲5
order and in the same viewing window. Compare the
79. 共x ⫹ 2y兲5 80. 共2v ⫹ 3兲6 graphs. Which two functions have identical graphs, and
why?
Using the Binomial Theorem In Exercises 81–84, use the
Binomial Theorem to expand and simplify the expression. 111. (a) f 共x兲 ⫽ 共1 ⫺ x兲3
g共x兲 ⫽ 1 ⫺ 3x
81. 共冪x ⫹ 5兲3 82. 共2冪t ⫺ 1兲
3 (b)
(c) h共x兲 ⫽ 1 ⫺ 3x ⫹ 3x 2
83. 共x 2兾3 ⫺ y1兾3兲3 84. 共u3兾5 ⫹ v1兾5兲5
(d) p共x兲 ⫽ 1 ⫺ 3x ⫹ 3x 2 ⫺ x 3
f 共x兲 ⫽ 共1 ⫺ 12x兲
4
Expanding an Expression In Exercises 85–90, expand 112. (a)
the expression in the difference quotient and simplify. (b) g共x兲 ⫽ 1 ⫺ 2x ⫹ 32x 2
f 冇x ⴙ h冈 ⴚ f 冇x冈 (c) h共x兲 ⫽ 1 ⫺ 2x ⫹ 32x 2 ⫺ 12x 3
, hⴝ0
h (d) p共x兲 ⫽ 1 ⫺ 2x ⫹ 32x 2 ⫺ 12x 3 ⫹ 16
1 4
x
85. f 共x兲 ⫽ x 3 86. f 共x兲 ⫽ x 4
Finding a Probability In Exercises 113–116, consider
87. f共x兲 ⫽ x6 88. f共x兲 ⫽ x8
n independent trials of an experiment in which each
1 trial has two possible outcomes, success or failure.
89. f 共x兲 ⫽ 冪x 90. f 共x兲 ⫽
x The probability of a success on each trial is p and the
probability of a failure is q ⴝ 1 ⴚ p. In this context,
Expanding a Complex Number In Exercises 91–104, use the term nCk p kq nⴚk in the expansion of 冇 p ⴙ q冈 n gives
the Binomial Theorem to expand the complex number. the probability of k successes in the n trials of the
Simplify your result. 共 Remember that i ⴝ 冪ⴚ1.兲 experiment.
共1 ⫹ i 兲4
91. 92. 共4 ⫺ i 兲5 113. A fair coin is tossed seven times. To find the
共4 ⫹ i兲4
93. 94. 共2 ⫺ i兲5 probability of obtaining four heads, evaluate the term
共2 ⫺ 3i 兲6
95. 96. 共3 ⫺ 2i兲6 7 C4 2 共 1 兲4共 12 兲3
共5 ⫹ 冪⫺16兲3
97. 98. 共5 ⫹ 冪⫺9 兲3 in the expansion of 共 2 ⫹ 2 兲 .
1 1 7

共4 ⫹ 冪3i兲4
99. 100. 共5 ⫺ 冪3i兲4 114. The probability of a baseball player getting a hit
1 冪3 3
冢 冣 冢
1 冪3 3

1
101. ⫺ ⫹ i 102. ⫺ i during any given time at bat is 4. To find the
2 2 2 2 probability that the player gets three hits during the

冢 冣 冢 冣
1 冪3 3 1 冪3 3 next 10 times at bat, evaluate the term
103. ⫺ i 104. ⫺ i
4 4 3 3
10C3 4 共1 兲3共34 兲7
in the expansion of 共4 ⫹ 4 兲 .
1 3 10
Using the Binomial Theorem to Approximate In Exercises
105–108, use the Binomial Theorem to approximate the 115. The probability of a sales representative making a sale
1
quantity accurate to three decimal places. For example, to any one customer is 3. The sales representative
in Exercise 105, use the expansion makes eight contacts a day. To find the probability of
making four sales, evaluate the term
冇1.02冈8 ⴝ 冇1 ⴙ 0.02冈8
ⴝ 1 ⴙ 8冇0.02冈 ⴙ 28冇0.02冈2 ⴙ . . . . 8C4 3 共1 兲4共23 兲4
in the expansion of 共3 ⫹ 3 兲 .
1 2 8
105. 共1.02兲 8 106. 共2.005兲 10
116. To find the probability that the sales representative in
107. 共2.99兲12 108. 共1.98兲9
Exercise 115 makes four sales when the probability of
1
a sale to any one customer is 2, evaluate the term
Using the Binomial Theorem In Exercises 109 and 110,
use a graphing utility to graph f and g in the same 8C4 2 共 1 兲4共 12 兲4
in the expansion of 共 2 ⫹ 2 兲 .
viewing window. What is the relationship between the 1 1 8

two graphs? Use the Binomial Theorem to write the


polynomial function g in standard form.

Copyright 2011 Cengage Learning. All Rights Reserved. May not be copied, scanned, or duplicated, in whole or in part. Due to electronic rights, some third party content may be suppressed from the eBook and/or eChapter(s).
Editorial review has deemed that any suppressed content does not materially affect the overall learning experience. Cengage Learning reserves the right to remove additional content at any time if subsequent rights restrictions require it.
606 Chapter 8 Sequences, Series, and Probability

117. MODELING DATA (b) Use a graphing utility to graph f and g in the same
viewing window.
The per capita consumption of bottled water f (in
gallons) in the United States from 1990 through 2008
can be approximated by the model
Conclusions
True or False? In Exercises 119–122, determine whether
f 共t兲 ⫽ 0.044t2 ⫹ 0.44t ⫹ 8.3, 0 ⱕ t ⱕ 18
the statement is true or false. Justify your answer.
where t represents the year, with t ⫽ 0 corresponding
119. The Binomial Theorem can be used to produce each
to 1990 (see figure). (Source: Economic Research
row of Pascal’s Triangle.
Service, U.S. Department of Agriculture)
120. A binomial that represents a difference cannot always
f(t)
be accurately expanded using the Binomial Theorem.
Per capita consumption of
bottled water (in gallons)

35 121. One of the terms in the expansion of 共x ⫺ 2y兲12 is


30 7920x 4y8.
25
122. The x10-term and the x14-term in the expansion of
20
共x2 ⫹ 3兲12 have identical coefficients.
15
10
123. Think About It How many terms are in the expansion
5
of 共x ⫹ y兲n ?
t
3 6 9 12 15 18 124. Writing In your own words, explain how to form the
Year (0 ↔ 1990) rows of Pascal’s Triangle. Then form rows 8–10 of
Pascal’s Triangle.
(a) Adjust the model so that t ⫽ 0 corresponds to
2000 rather than 1990. To do this, shift the graph 125. Error Analysis You are a math instructor and receive
of f 10 units to the left and obtain g共t兲 ⫽ f 共t ⫹ 10兲. the following solutions from one of your students on a
Write g共t兲 in standard form. quiz. Find the error(s) in each solution and write a
short paragraph discussing ways that your student
(b) Use a graphing utility to graph f and g in the same
could avoid the error(s) in the future.
viewing window.
(a) Find the second term in the expansion of
共2x ⫺ 3y兲5.
118. (p. 599) The amounts f
(in millions of dollars) of child support 5共2x兲4共3y兲2 ⫽ 720x 4y2
collected in the United States from 1990 (b) Find the fourth term in the expansion of
through 2007 can be approximated by the 共12 x ⫹ 7y兲6.
model
6C4 2 x共1 兲2共7y兲 4 ⫽ 9003.75x2y 4
f 共t兲 ⫽ 0.26t 2 ⫹ 1162.5t ⫹ 5531,
0 ⱕ t ⱕ 17
126. C A P S T O N E How do the expansions of 共x ⫹ y兲n
where t represents the year, with t ⫽ 0 corresponding and 共x ⫺ y兲n differ? Support your explanation with
to 1990 (see figure). (Source: U.S. Department of an example.
Health and Human Services)
f(t)
Proof In Exercises 127–130, prove the property for all
Child support collections

integers r and n, where 0 ⱕ r ⱕ n.


(in millions of dollars)

30,000
25,000
127. nCr ⫽ nCn⫺r
20,000
128. nC0 ⫺ nC1 ⫹ nC2 ⫺ . . . ± nCn ⫽ 0
15,000
10,000
129. n⫹1Cr ⫽ n Cr ⫹ n Cr⫺1
5,000 130. The sum of the numbers in the nth row of Pascal’s
t
Triangle is 2n.
3 6 9 12 15 18
Year (0 ↔ 1990) Cumulative Mixed Review
(a) Adjust the model so that t ⫽ 0 corresponds to 2000 Finding the Inverse of a Matrix In Exercises 131 and 132,
rather than 1990. To do this, shift the graph of f find the inverse of the matrix.
10 units to the left and obtain g共t兲 ⫽ f 共t ⫹ 10). ⫺1 ⫺4 ⫺12
Write g共t兲 in standard form. 131. 冤 1 2 冥 132. 冤112 ⫺2 冥
Vasiliy Koval 2010/used under license from Shutterstock.com

Copyright 2011 Cengage Learning. All Rights Reserved. May not be copied, scanned, or duplicated, in whole or in part. Due to electronic rights, some third party content may be suppressed from the eBook and/or eChapter(s).
Editorial review has deemed that any suppressed content does not materially affect the overall learning experience. Cengage Learning reserves the right to remove additional content at any time if subsequent rights restrictions require it.
Section 8.5 Counting Principles 607

8.5 Counting Principles

Simple Counting Problems What you should learn


● Solve simple counting problems.
The last two sections of this chapter present a brief introduction to some of the basic
● Use the Fundamental Counting
counting principles and their application to probability. In the next section, you will see
Principle to solve more
that much of probability has to do with counting the number of ways an event can occur.
complicated counting problems.
The following two examples describe simple counting problems.
● Use permutations to solve
counting problems.
Example 1 Selecting Pairs of Numbers at Random ● Use combinations to solve
counting problems.
Eight pieces of paper are numbered from 1 to 8 and placed in a box. One piece of paper
is drawn from the box, its number is written down, and the piece of paper is returned Why you should learn it
to the box. Then, a second piece of paper is drawn from the box, and its number is You can use counting principles to
written down. Finally, the two numbers are added together. In how many different ways solve counting problems that occur
can a sum of 12 be obtained? in real life. For instance, in Exercise
65 on page 615, you are asked to use
Solution
counting principles to determine in
To solve this problem, count the number of different ways that a sum of 12 can be how many ways a player can select
obtained using two numbers from 1 to 8. six numbers in a Powerball lottery.

First number 4 5 6 7 8

Second number 8 7 6 5 4

From this list, you can see that a sum of 12 can occur in five different ways.
Now try Exercise 9.

Example 2 Selecting Pairs of Numbers at Random


Eight pieces of paper are numbered from 1 to 8 and placed in a box. Two pieces of paper
are drawn from the box at the same time, and the numbers on the pieces of paper are
written down and totaled. In how many different ways can a sum of 12 be obtained?

Solution
To solve this problem, count the number of different ways that a sum of 12 can be
obtained using two different numbers from 1 to 8.

First number 4 5 7 8

Second number 8 7 5 4

So, a sum of 12 can be obtained in four different ways.


Now try Exercise 11.

The difference between the counting problems in Examples 1 and 2 can be


described by saying that the random selection in Example 1 occurs with replacement,
whereas the random selection in Example 2 occurs without replacement, which
eliminates the possibility of choosing two 6’s.
kaarsten 2010/used under license from Shutterstock.com

Copyright 2011 Cengage Learning. All Rights Reserved. May not be copied, scanned, or duplicated, in whole or in part. Due to electronic rights, some third party content may be suppressed from the eBook and/or eChapter(s).
Editorial review has deemed that any suppressed content does not materially affect the overall learning experience. Cengage Learning reserves the right to remove additional content at any time if subsequent rights restrictions require it.
608 Chapter 8 Sequences, Series, and Probability

The Fundamental Counting Principle


Examples 1 and 2 describe simple counting problems in which you can list each
possible way that an event can occur. When it is possible, this is always the best way to
solve a counting problem. However, some events can occur in so many different
ways that it is not feasible to write out the entire list. In such cases, you must rely on
formulas and counting principles. The most important of these is the Fundamental
Counting Principle.

Fundamental Counting Principle


Let E1 and E2 be two events. The first event E1 can occur in m1 different ways.
After E1 has occurred, E2 can occur in m2 different ways. The number of ways
that the two events can occur is m1 ⭈ m2.

The Fundamental Counting Principle can be extended to three or more events. For
instance, the number of ways that three events E1, E2, and E3 can occur is
m1 ⭈ m2 ⭈ m3.

Example 3 Using the Fundamental Counting Principle


How many different pairs of letters from the English alphabet are possible?

Solution
There are two events in this situation. The first event is the choice of the first letter,
and the second event is the choice of the second letter. Because the English alphabet
contains 26 letters, it follows that the number of two-letter pairs is
26 ⭈ 26 ⫽ 676.
Now try Exercise 13.

Example 4 Using the Fundamental Counting Principle


Telephone numbers in the United States currently have 10 digits. The first three are the
area code and the next seven are the local telephone number. How many different
telephone numbers are possible within each area code? (Note that at this time, a local
telephone number cannot begin with 0 or 1.)

Solution
Because the first digit cannot be 0 or 1, there are only eight choices for the first digit.
For each of the other six digits, there are 10 choices.

Area code Local number

8 10 10 10 10 10 10

So, the number of local telephone numbers that are possible within each area code is
8 ⭈ 10 ⭈ 10 ⭈ 10 ⭈ 10 ⭈ 10 ⭈ 10 ⫽ 8,000,000.
Now try Exercise 19.

Copyright 2011 Cengage Learning. All Rights Reserved. May not be copied, scanned, or duplicated, in whole or in part. Due to electronic rights, some third party content may be suppressed from the eBook and/or eChapter(s).
Editorial review has deemed that any suppressed content does not materially affect the overall learning experience. Cengage Learning reserves the right to remove additional content at any time if subsequent rights restrictions require it.
Section 8.5 Counting Principles 609

Permutations
One important application of the Fundamental Counting Principle is in determining the
number of ways that n elements can be arranged (in order). An ordering of n elements
is called a permutation of the elements.

Definition of Permutation
A permutation of n different elements is an ordering of the elements such that
one element is first, one is second, one is third, and so on.

Example 5 Finding the Number of Permutations of n Elements


How many permutations of the following letters are possible?
A B C D E F

Solution
Consider the following reasoning.
First position: Any of the six letters
Second position: Any of the remaining five letters
Third position: Any of the remaining four letters
Fourth position: Any of the remaining three letters
Fifth position: Either of the remaining two letters
Sixth position: The one remaining letter
So, the numbers of choices for the six positions are as follows.

Permutations of six letters

6 5 4 3 2 1

The total number of permutations of the six letters is


6! ⫽ 6 ⭈ 5 ⭈ 4 ⭈ 3 ⭈ 2 ⭈ 1 ⫽ 720.
Now try Exercise 37.

Number of Permutations of n Elements


The number of permutations of n elements is given by
n ⭈ 共n ⫺ 1兲 . . . 4 ⭈ 3 ⭈ 2 ⭈ 1 ⫽ n!.
In other words, there are n! different ways that n elements can be ordered.

It is useful, on occasion, to order a subset of a collection of elements rather than


the entire collection. For example, you might want to choose and order r elements out
of a collection of n elements. Such an ordering is called a permutation of n elements
taken r at a time.

Copyright 2011 Cengage Learning. All Rights Reserved. May not be copied, scanned, or duplicated, in whole or in part. Due to electronic rights, some third party content may be suppressed from the eBook and/or eChapter(s).
Editorial review has deemed that any suppressed content does not materially affect the overall learning experience. Cengage Learning reserves the right to remove additional content at any time if subsequent rights restrictions require it.
610 Chapter 8 Sequences, Series, and Probability

Example 6 Counting Horse Race Finishes


Eight horses are running in a race. In how many different ways can these
horses come in first, second, and third? (Assume that there are no ties.)

Solution
Here are the different possibilities.
Win (first position): Eight choices
Place (second position): Seven choices
Show (third position): Six choices
The numbers of choices for the three positions are as follows.

Different orders of horses

Jockey

8 7 6

So, using the Fundamental Counting Principle, you can determine that there are
8 ⭈ 7 ⭈ 6 ⫽ 336
different ways in which the eight horses can come in first, second, and third.
Now try Exercise 41.

Permutations of n Elements Taken r at a Time


The number of permutations of n elements taken r at a time is given by
n!
n Pr ⫽
共n ⫺ r兲!
⫽ n共n ⫺ 1兲共n ⫺ 2兲 . . . 共n ⫺ r ⫹ 1兲.

Using this formula, you can rework Example 6 to find that the number of
permutations of eight horses taken three at a time is
8!
8 P3 ⫽
共8 ⫺ 3兲!
8!

5!
8⭈7 ⭈6 ⭈ 5!

5!
⫽ 336
which is the same answer obtained in the example.

Technology Tip
Most graphing utilities are programmed to evaluate nPr. Figure 8.12
shows how one graphing utility evaluates the permutation 8 P3. For
instructions on how to use the n Pr feature, see Appendix A; for specific
keystrokes, go to this textbook’s Companion Website. Figure 8.12

MAT 2010/used under license from Shutterstock.com

Copyright 2011 Cengage Learning. All Rights Reserved. May not be copied, scanned, or duplicated, in whole or in part. Due to electronic rights, some third party content may be suppressed from the eBook and/or eChapter(s).
Editorial review has deemed that any suppressed content does not materially affect the overall learning experience. Cengage Learning reserves the right to remove additional content at any time if subsequent rights restrictions require it.
Section 8.5 Counting Principles 611

Remember that for permutations, order is important. So, to find the possible
permutations of the letters
A, B, C, and D
taken three at a time, the permutations (A, B, D) and (B, A, D) would be counted as
different because the order of the elements is different.
Consider, however, the possible permutations of the letters A, A, B, and C. The
total number of permutations of the four letters would be

4 P4 ⫽ 4!.
However, not all of these arrangements would be distinguishable because there are
two A’s in the list. To find the number of distinguishable permutations, you can use the
following formula.

Distinguishable Permutations
Suppose a set of n objects has n1 of one kind of object, n2 of a second kind, n3 of
a third kind, and so on, with
n ⫽ n1 ⫹ n2 ⫹ n3 ⫹ . . . ⫹ nk.
The number of distinguishable permutations of the n objects is given by
n!
.
n1! ⭈ n 2! ⭈ n 3! ⭈ . . . ⭈ nk !

Example 7 Distinguishable Permutations


In how many distinguishable ways can the letters in BANANA be written?

Solution
This word has six letters, of which three are A’s, two are N’s, and one is a B. So, the
number of distinguishable ways in which the letters can be written is
6! 6 ⭈ 5 ⭈ 4 ⭈ 3!
⫽ ⫽ 60.
3! ⭈ 2! ⭈ 1! 3! ⭈ 2!
The 60 different distinguishable permutations are as follows.
AAABNN AAANBN AAANNB AABANN
AABNAN AABNNA AANABN AANANB
AANBAN AANBNA AANNAB AANNBA
ABAANN ABANAN ABANNA ABNAAN
ABNANA ABNNAA ANAABN ANAANB
ANABAN ANABNA ANANAB ANANBA
ANBAAN ANBANA ANBNAA ANNAAB
ANNABA ANNBAA BAAANN BAANAN
BAANNA BANAAN BANANA BANNAA
BNAAAN BNAANA BNANAA BNNAAA
NAAABN NAAANB NAABAN NAABNA
NAANAB NAANBA NABAAN NABANA
NABNAA NANAAB NANABA NANBAA
NBAAAN NBAANA NBANAA NBNAAA
NNAAAB NNAABA NNABAA NNBAAA
Now try Exercise 49.

Copyright 2011 Cengage Learning. All Rights Reserved. May not be copied, scanned, or duplicated, in whole or in part. Due to electronic rights, some third party content may be suppressed from the eBook and/or eChapter(s).
Editorial review has deemed that any suppressed content does not materially affect the overall learning experience. Cengage Learning reserves the right to remove additional content at any time if subsequent rights restrictions require it.
612 Chapter 8 Sequences, Series, and Probability

Combinations
When you count the number of possible permutations of a set of elements, order is
important. As a final topic in this section, you will look at a method for selecting subsets
of a larger set in which order is not important. Such subsets are called combinations of
n elements taken r at a time. For instance, the combinations
再A, B, C冎 and 再B, A, C冎
are equivalent because both sets contain the same three elements, and the order in which
the elements are listed is not important. So, you would count only one of the two sets.
A common example of a combination is a card game in which the player is free to
reorder the cards after they have been dealt.

Combinations of n Elements Taken r at a Time


The number of combinations of n elements taken r at a time is given by
n!
n Cr ⫽ .
共n ⫺ r兲!r!

Note that the formula for nCr is the same one given for binomial coefficients.

Example 8 Combinations of n Elements Taken r at a Time Study Tip


a. In how many different ways can three letters be chosen from the letters A, B, C, D, In Example 8(a), you
and E? (The order of the three letters is not important.) could also make a
b. A standard poker hand consists of five cards dealt from a deck of 52. How many list of the different
different poker hands are possible? (After the cards are dealt, the player may reorder combinations of three letters
them, so order is not important.) chosen from five letters.
再A, B, C冎 再A, B, D冎
Solution 再A, B, E冎 再A, C, D冎
a. You can find the number of different ways in which the letters can be chosen by 再A, C, E冎 再A, D, E冎
using the formula for the number of combinations of five elements taken three at a 再B, C, D冎 再B, C, E冎
time, as follows. 再B, D, E冎 再C, D, E冎
2 From this list, you can conclude
5! 5⭈4 ⭈ 3! ⫽ 10
5C3 ⫽ ⫽ that there are 10 different ways
2!3! 2⭈1 ⭈ 3! in which three letters can be
b. You can find the number of different poker hands by using the formula for the chosen from five letters, which
number of combinations of 52 elements taken five at a time, as follows. is the same answer obtained in
Example 8(a).
52! 52 ⭈ 51 ⭈ 50 ⭈ 49 ⭈ 48 ⭈ 47!
52C5 ⫽ ⫽ ⫽ 2,598,960
47!5! 5 ⭈ 4 ⭈ 3 ⭈ 2 ⭈ 1 ⭈ 47!
Now try Exercise 61.

When solving problems involving counting principles, you need to be able to


distinguish among the various counting principles in order to determine which is
necessary to solve the problem correctly. To do this, ask yourself the following
questions.
1. Is the order of the elements important? Permutation
2. Are the chosen elements a subset of a larger set in which order is not important?
Combination
3. Are there two or more separate events? Fundamental Counting Principle

Copyright 2011 Cengage Learning. All Rights Reserved. May not be copied, scanned, or duplicated, in whole or in part. Due to electronic rights, some third party content may be suppressed from the eBook and/or eChapter(s).
Editorial review has deemed that any suppressed content does not materially affect the overall learning experience. Cengage Learning reserves the right to remove additional content at any time if subsequent rights restrictions require it.
Section 8.5 Counting Principles 613

See www.CalcChat.com for worked-out solutions to odd-numbered exercises.


8.5 Exercises For instructions on how to use a graphing utility, see Appendix A.

Vocabulary and Concept Check


In Exercises 1 and 2, fill in the blank.
1. The _______ states that if there are m1 ways for one event to occur and m2 ways
for a second event to occur, then there are m1 ⭈ m2 ways for both events to occur.
n!
2. The number of _______ of n objects is given by .
n1! ⭈ n2! ⭈ n3! ⭈ . . . ⭈ nk!

3. Is the ordering of n elements called a permutation or a combination of the elements?


n!
4. What do n and r represent in the formula nPr ⫽ ?
共n ⫺ r兲!

Procedures and Problem Solving


Selecting Numbers at Random In Exercises 5–12, 18. Fundamental Counting Principle How many four-
determine the number of ways in which a computer can digit numbers can be formed under each condition?
randomly generate one or more such integers, or pairs of (a) The leading digit cannot be 0.
integers, from 1 through 15.
(b) The leading digit cannot be 0 and the number must
5. An odd integer 6. An even integer be less than 5000.
7. A prime integer (c) The leading digit cannot be 0 and the number must
8. An integer that is greater than 6 be even.
9. A pair of integers whose sum is 20 19. Telecommunication In 2010, the state of Nevada
had two area codes. Using the information about
10. An integer that is divisible by 4
telephone numbers given in Example 4, how many
11. A pair of distinct integers whose sum is 20 telephone numbers could Nevada’s phone system have
12. An integer that is divisible by 3 accommodated?
20. Fundamental Counting Principle In Pennsylvania,
13. Audio Engineering A customer can choose one of four
each standard automobile license plate number consists of
amplifiers, one of six compact disc players, and one of
three letters followed by a four-digit number. How many
five speaker models for an entertainment system.
distinct Pennsylvania license plate numbers can be formed?
Determine the number of possible system configurations.
21. Radio Broadcasting Typically radio stations are
14. Education A college student is preparing a course
identified by four “call letters.” Radio stations east of
schedule for the next semester. The student must select
the Mississippi River have call letters that start with the
one of two mathematics courses, one of three science
letter W and radio stations west of the Mississippi River
courses, and one of five courses from the social sciences
have call letters that start with the letter K.
and humanities. How many schedules are possible?
(a) Find the number of different sets of radio station
15. Education In how many ways can a 10-question
call letters that are possible in the United States.
true-false exam be answered? (Assume that no questions
are omitted.) (b) Find the number of different sets of radio station
call letters that are possible when the call letters
16. Physiology In a physiology class, a student must
must include a Q.
dissect three different specimens. The student can select
one of nine earthworms, one of four frogs, and one of 22. Banking ATM personal identification number (PIN)
seven fetal pigs. In how many ways can the student codes typically consist of four-digit sequences of
select the specimens? numbers.
17. Fundamental Counting Principle How many three- (a) Find the total number of ATM codes possible.
digit numbers can be formed under each condition? (b) Find the total number of ATM codes possible when
(a) The leading digit cannot be 0. the first digit is not 0.
(b) The leading digit cannot be 0 and no repetition of 23. Operations Research In 1963, the United States
digits is allowed. Postal Service launched the Zoning Improvement Plan
(ZIP) Code to streamline the mail-delivery system. A
(c) The leading digit cannot be 0 and the number must
ZIP code consists of a five-digit sequence of numbers.
be a multple of 5.

Copyright 2011 Cengage Learning. All Rights Reserved. May not be copied, scanned, or duplicated, in whole or in part. Due to electronic rights, some third party content may be suppressed from the eBook and/or eChapter(s).
Editorial review has deemed that any suppressed content does not materially affect the overall learning experience. Cengage Learning reserves the right to remove additional content at any time if subsequent rights restrictions require it.
614 Chapter 8 Sequences, Series, and Probability

(a) Find the number of ZIP codes consisting of five 42. Physical Education How many different batting
digits. orders can a baseball coach create from a team of
(b) Find the number of ZIP codes consisting of five 15 players when there are nine positions to fill?
digits when the first digit is 1 or 2. 43. Combinatorics A school locker has a dial lock on
24. Operations Research In 1983, in order to identify which there are 37 numbers from 0 to 36. Find the total
small geographic segments, such as city blocks or a number of possible lock combinations when the lock
single building, within a delivery code, the post office requires a three-digit sequence of left-right-left and the
began to use an expanded ZIP code called ZIP⫹4, numbers can be repeated.
which is composed of the original five-digit code plus a 44. Athletics Eight sprinters have qualified for the finals
four-digit add-on code. in the 100-meter dash at the NCAA national track meet.
(a) Find the number of ZIP codes consisting of five How many different orders of the top three finishes are
digits followed by the four additional digits. possible? (Assume there are no ties.)
(b) Find the number of ZIP codes consisting of five
Finding Permutations In Exercises 45 and 46, use the
digits followed by the four additional digits when
letters A, B, C, and D.
the first number of the five-digit code is 1 or 2.
25. Finding Permutations Three couples have reserved 45. Write all permutations of the letters.
seats in a row for a concert. In how many different 46. Write all permutations of the letters when the letters B
ways can they be seated when (a) there are no seating and C must remain between the letters A and D.
restrictions? (b) each couple sits together?
26. Finding Permutations In how many orders can five Finding Distinguishable Permutations In Exercises
girls and three boys walk through a doorway single file 47–50, find the number of distinguishable permutations
when (a) there are no restrictions? (b) the girls walk of the group of letters.
through before the boys? 47. A, A, G, E, E, E, M 48. B, B, B, T, T, T, T, T
49. A, L, G, E, B, R, A 50. M, I, S, S, I, S, S, I, P, P, I
Finding a Permutation In Exercises 27–32, evaluate n Pr
using the formula from this section.
Finding a Combination In Exercises 51–56, evaluate
27. 4P4 28. 5 P5 n Cr using the formula from this section.
29. 8 P3 30. 20 P2 51. 5C2 52. 6C3
31. 5 P4 32. 7P4 53. 4C1 54. 5C1
55. 25C0 56. 20C0
Finding a Permutation Using a Graphing Utility In
Exercises 33–36, evaluate n Pr using a graphing utility.
Finding a Combination Using a Graphing Utility In
33. 20 P6 34. 10 P8 Exercises 57–60, evaluate n Cr using a graphing utility.
35. 120 P4 36. 100 P3 57. 20C5 58. 10C7

37. Photography In how many ways can five children 59. 42C5 60. 50C6
posing for a photograph line up in a row?
Finding a Combination In Exercises 61 and 62, use the
38. Transportation Design In how many ways can six
letters A, B, C, D, E, and F.
people sit in a six-passenger minivan?
39. Politics The nine justices of the U.S. Supreme Court 61. Write all possible selections of two letters that can be
pose for a photograph while standing in a straight line, formed from the letters. (The order of the two letters is
as opposed to the typical pose of two rows. How many not important.)
different orders of the justices are possible for this 62. Write all possible selections of three letters that can be
photograph? formed from the letters. (The order of the three letters is
40. Manufacturing Four processes are involved in not important.)
assembling a product, and they can be performed in any
63. Politics As of March 2010, the U.S. Senate Committee
order. The management wants to test each order to
on Indian Affairs had 15 members. Party affiliation is
determine which is the least time-consuming. How
not a factor in selection. How many different committees
many different orders will have to be tested?
are possible from the 100 U.S. senators?
41. Politics From a pool of 12 candidates, the offices of
64. Education You can answer any 12 questions from a
president, vice-president, secretary, and treasurer will
total of 14 questions on an exam. In how many different
be filled. In how many ways can the offices be filled?
ways can you select the questions?

Copyright 2011 Cengage Learning. All Rights Reserved. May not be copied, scanned, or duplicated, in whole or in part. Due to electronic rights, some third party content may be suppressed from the eBook and/or eChapter(s).
Editorial review has deemed that any suppressed content does not materially affect the overall learning experience. Cengage Learning reserves the right to remove additional content at any time if subsequent rights restrictions require it.
Section 8.5 Counting Principles 615

65. (p. 607) Powerball is 81. n⫹1P3 ⫽ 4 ⭈ nP2 82. n⫹2P3 ⫽ 6 ⭈ n⫹2P1
played with 59 white balls, numbered 83. 4 ⭈ n⫹1P2 ⫽ n⫹2P3 84. 5 ⭈ n⫺1P1 ⫽ nP2
1 through 59, and 39 red balls, numbered
1 through 39. Five white balls and one Conclusions
red ball, the Powerball, are drawn. In how
many ways can a player select the True or False? In Exercises 85 and 86, determine whether
six numbers? the statement is true or false. Justify your answer.
85. The number of pairs of letters that can be formed from
66. Geometry Three points that are not collinear any of the first 13 letters in the alphabet (A–M), where
determine three lines. How many lines are determined repetitions are allowed, is an example of a permutation.
by nine points, no three of which are collinear? 86. The number of permutations of n elements can be
67. Quality Engineering A shipment of 30 flat screen derived by using the Fundamental Counting Principle.
televisions contains three defective units. In how many
ways can a vending company purchase four of these 87. Think About It Can your calculator evaluate 100 P80? If
units and receive (a) all good units, (b) two good units, not, explain why.
and (c) at least two good units?
68. Game Theory You are dealt five cards from an 88. C A P S T O N E Decide whether each scenario should
ordinary deck of 52 playing cards. In how many ways be counted using permutations or combinations.
can you get a full house? (A full house consists of three Explain your reasoning. (Do not calculate.)
of one kind and two of another. For example, 8-8-8-5-5 (a) Number of ways 10 people can line up in a row for
and K-K-K-10-10 are full houses.) concert tickets
69. Game Theory Five cards are chosen from a standard (b) Number of different arrangements of three types
deck of 52 cards. How many five-card combinations of flowers from an array of 20 types
contain two jacks and three aces? (c) Number of four-digit pin numbers for a debit card
70. Law A law office interviews paralegals for 10 openings. (d) Number of two-scoop ice cream sundaes created
There are 13 paralegals with two years of experience from 31 different flavors
and 20 paralegals with one year of experience. How
many combinations of seven paralegals with two years
of experience and three paralegals with one year of 89. Writing Explain in your own words the meaning of
experience are possible? n Pr .

71. Administration A six-member research committee 90. Without calculating the numbers, determine which of
is to be formed having one administrator, three the following is greater. Explain.
faculty members, and two students. There are seven (a) The number of combinations of 10 elements taken
administrators, 12 faculty members, and 25 students in six at a time
contention for the committee. How many six-member (b) The number of permutations of 10 elements taken
committees are possible? six at a time
72. Psychology The number of possible interpersonal
relationships increases dramatically as the size of a Proof In Exercises 91–94, prove the identity.
group increases. Determine the numbers of different
two-person relationships that are possible in groups of 91. n Pn ⫺1 ⫽ n Pn 92. n Cn ⫽ n C0
people of sizes (a) 3, (b) 8, (c) 12, and (d) 20. n Pr
93. n Cn ⫺1 ⫽ n C1 94. n Cr ⫽
r!
Geometry In Exercises 73–76, find the number of Cumulative Mixed Review
diagonals of the polygon. (A line segment connecting
any two nonadjacent vertices is called a diagonal of a Solving an Equation In Exercises 95 and 96, solve the
polygon.) equation. Round your answer to three decimal places, if
necessary.
73. Pentagon 74. Hexagon
95. log2共x ⫺ 3兲 ⫽ 5 96. e x /3 ⫽ 16
75. Octagon 76. Nonagon
Using Cramer’s Rule In Exercises 97 and 98, use
Solving an Equation In Exercises 77–84, solve for n.
Cramer’s Rule to solve the system of equations.
77. 14 ⭈ n P3 ⫽ n⫹2 P4 78. n P5 ⫽ 18 ⭈n ⫺2 P4 97. ⫺5x ⫹ 3y ⫽ ⫺14 98. 8x ⫹ y ⫽ 35
79. nP4 ⫽ 10 ⭈ n⫺1P3 80. nP6 ⫽ 12 ⭈ n⫺1P5 冦
7x ⫺ 2y ⫽ 2 冦
6x ⫹ 2y ⫽ 10
kaarsten 2010/used under license from Shutterstock.com

Copyright 2011 Cengage Learning. All Rights Reserved. May not be copied, scanned, or duplicated, in whole or in part. Due to electronic rights, some third party content may be suppressed from the eBook and/or eChapter(s).
Editorial review has deemed that any suppressed content does not materially affect the overall learning experience. Cengage Learning reserves the right to remove additional content at any time if subsequent rights restrictions require it.
616 Chapter 8 Sequences, Series, and Probability

8.6 Probability

The Probability of an Event What you should learn


● Find probabilities of events.
Any happening whose result is uncertain is called an experiment. The possible results
● Find probabilities of mutually
of the experiment are outcomes, the set of all possible outcomes of the experiment is
exclusive events.
the sample space of the experiment, and any subcollection of a sample space is an
● Find probabilities of independent
event.
events.
For instance, when a six-sided die is tossed, the sample space can be represented
by the numbers 1 through 6. For the experiment to be fair, each of the outcomes is Why you should learn it
equally likely. You can use probability to solve a
To describe a sample space in such a way that each outcome is equally variety of problems that occur in real
likely, you must sometimes distinguish between or among various outcomes in ways life. For instance, in Exercise 54 on
that appear artificial. Example 1 illustrates such a situation. page 623, you are asked to use
probability to help analyze the age
Example 1 Finding the Sample Space distribution of unemployed workers.

Find the sample space for each of the following.

a. One coin is tossed.


b. Two coins are tossed.
c. Three coins are tossed.

Solution
a. Because the coin will land either heads up 共denoted by H 兲
or tails up 共denoted by T 兲, the sample space S is
S ⫽ 再H, T 冎.
b. Because either coin can land heads up or tails up, the possible
outcomes are as follows.

HH ⫽ heads up on both coins


HT ⫽ heads up on first coin and tails up on second coin
TH ⫽ tails up on first coin and heads up on second coin
T T ⫽ tails up on both coins
So, the sample space is
S ⫽ 再HH, HT, TH, TT 冎.
Note that this list distinguishes between the two cases
HT and TH
even though these two outcomes appear to be similar.
c. Following the notation in part (b), the sample space is
S ⫽ 再HHH, HHT, HTH, HTT, TH H, THT, TTH, TTT 冎.
Note that this list distinguishes among the cases
HHT, HTH, and THH
and among the cases
HTT, THT, and TTH.
Now try Exercise 9.
Kurhan 2010/used under license from Shutterstock.com

Copyright 2011 Cengage Learning. All Rights Reserved. May not be copied, scanned, or duplicated, in whole or in part. Due to electronic rights, some third party content may be suppressed from the eBook and/or eChapter(s).
Editorial review has deemed that any suppressed content does not materially affect the overall learning experience. Cengage Learning reserves the right to remove additional content at any time if subsequent rights restrictions require it.
Section 8.6 Probability 617

To calculate the probability of an event, count the number of outcomes in the event
and in the sample space. The number of equally likely outcomes in event E is denoted
by n共E 兲, and the number of equally likely outcomes in the sample space S is denoted by Explore the Concept
n共S 兲. The probability that event E will occur is given by n共E 兲兾n共S 兲. Toss two coins 40 times
and write down the
The Probability of an Event number of heads that
occur on each toss (0, 1, or 2).
If an event E has n共E 兲 equally likely outcomes and its sample space S has n共S 兲 How many times did two heads
equally likely outcomes, then the probability of event E is given by occur? Without performing the
n共E 兲 experiment, how many times
P共E 兲 ⫽ .
n共S 兲 would you expect two heads to
occur when two coins are
tossed 1000 times?
Because the number of outcomes in an
event must be less than or equal to the number Increasing likelihood
of outcomes in the sample space, the probability of occurrence
of an event must be a number from 0 to 1, 0.0 0.5 1.0
inclusive. That is,
0 ≤ P共E 兲 ≤ 1 Impossible The occurrence Certain
event of the event is event
as indicated in Figure 8.13. If P共E 兲 ⫽ 0, then (cannot just as likely as (must
event E cannot occur, and E is called an occur) it is unlikely. occur)
impossible event. If P共E 兲 ⫽ 1, then event E
must occur, and E is called a certain event. Figure 8.13

Example 2 Finding the Probability of an Event

a. Two coins are tossed. What is the probability that both land heads up?
b. A card is drawn at random from a standard deck of playing cards. What is the
probability that it is an ace?

Solution
a. Following the procedure in Example 1(b), let

E ⫽ 再HH 冎
and
S ⫽ 再HH, HT, TH, TT 冎.
The probability of getting two heads is
n共E 兲 1
P共E 兲 ⫽ ⫽ .
n共S 兲 4
b. Because there are 52 cards in a standard deck of playing cards and there are four
aces (one of each suit), the probability of drawing an ace is
n共E 兲 4 1
P共E 兲 ⫽ ⫽ ⫽ .
n共S 兲 52 13
Now try Exercise 13.

Note that a probability can be written as a fraction, a decimal, or a percent. For


instance, in Example 2(a), the probability of getting two heads can be written as
1
4, 0.25, or 25%.

Copyright 2011 Cengage Learning. All Rights Reserved. May not be copied, scanned, or duplicated, in whole or in part. Due to electronic rights, some third party content may be suppressed from the eBook and/or eChapter(s).
Editorial review has deemed that any suppressed content does not materially affect the overall learning experience. Cengage Learning reserves the right to remove additional content at any time if subsequent rights restrictions require it.
618 Chapter 8 Sequences, Series, and Probability

You could have written out the sample space in Example 2(b) and simply counted
the outcomes in the desired event. For larger sample spaces, however, using the counting
principles discussed in Section 8.5 should save you time.

Example 3 Finding the Probability of an Event


Twelve-sided dice, as shown in Figure 8.14, can be constructed (in the shape of regular
dodecahedrons) such that each of the numbers from 1 to 6 appears twice on each die.
Show that these dice can be used in any game requiring ordinary six-sided dice without
changing the probabilities of the various outcomes.

Solution
For an ordinary six-sided die, each of the numbers 1, 2, 3, 4, 5, and 6 occurs only once,
so the probability of any particular number coming up is
n共E 兲 1
P共E 兲 ⫽ ⫽ .
n共S 兲 6 Figure 8.14
For a 12-sided die, each number occurs twice, so the probability of any particular
number coming up is
n共E 兲 2 1
P共E 兲 ⫽ ⫽ ⫽ .
n共S 兲 12 6
Now try Exercise 23.

Example 4 Random Selection


The numbers of colleges and universities in various regions of the United States in 2008
are shown in Figure 8.15. One institution is selected at random. What is the probability
that the institution is in one of the three southern regions? (Source: U.S. National
Center for Education Statistics)

Solution
From the figure, the total number of colleges and universities is 4347. Because there are
406 ⫹ 286 ⫹ 754 ⫽ 1446
colleges and universities in the three southern regions, the probability that the institution
is in one of these regions is
n共E兲 1446
P共E兲 ⫽ ⫽ ⬇ 0.333.
n共S兲 4347

Mountain West North


304 Central East North New
453 Central England
Pacific 668 265
581 Middle
Atlantic
630
South
Atlantic
754
West South Central East South Central
406 286

Figure 8.15

Now try Exercise 27.

Copyright 2011 Cengage Learning. All Rights Reserved. May not be copied, scanned, or duplicated, in whole or in part. Due to electronic rights, some third party content may be suppressed from the eBook and/or eChapter(s).
Editorial review has deemed that any suppressed content does not materially affect the overall learning experience. Cengage Learning reserves the right to remove additional content at any time if subsequent rights restrictions require it.
Section 8.6 Probability 619

Mutually Exclusive Events


Two events A and B (from the same sample space) are mutually exclusive when A and
B have no outcomes in common. In the terminology of sets, the intersection of A and B
is the empty set, which implies that
P共A 傽 B 兲 ⫽ 0.
For instance, when two dice are tossed, the event A of rolling a total of 6 and the
event B of rolling a total of 9 are mutually exclusive. To find the probability that one
or the other of two mutually exclusive events will occur, you can add their individual
probabilities.

Probability of the Union of Two Events


If A and B are events in the same sample space, then the probability of A or B
occurring is given by
P共A 傼 B兲 ⫽ P共A兲 ⫹ P共B兲 ⫺ P共A 傽 B兲.
If A and B are mutually exclusive, then
P共A 傼 B兲 ⫽ P共A兲 ⫹ P共B兲.

Example 5 The Probability of a Union


One card is selected at random from a standard deck of 52 playing cards. What is the
probability that the card is either a heart or a face card?

Solution
Because the deck has 13 hearts, the probability of selecting a heart (event A) is
13 Hearts
P共A兲 ⫽ .
52 A♥ 2♥
3♥ 4♥ n(A ∩ B) = 3
Similarly, because the deck has 12 face cards, the 5♥ 6♥
probability of selecting a face card (event B) is 7♥ 8♥
9♥ K♥
12 Q♥ K♣
P共B兲 ⫽ . 10♥ J♥
52 Q♣ K♦
J♣
Because three of the cards are hearts and face Q♦ K♠
cards (see Figure 8.16), it follows that J♦
Q♠
3 Face cards J ♠
P共A 傽 B兲 ⫽ .
52 Figure 8.16
Finally, applying the formula for the probability
of the union of two events, you can conclude that
the probability of selecting a heart or a face card is
P共A 傼 B兲 ⫽ P共A兲 ⫹ P共B兲 ⫺ P共A 傽 B兲
13 12 3
⫽ ⫹ ⫺
52 52 52
22

52
⬇ 0.42.
Now try Exercise 41.

Copyright 2011 Cengage Learning. All Rights Reserved. May not be copied, scanned, or duplicated, in whole or in part. Due to electronic rights, some third party content may be suppressed from the eBook and/or eChapter(s).
Editorial review has deemed that any suppressed content does not materially affect the overall learning experience. Cengage Learning reserves the right to remove additional content at any time if subsequent rights restrictions require it.
620 Chapter 8 Sequences, Series, and Probability

Example 6 Probability of Mutually Exclusive Events


The human resources department of a company has compiled data on the numbers of
employees who have been with the company for various periods of time. The results are
shown in the table.

Years of service Number of employees


0–4 157
5–9 89
10–14 74
15–19 63
20–24 42
25–29 38
30–34 37
35–39 21
40–44 8

An employee is chosen at random. What is the probability that the employee has
a. 4 or fewer years of service?
b. 9 or fewer years of service?

Solution Human Resources Manager


a. To begin, add the numbers of employees.
157 ⫹ 89 ⫹ 74 ⫹ 63 ⫹ 42 ⫹ 38 ⫹ 37 ⫹ 21 ⫹ 8 ⫽ 529
So, the total number of employees is 529. Next, let event A represent choosing
an employee with 0 to 4 years of service. From the table, you know there are
157 employees with 0 to 4 years of service. The probability of choosing an employee
who has 4 or fewer years of service is
157
P共A兲 ⫽ ⬇ 0.30.
529
b. Let event B represent choosing an employee with 5 to 9 years of service. From the
table, you know that there are 89 employees with 5 to 9 years of service. Then
89
P共B兲 ⫽ .
529
Because event A from part (a) and event B have no outcomes in common, you can
conclude that these two events are mutually exclusive and that
P共A 傼 B兲 ⫽ P共A兲 ⫹ P共B兲
157 89
⫽ ⫹
529 529
246

529
⬇ 0.47.
So, the probability of choosing an employee who has 9 or fewer years of service is
about 0.47.
Now try Exercise 47.
jo unruh/iStockphoto.com

Copyright 2011 Cengage Learning. All Rights Reserved. May not be copied, scanned, or duplicated, in whole or in part. Due to electronic rights, some third party content may be suppressed from the eBook and/or eChapter(s).
Editorial review has deemed that any suppressed content does not materially affect the overall learning experience. Cengage Learning reserves the right to remove additional content at any time if subsequent rights restrictions require it.
Section 8.6 Probability 621

Independent Events
Two events are independent when the occurrence of one has no effect on the
occurrence of the other. For instance, rolling a total of 12 with two six-sided dice has
no effect on the outcome of future rolls of the dice. (See Figure 8.17.) To find the
probability that two independent events will occur, multiply the probabilities of each.

Figure 8.17

Probability of Independent Events


If A and B are independent events, then the probability that both A and B will
occur is given by
P共A and B兲 ⫽ P共A兲 ⭈ P共B兲.

Example 7 Probability of Independent Events


A random number generator on a computer selects three integers from 1 to 20. What is
the probability that all three numbers are less than or equal to 5?

Solution
The probability of selecting a number from 1 to 5 is
5 1
P共A兲 ⫽ ⫽ .
20 4
So, the probability that all three numbers are less than or equal to 5 is

冢4冣冢4冣冢4冣 ⫽ 64.
1 1 1 1
P共A兲 ⭈ P共A兲 ⭈ P共A兲 ⫽

Now try Exercise 49.

Example 8 Probability of Independent Events


In 2009, approximately 13% of the adult population of the United States got most of
their news from the Internet. In a survey, 10 people were chosen at random from the
adult population. What is the probability that all 10 got most of their news from the
Internet? (Source: CBS News/New York Times Poll)

Solution
Let A represent choosing an adult who gets most of his or her news from the Internet.
The probability of choosing an adult who got most of his or her news from the Internet
is 0.13, the probability of choosing a second adult who got most of his or her news from
the Internet is 0.13, and so on. Because these events are independent, you can conclude
that the probability that all 10 people got most of their news from the Internet is
关P共A兲兴10 ⫽ 共0.13兲10 ⬇ 0.000000001.
Now try Exercise 61.

Copyright 2011 Cengage Learning. All Rights Reserved. May not be copied, scanned, or duplicated, in whole or in part. Due to electronic rights, some third party content may be suppressed from the eBook and/or eChapter(s).
Editorial review has deemed that any suppressed content does not materially affect the overall learning experience. Cengage Learning reserves the right to remove additional content at any time if subsequent rights restrictions require it.
622 Chapter 8 Sequences, Series, and Probability

See www.CalcChat.com for worked-out solutions to odd-numbered exercises.


8.6 Exercises For instructions on how to use a graphing utility, see Appendix A.

Vocabulary and Concept Check


In Exercises 1–4, fill in the blank(s).
1. The set of all possible outcomes of an experiment is called the _______ .
n共E兲
2. To determine the probability of an event, use the formula P共E兲 ⫽ , where
n共S兲
n共E兲 is _______ and n共S兲 is _______ .
3. If two events from the same sample space have no outcomes in common, then the
two events are _______ .
4. If the occurrence of one event has no effect on the occurrence of a second event,
then the events are _______ .

5. Write an inequality that represents the possible values of the probability P共E兲 of an event.
6. What is the probability of an impossible event?
7. What is the probability of a certain event?
8. Match the probability formula with the correct probability name.
(a) Probability of the union of two events (i) P共A 傼 B兲 ⫽ P共A兲 ⫹ P共B兲
(b) Probability of mutually exclusive events (ii) P共A 傼 B兲 ⫽ P共A兲 ⫹ P共B兲 ⫺ P共A 傽 B兲
(c) Probability of independent events (iii) P共A and B兲 ⫽ P共A兲 ⭈ P共B兲

Procedures and Problem Solving


Finding the Sample Space In Exercises 9–12, determine 19. The card is a face card in the suit of spades.
the sample space for the experiment. 20. The card is a numbered card (2–10).
9. A coin and a six-sided die are tossed.
Finding the Probability of an Event In Exercises 21–24,
10. A six-sided die is tossed twice and the results of roll 1
use the sample space from Exercise 10 to find the
and roll 2 are recorded.
probability for the experiment of tossing a six-sided die
11. A taste tester has to rank three varieties of orange juice, twice.
A, B, and C, according to preference.
21. The sum is 6. 22. The sum is at least 8.
12. Two marbles are selected at random (without replacement)
from a sack containing two red marbles, two blue 23. The sum is less than 11.
marbles, and one yellow marble. The color of each 24. The sum is odd or prime.
marble is recorded.
Random Selection In Exercises 25–28, one of a team’s
Finding the Probability of an Event In Exercises 13–16, 2200 season ticket holders is selected at random to win a
find the probability for the experiment of tossing a coin prize. The circle graph shows the ages of the season ticket
three times. Use the sample space holders. Find the probability of the event.
S ⴝ {HHH, HHT, HTH, HTT, THH, THT, TTH, TTT}. Ages of Season Ticket Holders

13. The probability of getting exactly two tails 60 and 18 or younger


14. The probability of getting a head on the first toss older 66
506 19–29
15. The probability of getting at least one head
264
16. The probability of getting at least two heads 40–59
836 30–39
528
Finding the Probability of an Event In Exercises 17–20,
find the probability for the experiment of selecting one
card at random from a standard deck of 52 playing cards. 25. The winner is younger than 19 years old.
17. The card is a face card. 26. The winner is older than 39 years old.
18. The card is a black card.

Copyright 2011 Cengage Learning. All Rights Reserved. May not be copied, scanned, or duplicated, in whole or in part. Due to electronic rights, some third party content may be suppressed from the eBook and/or eChapter(s).
Editorial review has deemed that any suppressed content does not materially affect the overall learning experience. Cengage Learning reserves the right to remove additional content at any time if subsequent rights restrictions require it.
Section 8.6 Probability 623

27. The winner is 19 to 39 years old. A student from the class is randomly chosen for a project.
28. The winner is younger than 19 years old or older than Find the probability that the student is the given age.
59 years old. 45. 20 or 21 years old
46. 18 to 21 years old
Using Combinations In Exercises 29–32, find the
probability for the experiment of drawing two marbles 47. Older than 21 years old
at random (without replacement) from a bag containing 48. Younger than 31 years old
one green, two yellow, and three red marbles. (Hint: Use
combinations to find the numbers of outcomes for the Probability of Independent Events In Exercises 49–52,
given event and sample space.) a random number generator selects three numbers from
1 through 10. Find the probability of the event.
29. Both marbles are red.
30. Both marbles are yellow. 49. All three numbers are even.
31. Neither marble is yellow. 50. All three numbers are less than or equal to 4.
32. The marbles are of different colors. 51. Two numbers are less than 5 and the other number is 10.
52. One number is 2, 4, or 6, and the other two numbers are
Finding the Probability of a Complement The complement odd.
of an event A is the collection of all outcomes in the
sample space that are not in A. If the probability of A is 53. Political Science Taylor, Moore, and Perez are
P冇A冈, then the probability of the complement A⬘ is given candidates for public office. It is estimated that Moore
by P冇A⬘ 冈 ⫽ 1 ⫺ P冇A冈. In Exercises 33–36, you are and Perez have about the same probability of winning,
given the probability that an event will happen. Find the and Taylor is believed to be twice as likely to win as
probability that the event will not happen. either of the others. Find the probability of each
candidate’s winning the election.
33. P共E兲 ⫽ 0.75 34. P共E兲 ⫽ 0.2
54. (p. 616) In 2008, there were
35. P共E兲 ⫽ 3 36. P共E兲 ⫽ 8
2 7
approximately 8.92 million unemployed
workers in the United States. The circle
Using the Probability of a Complement In Exercises graph shows the age profile of these
37–40, you are given the probability that an event will not unemployed workers. (Source: U.S. Bureau
happen. Find the probability that the event will happen. of Labor Statistics)
37. P共E⬘ 兲 ⫽ 0.12 38. P共E⬘ 兲 ⫽ 0.84 Ages of Unemployed Workers
39. P共E⬘ 兲 ⫽ 20 40. P共E⬘ 兲 ⫽ 100
13 61

20–24
The Probability of a Union In Exercises 41–44, one card 17% 16–19
is selected at random from a standard deck of 52 playing
14%
cards. Use a formula to find the probability of the union 25–44 65 and older
of the two events. 40% 3%
45–64
41. The card is a club or a king. 26%
42. The card is a face card or a black card.
43. The card is a face card or a 2.
(a) Estimate the number of unemployed workers in the
44. The card is a heart or a spade. 16–19 age group.
(b) What is the probability that a person selected at
Probability of Mutually Exclusive Events In Exercises
random from the population of unemployed workers
45–48, use the table, which shows the age groups of
is in the 25–44 age group?
students in a college sociology class.
(c) What is the probability that a person selected at
random from the population of unemployed workers
Age Number of students is 25 to 64 years old?
18–19 11 (d) What is the probability that a person selected at
random from the population of unemployed workers
20–21 18
is 45 or older?
22–30 2
Kurhan 2010/used under license from Shutterstock.com
31–40 1

Copyright 2011 Cengage Learning. All Rights Reserved. May not be copied, scanned, or duplicated, in whole or in part. Due to electronic rights, some third party content may be suppressed from the eBook and/or eChapter(s).
Editorial review has deemed that any suppressed content does not materially affect the overall learning experience. Cengage Learning reserves the right to remove additional content at any time if subsequent rights restrictions require it.
624 Chapter 8 Sequences, Series, and Probability

55. Education The levels of educational attainment of the 58. Using Counting Principles The deck for a card game
United States population age 25 years or older in 2008 is made up of 108 cards. Twenty-five each are red,
are shown in the circle graph. Use the fact that the yellow, blue, and green, and eight are wild cards. Each
population of people 25 years or older was 196.31 million player is randomly dealt a seven-card hand. What is the
in 2008. (Source: U.S. Census Bureau) probability that a hand will contain (a) exactly two wild
cards, and (b) two wild cards, two red cards, and three
Educational Attainment
blue cards?
High school 59. Using Counting Principles A shipment of 12
graduate 31.2% Not a high school microwave ovens contains three defective units. A
graduate 13.4% vending company has ordered four of these units, and
because all are packaged identically, the selection will
Some college but Advanced be random. What is the probability that (a) all four units
no degree 17.2% degree 10.3% are good, (b) exactly two units are good, and (c) at least
Associateís Bachelor’s two units are good?
degree 8.8% degree 19.1% 60. Using Counting Principles Two integers from 1
(a) Estimate the number of people 25 years old or older through 40 are chosen by a random number generator.
who have advanced degrees. What is the probability that (a) the numbers are both
even, (b) one number is even and one is odd, (c) both
(b) Find the probability that a person 25 years old or older
numbers are less than 30, and (d) the same number is
selected at random has earned a Bachelor’s degree
chosen twice?
or higher.
(c) Find the probability that a person 25 years old or 61. Marketing Suppose that the methods used by
older selected at random has earned a high school shoppers to pay for merchandise are as shown in the
diploma or gone on to post-secondary education. circle graph. Two shoppers are chosen at random. What
56. Political Science One hundred college students were is the probability that both shoppers paid for their
interviewed to determine their political party affiliations purchases only in cash?
and whether they favored a balanced budget amendment How Shoppers Pay for Merchandise
to the Constitution. The results of the study are listed in
the table, where D represents Democrat and R represents Mostly Half cash,
cash half credit
Republican. 27% 30%

32% Mostly credit 7%


D R Total
Only cash Only credit 4%
Favor 23 32 55
Oppose 25 9 34 62. Aerospace Engineering A space vehicle has an
Unsure 7 4 11 independent backup system for one of its communication
Total 55 45 100 networks. The probability that either system will
function satisfactorily for the duration of a flight is 0.985.
A person is selected at random from the sample. Find What is the probability that during a given flight (a) both
the probability that the person selected is (a) a person systems function satisfactorily, (b) at least one system
who doesn’t favor the amendment, (b) a Republican, functions satisfactorily, and (c) both systems fail?
and (c) a Democrat who favors the amendment. 63. Marketing A sales representative makes sales on
approximately one-fifth of all calls. On a given day, the
In Exercises 57–60, the sample spaces are large and representative contacts six potential clients. What is the
you should use the counting principles discussed in probability that a sale will be made with (a) all six
Section 8.5. contacts, (b) none of the contacts, and (c) at least one
contact?
57. Using Counting Principles On a game show you are
given five digits to arrange in the proper order to form 64. Genetics Assume that the probability of the birth of a
the price of a car. If you are correct, you win the car. child of a particular sex is 50%. In a family with four
What is the probability of winning if you (a) guess the children, what is the probability that (a) all the children
position of each digit and (b) know the first digit and are boys, (b) all the children are the same sex, and
guess the others? (c) there is at least one boy?

Copyright 2011 Cengage Learning. All Rights Reserved. May not be copied, scanned, or duplicated, in whole or in part. Due to electronic rights, some third party content may be suppressed from the eBook and/or eChapter(s).
Editorial review has deemed that any suppressed content does not materially affect the overall learning experience. Cengage Learning reserves the right to remove additional content at any time if subsequent rights restrictions require it.
Section 8.6 Probability 625

65. Estimating ␲ A coin of diameter d is dropped onto a (b) Use the pattern in part (a) to write an expression
paper that contains a grid of squares d units on a side. for the probability that four people 共n ⫽ 4兲 have
(See figure.) distinct birthdays.
(a) Find the probability that the coin covers a vertex of (c) Let Pn be the probability that the n people have
one of the squares on the grid. distinct birthdays. Verify that this probability can be
obtained recursively by
(b) Perform the experiment 100 times and use the
results to approximate ␲. 365 ⫺ 共n ⫺ 1兲
P1 ⫽ 1 and Pn ⫽ Pn⫺1.
365
(d) Explain why Qn ⫽ 1 ⫺ Pn gives the probability
that at least two people in a group of n people have
the same birthday.
(e) Use the results of parts (c) and (d) to complete the
table.

n 10 15 20 23 30 40 50
Pn
Qn
66. Geometry You and a friend agree to meet at your
favorite fast food restaurant between 5:00 and 6:00 P.M. The (f) How many people must be in a group so that the
one who arrives first will wait 15 minutes for the other, after probability of at least two of them having the same
which the first person will leave (see figure). What is the 1
birthday is greater than 2? Explain.
probability that the two of you will actually meet, assuming
70. Writing Write a paragraph describing in your own
that your arrival times are random within the hour?
words the difference between mutually exclusive events
and independent events.
60 You meet
(in minutes past 5:00 P.M.)
Your friend’s arri val time

You meet 71. Think About It Let A and B be two events from the
45
You don’t same sample space such that P共A兲 ⫽ 0.76 and
t

meet P共B兲 ⫽ 0.58.


s

t
fir

s
fir
e
riv

es
riv

(a) Is it possible that A and B are mutually exclusive?


ar

30
ar
u

Explain. Draw a diagram to support your answer.


Yo

nd
rie

(b) Is it possible that A⬘ and B⬘ are mutually exclusive?


rf

15
u
Yo

Explain. Draw a diagram to support your answer.


15 30 45 60
(c) Determine the possible range of P共A 傼 B兲.
Your arrival time
(in minutes past 5:00 P.M.) 72. C A P S T O N E Give an example of each of the
following events for the experiment of selecting one
Conclusions or more cards at random from a standard deck of
52 playing cards.
True or False? In Exercises 67 and 68, determine whether
the statement is true or false. Justify your answer. (a) A union of two mutually exclusive events
(b) A union of two events that are not mutually
67. If the probability of an outcome in a sample space is 1, then exclusive
the probability of the other outcomes in the sample space is 0.
(c) Independent events
68. If A and B are independent events with nonzero
probabilities, then A can occur when B occurs. (d) The complement of an event

69. Exploration Consider a group of n people.


(a) Explain why the following pattern gives the Cumulative Mixed Review
probability that the n people have distinct birthdays. Finding a Combination In Exercises 73–76, evaluate
365 364 365 ⭈ 364 nCr .
Verify your result using a graphing utility.
n ⫽ 2: ⭈ 365 ⫽ 3652
365 73. 6C2 74. 9C5
365 364 363 365 ⭈ 364 ⭈ 363 75. 11C8 76. 16C13
n ⫽ 3:
365
⭈ 365 ⭈ 365 ⫽ 3653

Copyright 2011 Cengage Learning. All Rights Reserved. May not be copied, scanned, or duplicated, in whole or in part. Due to electronic rights, some third party content may be suppressed from the eBook and/or eChapter(s).
Editorial review has deemed that any suppressed content does not materially affect the overall learning experience. Cengage Learning reserves the right to remove additional content at any time if subsequent rights restrictions require it.
626 Chapter 8 Sequences, Series, and Probability

8 Chapter Summary

What did you learn? Explanation and Examples Review


Exercises
Use sequence notation to write the an ⫽ 7n ⫺ 4; a1 ⫽ 7共1兲 ⫺ 4 ⫽ 3, a2 ⫽ 7共2兲 ⫺ 4 ⫽ 10,
1–8
terms of sequences ( p. 570). a3 ⫽ 7共3兲 ⫺ 4 ⫽ 17, a4 ⫽ 7共4兲 ⫺ 4 ⫽ 24

Use factorial notation (p. 573). If n is a positive integer, then n! ⫽ 1 ⭈ 2 ⭈ 3 ⭈ 4 . . . 共n ⫺ 1兲 ⭈ n. 9–12

Use summation notation to write The sum of the first n terms of a sequence is represented by
sums ( p. 574). n
13–20
兺a ⫽a
i⫽1
i 1 ⫹ a2 ⫹ a3 ⫹ a4 ⫹ . . . ⫹ an.

Find sums of infinite series ⬁ 5 5 5 5 5 5


8.1
( p. 575). 兺 10 ⫽ 10
i⫽1
i 1 ⫹ ⫹ ⫹ ⫹
102 103 104 105
⫹ . . .

⫽ 0.5 ⫹ 0.05 ⫹ 0.005 ⫹ 0.0005 ⫹ 0.00005 ⫹ . . .


21–24
⫽ 0.55555 . . .

⫽ 59

Use sequences and series to model A sequence can be used to model the resident population of
and solve real-life problems the United States from 1980 through 2008. (See Example 10.) 25, 26
( p. 576).

Recognize, write, and find the nth an ⫽ 9n ⫹ 5; a1 ⫽ 9共1兲 ⫹ 5 ⫽ 14, a2 ⫽ 9共2兲 ⫹ 5 ⫽ 23,
terms of arithmetic sequences a3 ⫽ 9共3兲 ⫹ 5 ⫽ 32, a4 ⫽ 9共4兲 ⫹ 5 ⫽ 41 27–38
( p. 581).

Find nth partial sums of The sum of a finite arithmetic sequence with n terms is
8.2 arithmetic sequences (p. 584). n 39–42
Sn ⫽ 共a1 ⫹ an 兲.
2

Use arithmetic sequences to model An arithmetic sequence can be used to find the total sales of
and solve real-life problems a small business. (See Example 7.) 43, 44
( p. 585).

Recognize, write, and find the an ⫽ 3共4n兲; a1 ⫽ 3共41兲 ⫽ 12, a2 ⫽ 3共42兲 ⫽ 48,
nth terms of geometric sequences a3 ⫽ 3共43兲 ⫽ 192, a4 ⫽ 3共44兲 ⫽ 768 45–56
( p. 589).

Find nth partial sums of The sum of the finite geometric sequence
geometric sequences (p. 592). a1, a1r, a1r 2, . . . , a1r n⫺1 with common ratio r ⫽ 1 is given
1 ⫺ rn 57–60
冢 冣
n
by Sn ⫽
i⫽1

a1r i⫺1 ⫽ a1
1⫺r
.

8.3 Find sums of infinite geometric ⱍⱍ


If r < 1, then the infinite geometric series
series ( p. 593).
a1 ⫹ a1r ⫹ a1r 2 ⫹ . . . ⫹ a1r n⫺1 ⫹ . . .
61–64
has the sum
⬁ a1
S⫽ 兺ar
i⫽0
1
i

1⫺r
.

Use geometric sequences to A finite geometric sequence can be used to find the balance
model and solve real-life problems in an annuity at the end of 2 years. (See Example 8.) 65, 66
( p. 594).

Copyright 2011 Cengage Learning. All Rights Reserved. May not be copied, scanned, or duplicated, in whole or in part. Due to electronic rights, some third party content may be suppressed from the eBook and/or eChapter(s).
Editorial review has deemed that any suppressed content does not materially affect the overall learning experience. Cengage Learning reserves the right to remove additional content at any time if subsequent rights restrictions require it.
Chapter Summary 627

What did you learn? Explanation and Examples Review


Exercises
Use the Binomial Theorem to The Binomial Theorem: In the expansion of 共x ⫹ y兲n ⫽
calculate binomial coefficients x n ⫹ nx n⫺1y ⫹ . . . ⫹ nCr x n⫺ryr ⫹ . . . ⫹ nxy n⫺1 ⫹ y n,
( p. 599). 67–70
the coefficient of xn⫺ryr is
n!
nCr ⫽ .
共n ⫺ r兲!r!
8.4
Use binomial coefficients to write 共x ⫹ 1兲3 ⫽ x3 ⫹ 3x2 ⫹ 3x ⫹ 1
71–74
binomial expansions (p. 601). 共x ⫺ 1兲4 ⫽ x4 ⫺ 4x3 ⫹ 6x2 ⫺ 4x ⫹ 1

Use Pascal’s Triangle to calculate First several rows 1


1 1
binomial coefficients (p. 603). of Pascal’s triangle: 1 2 1 75–78
1 3 3 1
1 4 6 4 1

Solve simple counting problems A computer randomly generates an integer from 1 through
( p. 607). 15. The computer can generate an integer that is divisible by 79, 80
3 in 5 ways (3, 6, 9, 12, and 15).

Use the Fundamental Counting Fundamental Counting Principle: Let E1 and E2 be two
Principle to solve more complicated events. The first event E1 can occur in m1 different ways.
counting problems ( p. 608). After E1 has occurred, E2 can occur in m2 different ways. 81, 82
The number of ways that the two events can occur is
m1 ⭈ m2.

Use permutations to solve counting A permutation of n different elements is an ordering of the


problems ( p. 609). elements such that one element is first, one is second, one is
third, and so on. The number of permutations of n elements
8.5
is given by
n ⭈ 共n ⫺ 1兲 . . . 4 ⭈ 3 ⭈ 2 ⭈ 1 ⫽ n!.
83–88, 91
In other words, there are n! different ways that n elements can
be ordered.
The number of permutations of n elements taken r at a time is
n!
n Pr ⫽ .
共n ⫺ r兲!

Use combinations to solve counting The number of combinations of n elements taken r at a time is
problems ( p. 612). n! 89, 90, 92
nCr ⫽ .
共n ⫺ r兲!r!

Find probabilities of events If an event E has n共E兲 equally likely outcomes and its sample
( p. 616). space S has n共S兲 equally likely outcomes, then the probability
of event E is 93, 94
n共E 兲
P共E 兲 ⫽ .
n共S 兲

Find probabilities of mutually If A and B are events in the same sample space, then the
8.6 exclusive events ( p. 619). probability of A or B occurring is
P共A 傼 B兲 ⫽ P共A兲 ⫹ 共B兲 ⫺ P共A 傽 B兲. 95
If A and B are mutually exclusive, then
P(A 傼 B) ⫽ P(A) ⫹ P(B).

Find probabilities of independ- If A and B are independent events, then the probability that
96
ent events ( p. 621). both A and B will occur is P共A and B兲 ⫽ P共A兲 ⭈ P共B兲.

Copyright 2011 Cengage Learning. All Rights Reserved. May not be copied, scanned, or duplicated, in whole or in part. Due to electronic rights, some third party content may be suppressed from the eBook and/or eChapter(s).
Editorial review has deemed that any suppressed content does not materially affect the overall learning experience. Cengage Learning reserves the right to remove additional content at any time if subsequent rights restrictions require it.
628 Chapter 8 Sequences, Series, and Probability

See www.CalcChat.com for worked-out solutions to odd-numbered exercises.


8 Review Exercises For instructions on how to use a graphing utility, see Appendix A.

⬁ ⬁
8.1
Writing the Terms of a Sequence In Exercises 1 and 2,
23. 兺 2共0.5兲
k⫽1
k
24. 兺 4共0.25兲
k⫽1
k

write the first five terms of the sequence. (Assume n


25. Compound Interest A deposit of $3000 is made in an
begins with 1.)
account that earns 2% interest compounded quarterly.
2n 共⫺1兲n The balance in the account after n quarters is given by
1. an ⫽ 2. an ⫽
2n ⫹1 n!
an ⫽ 3000 1 ⫹ 冢 0.02 n
4
, 冣 n ⫽ 1, 2, 3, . . . .
Finding the nth Term of a Sequence In Exercises 3–6,
write an expression for the apparent nth term of the (a) Compute the first eight terms of this sequence.
sequence. (Assume n begins with 1.) (b) Find the balance in this account after 10 years by
3. 5, 10, 15, 20, 25, . . . 4. 50, 48, 46, 44, 42, . . . computing the 40th term of the sequence.
2 2 2 2
5. 2, 3, 5, 7, 9, . . .
3 4 5 6 7
6. 2, 3, 4, 5, 6, . . . 26. Education The average salaries an (in thousands of
dollars) of college faculty members in the United States
A Recursive Sequence In Exercises 7 and 8, write the from 2001 through 2009 can be approximated by the
first five terms of the sequence defined recursively. model

7. a1 ⫽ 9, ak⫹1 ⫽ ak ⫺ 4 8. a1 ⫽ 49, ak⫹1 ⫽ ak ⫹ 6 an ⫽ 0.065n2 ⫹ 1.43n ⫹ 58.2, n ⫽ 1, 2, 3, . . . , 9


where n is the year, with n ⫽ 1 corresponding to 2001.
Simplifying Factorial Expressions In Exercises 9–12, (Source: American Association of University
simplify the factorial expression. Professors)
18! 10! (a) Find the terms of this finite sequence for the given
9. 10. values of n.
20! 8!
共n ⫹ 1兲! 2n! (b) Use a graphing utility to graph the sequence for the
11. 12.
共n ⫺ 1兲! 共n ⫹ 1兲! given values of n.
(c) Use the graphing utility to construct a bar graph of
Sigma Notation for Sums In Exercises 13–16, find the the sequence for the given values of n.
sum. (d) Use the sequence to predict the average salaries of
6 5 college faculty members for the years 2010 through
13. 兺5
i⫽1
14. 兺 4k
k⫽2
2016. Do your results seem reasonable? Explain.
4 8 8.2
6 i
15. 兺j
j ⫽1
2 16. 兺 i⫹1
i⫽1 Determining Whether a Sequence Is Arithmetic In
Exercises 27–30, determine whether or not the sequence
Writing a Sum Using Sigma Notation In Exercises 17–20, is arithmetic. If it is, find the common difference.
use sigma notation to write the sum. Then use a graphing
27. 5, 3, 1, ⫺1, ⫺3, . . . 28. 0, 1, 3, 6, 10, . . .
utility to find the sum. 1 3 5 9 8 7 6 5
29. 2, 1, 2, 2, 2, . . . 30. 9, 9, 9, 9, 9, . . .
1 1 1 1
17. ⫹ ⫹ ⫹. . .⫹
2共1兲 2共2兲 2共3兲 2共20兲 Writing the Terms of an Arithmetic Sequence In
18. 2共12兲 ⫹ 2共22兲 ⫹ 2共32兲 ⫹ . . . ⫹ 2共92兲 Exercises 31–34, write the first five terms of the
1 2 3 9 arithmetic sequence. Use the table feature of a graphing
19. ⫹ ⫹ ⫹ . . . ⫹ utility to verify your results.
2 3 4 10
1 1 1 31. a1 ⫽ 3, d ⫽ 4 32. a1 ⫽ 8, d ⫽ ⫺2
20. 1 ⫺ ⫹ ⫺ ⫹. . .
33. a4 ⫽ 10, a10 ⫽ 28 34. a2 ⫽ 14, a6 ⫽ 22
3 9 27

Finding the Sum of a Series In Exercises 21–24, find Writing the Terms of an Arithmetic Sequence In
(a) the fourth partial sum and (b) the sum of the infinite Exercises 35 and 36, write the first five terms of the
series. arithmetic sequence. Find the common difference and
write the nth term of the sequence as a function of n.
⬁ 5 ⬁ 3
21. 兺 10
k⫽1
k 22. 兺2
k⫽1
k 35. a1 ⫽ 35, ak⫹1 ⫽ ak ⫺ 3 36. a1 ⫽ 15, ak⫹1 ⫽ ak ⫹ 52

Copyright 2011 Cengage Learning. All Rights Reserved. May not be copied, scanned, or duplicated, in whole or in part. Due to electronic rights, some third party content may be suppressed from the eBook and/or eChapter(s).
Editorial review has deemed that any suppressed content does not materially affect the overall learning experience. Cengage Learning reserves the right to remove additional content at any time if subsequent rights restrictions require it.
Review Exercises 629

Finding the nth Term of an Arithmetic Sequence In Finding the Sum of a Finite Geometric Sequence In
Exercises 37 and 38, find a formula for an for the Exercises 57–60, find the sum. Use a graphing utility to
arithmetic sequence and find the sum of the first 25 terms verify your result.
of the sequence. 7 5

37. a1 ⫽ 100, d ⫽ ⫺3 38. a1 ⫽ 10, a3 ⫽ 28


57. 兺2
i⫽1
i⫺1
58. 兺3
i⫽1
i⫺1

7 4

兺 共⫺4兲 兺 12共⫺ 兲
n⫺1 1 n⫺1
Finding a Partial Sum of an Arithmetic Sequence In 59. 60. 2
n⫽1 n⫽1
Exercises 39 and 40, find the partial sum. Use a graphing
utility to verify your result.
Finding the Sum of an Infinite Geometric Series In
10 11

兺 共2j ⫺ 3兲 兺 共 k ⫹ 4兲
2 Exercises 61–64, find the sum of the infinite geometric
39. 40. 3
j⫽1 k⫽1 series.
⬁ ⬁
兺 4共 兲 兺 6共 兲
7 i⫺1 1 i⫺1
41. Find the sum of the first 50 positive multiples of 5. 61. 8 62. 3
i⫽1 i⫽1
42. Find the sum of the integers from 10 to 70.
⬁ ⬁
兺 4共 兲 兺 1.3共 兲
2 k⫺1 1 k⫺1
43. Accounting The starting salary for an accountant is 63. 3 64. 10
$36,000 with a guaranteed salary increase of $2250 per k⫽1 k⫽1

year. Determine (a) the salary during the fifth year and 65. Finance A company buys a fleet of six vans for
(b) the total compensation through 5 full years of $130,000. During the next 5 years, the fleet will
employment. depreciate at a rate of 30% per year. (That is, at the end
44. Agriculture In his first trip around a field, a farmer of each year, the depreciated value will be 70% of the
makes 123 bales. In his second trip, he makes 11 fewer value at the beginning of the year.)
bales. Because each trip is shorter than the preceding (a) Find the formula for the nth term of a geometric
trip, the farmer estimates that the same pattern will sequence that gives the value of the fleet t full years
continue. Estimate the total number of bales made if after it was purchased.
there are another five trips around the field.
(b) Find the depreciated value of the fleet at the end of
8.3 5 full years.
Identifying a Geometric Sequence In Exercises 45–48, 66. Finance A deposit of $80 is made at the beginning of
determine whether or not the sequence is geometric. If it each month in an account that pays 4% interest,
is, find the common ratio. compounded monthly. The balance A in the account at
45. 5, 10, 20, 40, . . .
1 2 3 4
46. 2, 3, 4, 5, . . . the end of 4 years is given by
47. 54, ⫺18, 6, ⫺2, . . .
冢 冣 冢 冣
1 2 4 8
48. 3, ⫺ 3, 3. ⫺ 3, . . .
1 48
0.04 0.04
A ⫽ 80 1 ⫹ ⫹ . . . ⫹ 80 1 ⫹ .
12 12
Finding the Terms of a Geometric Sequence In
Find A.
Exercises 49–52, write the first five terms of the
geometric sequence. 8.4

49. a1 ⫽ 4, r ⫽ ⫺ 14 50. a1 ⫽ 2, r ⫽
3
2
Finding Binomial Coefficients In Exercises 67–70, find
the binomial coefficient. Use a graphing utility to verify
51. a1 ⫽ 9, a3 ⫽ 4 52. a1 ⫽ 2, a3 ⫽ 12
your result.
Finding the nth Term of a Geometric Sequence In 67. 10C8 68. 12C5
Exercises 53 and 54, write the first five terms of the 69. 9C4 70. 14C12
geometric sequence. Find the common ratio and write
the nth term of the sequence as a function of n. Expanding a Binomial In Exercises 71–74, use the
53. a1 ⫽ 120, ak⫹1 ⫽ 13ak 54. a1 ⫽ 200, ak⫹1 ⫽ 0.1ak Binomial Theorem to expand and simplify the expression.
71. 共x ⫹ 5兲4 72. 共y ⫺ 3兲3
Finding a Term of a Geometric Sequence In Exercises 55
73. 共a ⫺ 4b兲5 74. 共3x ⫹ y兲7
and 56, find the indicated term of the geometric sequence
(a) algebraically and (b) using the table feature of a graphing
Using Pascal’s Triangle In Exercises 75–78, use Pascal’s
utility.
Triangle to find the binomial coefficient.
55. a1 ⫽ 16, a2 ⫽ ⫺8, 6th term
75. 6C3 76. 9C7
56. a3 ⫽ 6, a4 ⫽ 1, 9th term
77. 8C4 78. 10C5

Copyright 2011 Cengage Learning. All Rights Reserved. May not be copied, scanned, or duplicated, in whole or in part. Due to electronic rights, some third party content may be suppressed from the eBook and/or eChapter(s).
Editorial review has deemed that any suppressed content does not materially affect the overall learning experience. Cengage Learning reserves the right to remove additional content at any time if subsequent rights restrictions require it.
630 Chapter 8 Sequences, Series, and Probability

8.5 Finding a Combination In Exercises 89 and 90, evaluate


79. Probability Slips of paper numbered 1 through 15 are nCr using the formula from Section 8.5. Use a graphing

placed in a hat. In how many ways can two numbers be utility to verify your result.
drawn so that the sum of the numbers is 12? Assume the 89. 8C6 90. 50C48
random selection is without replacement.
80. Aviation Ten people are boarding an aircraft. Two 91. Athletics There are 12 bicyclists entered in a race. In
have tickets for first class and board before those in how many different orders could the 12 bicyclists
economy class. In how many ways can the ten people finish? (Assume there are no ties.)
board the aircraft? 92. Athletics From a pool of seven juniors and eleven
81. Education A college student is preparing a course seniors, four co-captains will be chosen for the football
schedule of four classes for the next semester. The student team. How many different combinations are possible if
can choose from the open sections shown in the table. two juniors and two seniors are to be chosen?
8.6
Course Sections 93. Fashion A man has five pairs of socks (no two pairs
Math 100 001–004 are the same color). He randomly selects two socks
Economics 110 001–003 from a drawer. What is the probability that he gets a
matched pair?
English 105 001–006
Humanities 101 001–003 94. Library Science A child returns a five-volume set of
books to a bookshelf. The child is not able to read, and
(a) Find the number of possible schedules that the so cannot distinguish one volume from another. What is
student can create from the offerings. the probability that the books are shelved in the
(b) Find the number of possible schedules that the correct order?
student can create from the offerings when two of 95. Education A sample of college students, faculty
the Math 100 sections are closed. members, and administrators were asked whether they
(c) Find the number of possible schedules that the favored a proposed increase in the annual activity fee to
student can create from the offerings when two of enhance student life on campus. The results of the study
the Math 100 sections and four of the English 105 are shown in the table.
sections are closed.
82. Marketing A telemarketing firm is making calls to Favor Oppose Total
prospective customers by randomly dialing seven-digit
Students 237 163 400
phone numbers within an area code.
Faculty 37 38 75
(a) Find the number of possible calls that the telemarketer
can make. Admin. 18 7 25
(b) The telemarketing firm is calling only within Total 292 208 500
exchanges that begin with a “7” or a “6.” How many
different calls are possible? A person is selected at random from the sample. Find
each probability.
(c) The telemarketing firm is calling only within
exchanges that do not begin with a “0” or a “1.” (a) The person is not in favor of the proposal.
How many different calls are possible? (b) The person is a student.
(c) The person is a faculty member and is in favor of
Finding a Permutation In Exercises 83 and 84, evaluate the proposal.
nPr using the formula from Section 8.5. Use a graphing 96. Probability A six-sided die is rolled six times. What
utility to verify your result. is the probability that each side appears exactly once?
83. 12P10 84. 6P4
Conclusions
Finding Distinguishable Permutations In Exercises 85 True or False? In Exercises 97 and 98, determine
and 86, find the number of distinguishable permutations whether the statement is true or false. Justify your answer.
of the group of letters.
共n ⫹ 2兲! 8 8
85. C, A, L, C, U, L, U, S 86. I, N, T, E, G, R, A, T, E 97.
n!
⫽ 共n ⫹ 2兲共n ⫹ 1兲 98. 兺 3k ⫽ 3 兺 k
k⫽1 k⫽1

Solving an Equation In Exercises 87 and 88, solve for n. 99. Writing In your own words, explain what makes a
87. n⫹1P2 ⫽ 4 ⭈ n P1 88. 8 ⭈ nP2 ⫽ n⫹1P3 sequence (a) arithmetic and (b) geometric.

Copyright 2011 Cengage Learning. All Rights Reserved. May not be copied, scanned, or duplicated, in whole or in part. Due to electronic rights, some third party content may be suppressed from the eBook and/or eChapter(s).
Editorial review has deemed that any suppressed content does not materially affect the overall learning experience. Cengage Learning reserves the right to remove additional content at any time if subsequent rights restrictions require it.
Chapter Test 631

See www.CalcChat.com for worked-out solutions to odd-numbered exercises.


8 Chapter Test For instructions on how to use a graphing utility, see Appendix A.

Take this test as you would take a test in class. After you are finished, check your
work against the answers in the back of the book.

In Exercises 1– 4, write the first five terms of the sequence.


1. an ⫽ 共⫺ 3 兲
2 n ⫺1
(Begin with n ⫽ 1.) 2. a1 ⫽ 12 and ak⫹1 ⫽ ak ⫹ 4
共⫺1兲 x n n
3. bn ⫽ (Begin with n ⫽ 1.)
n
共⫺1兲2n⫹1x2n⫹1
4. bn ⫽ (Begin with n ⫽ 1.)
共2n ⫹ 1兲!
11! ⭈ 4! n! 2n!
5. Simplify . 6. Simplify . 7. Simplify .
4! ⭈ 7! 共n ⫹ 1兲! 共n ⫺ 1兲!
8. Write an expression for the apparent nth term of the sequence
2, 5, 10, 17, 26, . . . . (Assume n begins with 1.)

In Exercises 9 and 10, find a formula for the nth term of the sequence.
1
9. Arithmetic: a1 ⫽ 5000, d ⫽ ⫺100 10. Geometric: a1 ⫽ 4, ak⫹1 ⫽ 2ak
2 2 2
11. Use sigma notation to write ⫹ ⫹. . .⫹ .
3共1兲 ⫹ 1 3共2兲 ⫹ 1 3共12兲 ⫹ 1
1 1 1 1
12. Use sigma notation to write 2 ⫹ 2 ⫹ 8 ⫹ 32 ⫹ 128 ⫹ . . ..

In Exercises 13–15, find the sum.


7 8

兺 共8n ⫺ 5兲 兺 24共 兲
1 n⫺1
13. 14. 6
n⫽1 n⫽1

15. 5 ⫺ 2 ⫹ 5 ⫺ 25 ⫹ 125 ⫺ . . .
4 8 16

16. Use the Binomial Theorem to expand and simplify 共2a ⫺ 5b兲4.

In Exercises 17–20, evaluate the expression.


17. 9C3 18. 20C3 19. 9 P2 20. 70 P3

21. Solve for n in 4 ⭈ nP3 ⫽ n⫹1P4.


22. How many distinct license plates can be issued consisting of one letter followed by
a three-digit number?
23. Four students are randomly selected from a class of 25 to answer questions from a
reading assignment. In how many ways can the four be selected?
24. A card is drawn from a standard deck of 52 playing cards. Find the probability that
it is a red face card.
25. In 2010, five of the eleven men’s basketball teams in the Big Ten Conference
participated in the NCAA Men’s Basketball Championship Tournament. What is
the probability that a random selection of five teams from the Big Ten Conference
will be the same as the five teams that participated in the tournament?
26. Two integers from 1 to 60 are chosen by a random number generator. What is
the probability that (a) both numbers are odd, (b) both numbers are less than 12,
and (c) the same number is chosen twice?
27. A weather forecast indicates that the probability of snow is 75%. What is the
probability that it will not snow?

Copyright 2011 Cengage Learning. All Rights Reserved. May not be copied, scanned, or duplicated, in whole or in part. Due to electronic rights, some third party content may be suppressed from the eBook and/or eChapter(s).
Editorial review has deemed that any suppressed content does not materially affect the overall learning experience. Cengage Learning reserves the right to remove additional content at any time if subsequent rights restrictions require it.
632 Chapter 8 Sequences, Series, and Probability

Proofs in Mathematics

Properties of Sums (p. 574)


n
1. 兺 c ⫽ cn,
i⫽1
c is a constant.
n n
2. 兺
i⫽1
cai ⫽ c 兺a ,
i⫽1
i c is a constant.
n n n
3. 兺
i⫽1
共ai ⫹ bi 兲 ⫽ 兺
i⫽1
ai ⫹ 兺b
i⫽1
i

n n n
4. 兺 共a ⫺ b 兲 ⫽ 兺 a ⫺ 兺 b
i⫽1
i i
i⫽1
i
i⫽1
i

Proof
Each of these properties follows directly from the properties of real numbers.
n
Infinite Series
1. 兺 c ⫽ c ⫹ c ⫹ c ⫹ . . . ⫹ c ⫽ cn
i⫽1
n terms
The study of infinite series was
considered a novelty in the
The Distributive Property is used in the proof of Property 2. fourteenth century. Logician
n Richard Suiseth, whose
2. 兺 ca ⫽ ca
i⫽1
i 1 ⫹ ca2 ⫹ ca3 ⫹ . . . ⫹ can nickname was Calculator,
solved this problem.
⫽ c共a1 ⫹ a2 ⫹ a3 ⫹ . . . ⫹ an兲 If throughout the first
n half of a given time interval a
⫽c 兺a
i⫽1
i variation continues at a certain
intensity; throughout the next
quarter of the interval at double
The proof of Property 3 uses the Commutative and Associative Properties of Addition.
n the intensity; throughout the
3. 兺 共a ⫹ b 兲 ⫽ 共a
i⫽1
i i 1 ⫹ b1兲 ⫹ 共a2 ⫹ b2 兲 ⫹ 共a3 ⫹ b3兲 ⫹ . . . ⫹ 共an ⫹ bn 兲 following eighth at triple the
intensity and so ad infinitum;
⫽ 共a1 ⫹ a 2 ⫹ a3 ⫹ . . . ⫹ an 兲 ⫹ 共b1 ⫹ b2 ⫹ b3 ⫹ . . . ⫹ bn 兲 The average intensity for the
n n whole interval will be the

i⫽1
兺 ai ⫹ 兺b
i⫽1
i intensity of the variation during
the second subinterval (or
The proof of Property 4 uses the Commutative and Associative Properties of Addition double the intensity).
and the Distributive Property. This is the same as saying that
n the sum of the infinite series
4. 兺 共a ⫺ b 兲 ⫽ 共a
i⫽1
i i 1 ⫺ b1兲 ⫹ 共a2 ⫺ b2 兲 ⫹ 共a3 ⫺ b3兲 ⫹ . . . ⫹ 共an ⫺ bn 兲
1 2 3 . . . n
⫹ ⫹ ⫹ ⫹ n⫹. . .
⫽ 共a1 ⫹ a 2 ⫹ a3 ⫹ . . . ⫹ an 兲 ⫹ 共⫺b1 ⫺ b2 ⫺ b3 ⫺ . . . ⫺ bn 兲 2 4 8 2
⫽ 共a1 ⫹ a 2 ⫹ a3 ⫹ . . . ⫹ an 兲 ⫺ 共b1 ⫹ b2 ⫹ b3 ⫹ . . . ⫹ bn 兲 is 2.
n n

i⫽1
兺a ⫺兺b i
i⫽1
i

Copyright 2011 Cengage Learning. All Rights Reserved. May not be copied, scanned, or duplicated, in whole or in part. Due to electronic rights, some third party content may be suppressed from the eBook and/or eChapter(s).
Editorial review has deemed that any suppressed content does not materially affect the overall learning experience. Cengage Learning reserves the right to remove additional content at any time if subsequent rights restrictions require it.
Proofs in Mathematics 633

The Sum of a Finite Arithmetic Sequence (p. 584)


The sum of a finite arithmetic sequence with n terms is given by
n
Sn ⫽ 共a1 ⫹ an 兲.
2

Proof
Begin by generating the terms of the arithmetic sequence in two ways. In the first way,
repeatedly add d to the first term to obtain
Sn ⫽ a1 ⫹ a2 ⫹ a3 ⫹ . . . ⫹ an⫺2 ⫹ an⫺1 ⫹ an
⫽ a1 ⫹ 关a1 ⫹ d兴 ⫹ 关a1 ⫹ 2d兴 ⫹ . . . ⫹ 关a1 ⫹ 共n ⫺ 1兲d兴.
In the second way, repeatedly subtract d from the nth term to obtain
Sn ⫽ an ⫹ an⫺1 ⫹ an⫺2 ⫹ . . . ⫹ a3 ⫹ a2 ⫹ a1
⫽ an ⫹ 关an ⫺ d 兴 ⫹ 关an ⫺ 2d 兴 ⫹ . . . ⫹ 关an ⫺ 共n ⫺ 1兲d 兴.
When you add these two versions of Sn, the multiples of d subtract out and you obtain
2Sn ⫽ 共a1 ⫹ an兲 ⫹ 共a1 ⫹ an兲 ⫹ 共a1 ⫹ an兲 ⫹ . . . ⫹ 共a1 ⫹ an兲 n terms

2Sn ⫽ n共a1 ⫹ an兲


n
Sn ⫽ 共a1 ⫹ an兲.
2

The Sum of a Finite Geometric Sequence (p. 592)


The sum of the finite geometric sequence
a1, a1r, a1r 2, a1r 3, a1r 4, . . . , a1r n⫺1
with common ratio r ⫽ 1 is given by
1 ⫺ rn
冢 1 ⫺ r 冣.
n
Sn ⫽ 兺ar
i⫽1
1
i⫺1 ⫽ a1

Proof
Sn ⫽ a1 ⫹ a1r ⫹ a1r 2 ⫹ . . . ⫹ a1r n⫺2 ⫹ a1r n⫺1
rSn ⫽ a1r ⫹ a1r 2 ⫹ a1r 3 ⫹ . . . ⫹ a1r n⫺1 ⫹ a1r n Multiply by r.

Subtracting the second equation from the first yields


Sn ⫺ rSn ⫽ a1 ⫺ a1r n.

So, Sn共1 ⫺ r兲 ⫽ a1共1 ⫺ r n兲, and, because r ⫽ 1, you have


1 ⫺ rn
Sn ⫽ a1 冢 1 ⫺ r 冣.

Copyright 2011 Cengage Learning. All Rights Reserved. May not be copied, scanned, or duplicated, in whole or in part. Due to electronic rights, some third party content may be suppressed from the eBook and/or eChapter(s).
Editorial review has deemed that any suppressed content does not materially affect the overall learning experience. Cengage Learning reserves the right to remove additional content at any time if subsequent rights restrictions require it.
634 Chapter 8 Sequences, Series, and Probability

The Binomial Theorem (p. 599)


In the expansion of 共x ⫹ y兲n
共x ⫹ y兲n ⫽ x n ⫹ nx n⫺1y ⫹ . . . ⫹ nCr x n⫺r y r ⫹ . . . ⫹ nxy n⫺1 ⫹ y n
the coefficient of x n⫺ry r is
n!
nCr ⫽ .
共n ⫺ r兲!r!

Proof
The Binomial Theorem can be proved quite nicely using mathematical induction.
(For information about mathematical induction, see Appendix G on this textbook’s The Principle of
Companion Website.) The steps are straightforward but look a little messy, so only an Mathematical Induction
outline of the proof is presented.
Let Pn be a statement involving
1. For n ⫽ 1, you have the positive integer n. If
共x ⫹ y兲1 ⫽ x1 ⫹ y1 ⫽ 1C0 x ⫹ 1C1y 1. P1 is true, and
and the formula is valid. 2. the truth of Pk implies the
2. Assuming that the formula is true for n ⫽ k, the coefficient of x k⫺r r y is truth of Pk⫹1 for every
positive integer k,
k! k共k ⫺ 1兲共k ⫺ 2兲 . . . 共k ⫺ r ⫹ 1兲
kCr ⫽ ⫽ . then Pn must be true for all
共k ⫺ r兲!r! r!
positive integers n.
To show that the formula is true for n ⫽ k ⫹ 1, look at the coefficient of x k⫹1⫺r y r
in the expansion of
共x ⫹ y兲k⫹1 ⫽ 共x ⫹ y兲k共x ⫹ y兲.
From the right-hand side, you can determine that the term involving x k⫹1⫺r y r is the
sum of two products.
共 kCr x k⫺r y r兲共x兲 ⫹ 共 kCr⫺1x k⫹1⫺ry r⫺1兲共 y兲

冤 共k ⫺ r兲!r! ⫹ 共k ⫹ 1 ⫺ r兲!共r ⫺ 1兲!冥x


k! k!
⫽ k⫹1⫺ry r

共k ⫹ 1 ⫺ r兲k!
冤 共k ⫹ 1 ⫺ r兲!r! ⫹ 共k ⫹ 1 ⫺ r兲!r!冥x
k!r
⫽ k⫹1⫺ry r

k!共k ⫹ 1 ⫺ r ⫹ r兲 k⫹1⫺r r
⫽ 冤 共k ⫹ 1 ⫺ r兲!r!
x y 冥
共k ⫹ 1兲!
⫽ 冤 共k ⫹ 1 ⫺ r兲!r!冥x k⫹1⫺ry r

⫽ k⫹1Cr x k⫹1⫺ry r
So, by mathematical induction, the Binomial Theorem is valid for all positive
integers n.

Copyright 2011 Cengage Learning. All Rights Reserved. May not be copied, scanned, or duplicated, in whole or in part. Due to electronic rights, some third party content may be suppressed from the eBook and/or eChapter(s).
Editorial review has deemed that any suppressed content does not materially affect the overall learning experience. Cengage Learning reserves the right to remove additional content at any time if subsequent rights restrictions require it.

You might also like